Sei sulla pagina 1di 340

Learnerators

SAT Evidence-Based Reading & Writing


Review Guide

Copyright 2015 by Learnerator Education, Inc.


All rights reserved. Neither this book nor any portion thereof may be reproduced
or used in any manner whatsoever without the express written permission.
Published in the United States of America.

Take your S AT prep to the next level. Visit www.learnerator.com


4

Table of Contents
Introduction ............................................................................................................................... 4
About Us ..................................................................................................................................... 5
The 11 Things You Need to Know about the New 2016 SAT Exam ........................ 6
SAT vs. ACT .............................................................................................................................. 9
Determining Explicit Meaning in Passages ................................................................. 19
Determining Implicit Meanings ...................................................................................... 28
Analogical Reasoning ........................................................................................................... 39
Citing Textual Evidence ...................................................................................................... 50
Determining Main Ideas .................................................................................................... 59
Summarizing ........................................................................................................................... 72
Understanding Relationships ........................................................................................... 83
Interpreting Words and Phrases in Context .............................................................. 102
Analyzing Word Choice ................................................................................................... 110
Analyzing Overall Text Structure ................................................................................ 117
Analyzing Part-Whole Relationships .......................................................................... 139
Analyzing Point of View ................................................................................................. 156
Analyzing Claims and Counterclaims ........................................................................ 173
Assessing Reasoning ........................................................................................................ 184
Analyzing Evidence ........................................................................................................... 193
Analyzing Quantitative Information .......................................................................... 214
Development ....................................................................................................................... 221
Organization ........................................................................................................................ 231
Precision and Concision .................................................................................................. 243
Style and Tone .................................................................................................................... 248
Syntax ..................................................................................................................................... 260
Sentence Formation ........................................................................................................... 267
Shifts in Construction ...................................................................................................... 272
Pronouns ............................................................................................................................... 276
Possessive Determiners ................................................................................................... 284

Take your S AT prep to the next level. Visit www.learnerator.com


5


Grammatical Agreement ................................................................................................. 291
Frequently Confused Words .......................................................................................... 300
Logical Comparisons ........................................................................................................ 310
Conventional Expressions ............................................................................................... 315
Conventions of Punctuation ........................................................................................... 323
Sign up for Learnerator SAT Prep! ............................................................................... 340

Take your S AT prep to the next level. Visit www.learnerator.com


6

Introduction
Hi there! We created this book to serve as an overview of all the major concepts that
will be asked of you in the new SAT reading section. This is a conglomeration of a
series of review posts we originally published on our blog. Whether youre new to
the SAT or a veteran, we hope youll find this book helpful in your preparation for
the exam!

-The Learnerator Team
E-mail us at hello@learnerator.com if you have any questions, comments, or
suggestions.

Take your S AT prep to the next level. Visit www.learnerator.com


4

About Us
Learnerator bridges the gap between learning and mastery by curating high quality
practice materials for a variety of difficult academic subject areas. Our online test
prep offers:

Over 300 SAT Integrated Reading & Writing practice questions

Over 300 SAT Math practice questions

Hundreds of practice questions for SAT IIs: Biology, Chemistry, French,


Literature, Math 1 & 2, Physics, US History, World History

Detailed personalized statistics based on your performance

A competitive online leaderboard to see how you stand compared to


others

Convenience with anytime access from an Internet-enabled device

Visit us at www.learnerator.com
Use SAT2016 for 10% off any Learnerator SAT Subject Guide



Take your S AT prep to the next level. Visit www.learnerator.com
5

The 11 Things You Need to Know about


the New 2016 SAT Exam
It's been a while since the College Board has revamped its flagship exam, the
SAT. You may be wondering what the new 2016 SAT exam will entail. Below are
a list of key differences between the old exam and the one being introduced in
Spring 2016:
1. Exam length. It's shorter (sort of)! The exam is being cut from a 3 hour, 45
minute exam to a 3 hour exam with an OPTIONAL "Essay" portion lasting
50 minutes.

2. Section reorganization. The "Writing" and "Critical Reading" sections
have been morphed into "Evidence-Based Reading and Writing" (EBR&W)
with two subsections for reading and writing & language.

3. Vocab. Remember the days when you'd have to memorize endless flash
cards with words like "adroit" and "celerity"? Probably not, but I do
because I am getting old (I took the SAT in 2007!). Now, instead of focusing
on rarely-used, arcane words, the new SAT focuses more on words that
you'd actually use in college and in your career. An example would be
"synergy", which is a common business word.

4. Essay. Since it is now optional, whether or not you take it will be based on
the requirements of the colleges to which you apply. The essay is also no
longer about expressing your views on an issue, but rather, analyzing a
source text of another person's argument using evidence from the passage.

5. Reading passages. The passages will have more diversity, with the
following weighting: 20% U.S. and World Literature, 40% History and
Social Studies, and 40% Science and Math. Each passage or paired set will
Take your S AT prep to the next level. Visit www.learnerator.com
6

7

be 500-750 words and the total length of all passages will be
approximately 3,250 words.

6. Graphical representations. The Reading & Writing section is going to
oriented around command of evidence in context (i.e., words in a history
vs. a science context have different meanings). This also applies to
graphical representations of facts, which is going to be emphasized more in
this exam.

7. Historical documents. Each exam's Evidence-Based Reading and
Writing section will feature either a passage from an important U.S.
historical document (like the Declaration of Independence), or from
landmark speeches or arguments from important people like JFK, Martin
Luther King, or Winston Churchill (they are not necessarily all Americans).
This may present a disadvantage to international students; so if that is you,
make sure to note this. Even though the College Board says no prior
knowledge is needed to answer these questions, it certainly helps to have
the context of the documents ahead of time.

8. The math section. The old math section of the SAT was a bit willy-nilly
and spanned many areas. The new Math section is more narrowly defined
and introduces a section called "Problem Solving and Data Analysis", which
introduces some basic concepts of statistics. For more on this, go to
our SAT Review page.

9. Calculator usage. There are now some portions of the new SAT that does
not allow for the use of calculators.

10. Delivery. The SAT will be available in both digital as well as paper forms.

Take your S AT prep to the next level. Visit www.learnerator.com


7

8

11. Scoring. Hooray! No more guessing penalty! You will be given points for
correct answers, but blank and incorrect answers will not earn you any
deductions. The scoring returns to 400-1600, from the current 600-2400
range. You are also going to get separate test scores and subscores, shown
below:

Summary: It

is interesting to note that 2013 was the first year that the ACT

surpassed the SAT in terms of the number of exams taken. The ACT has become
viewed as more representative of the skills necessary for college and career
success, and this new SAT is the College Board's response. The result is an SAT
that is closer to the ACT with perhaps an even greater emphasis on the ability to
analyze sources, which is also where the CB is taking their AP exams.

Take your S AT prep to the next level. Visit www.learnerator.com


8

SAT vs. ACT


Among the Internets most oft-Googled questions is: What is the difference between
the ACT and the SAT? Now that the SAT is changing, the answer will no doubt
become more nuanced.
Below is a table summarizing the differences and similarities between the two tests.
Read on for a more detailed explanation of whats in the table.

ACT

SAT

LOGISTICS

*Offered 6x/year

*Offered 5x/year

*$38.00 without essay

*$52.50 now (may change)

*$54.50 with essay


ACHIEVEMENT v.

*Measures what you've

*Measures how well you

APTITUDE

learned in school

are likely to do in school in


the future

ENGLISH

*75 multiple-choice

*44 multiple-choice

questions, 4 answer

questions, 4 answer

choices per question.

choices per question.

*How much have you

*How good are your

learned about

grammar/rhetoric skills?

grammar/usage?

*Called Writing and


Language, integrated with
reading

READING

*40 multiple-choice

*52 multiple-choice

questions, 4 answer

questions, 4 answer

choices per question

choices per question

*4-8 passages about

*4-6 passages about

science, humanities,

history/social studies,

history/social studies,

science, literature.

literature*Measures

*Measures reading

Take your S AT prep to the next level. Visit www.learnerator.com


9

10

reading comprehension

comprehension
*Contains charts, graphs,
visuals

SCIENCE

*40 multiple-choice

N/A

questions
*Data Representation,
Research Summaries, and
Conflicting Viewpoints.
*Charts/graph/visual
analysis
MATH

*60 questions, 5 answer

*57 questions, 4 answer

choices*All multiple choice choices

ESSAY

*Pre-algebra and Plane

*12 grid-ins

Geometry most heavily

*Algebra most heavily

represented

represented

*Can use calculator on

*Can use calculator on

whole test

most of test

*Optional

*Optional

*Give your opinion on a

*Analyze someone elses

topic*Scored out of 12

opinion on a topic
*Scored out of 8

SCORING

*Scored out of 36 (average *Scored out of 1600 (adds


of all sections)

both area scores)

*7 subscores

*7 subscores


The SATs redesign is due to a number of factors, including the fact that the ACT is
now slightly more popular and many say more fair, or at least a more accurate
predictor of college and career readiness. But even with the redesign, there are

Take your S AT prep to the next level. Visit www.learnerator.com


10

11

some key differences and some new key similarities.

LOGISTICS:

Timing
Both tests can be taken more than once, in either junior or senior year. The ACT is
administered six times a year in the United States (in September, October,
December, February, April, and June), while the SAT is administered 5 times (in
December, January, March, May, and June). The new SAT will be administered for
the first time in 2016.

For both the ACT and the SAT, you can take the test in your junior and senior year.
(In fact, we recommend taking the test twice if youre not thrilled with your original
junior-year score, as ACT Math covers typical 12th-grade topics and SAT Integrated
Reading and Writing will include some college-level passages so it will be helpful
to go into the test feeling more confident and prepared!)

Costs
The ACT costs $38.00 without the writing portion and $54.50 with the writing.
Currently, the SAT costs $52.50, though this might change now that the new SATs
Writing portion is optional.

OK, BUT WHATS THE REAL DIFFERENCE?

The common answer to this is that the SAT measures aptitude while the ACT
measures achievement.

Aptitude
Aptitude tests measure how well you are likely to perform a task based on the skills
you already have, or have developed over 12 years of formal education. Aptitude

Take your S AT prep to the next level. Visit www.learnerator.com


11

12

has been compared to talent or intelligence, and aptitude tests ask questions that
measure how you reason or think. (Of course, there are multiple ways to be talented
and intelligent, and multiple ways to nurture and extend these qualities which is
one reason that the SAT, which measures and privileges only a few types of aptitude,
has drawn so much criticism.)

Achievement
On the other hand, achievement tests, like the ACT, measure how well you
can already perform certain tasks based on what you have already learned over 12
years of formal education. They measure skills or knowledge, and as such have been
called a more accurate predictor of whether students are ready for college and the
workplace.
However, now that the SAT is changing, it will have much more in common with the
ACT than it did previously. The debate over aptitude versus achievement is a hot
topic in education, and well beyond the scope of this post. The essential thing to
keep in mind is this: if you work hard in school and out of it, you can and will learn
the skills that are essential to succeeding on both the SAT and the ACT, as well as in
college, the workplace, and life.

Take your S AT prep to the next level. Visit www.learnerator.com


12

13

OK, BUT WHAT DOES THIS ACTUALLY MEAN FOR ME?

Unlike the new SAT, which is divided into two mandatory and one optional section,
the ACT is separated into four mandatory portions and one optional section. Those
portions are:

ACT ENGLISH which features 75 questions about a variety of strategically messed
up passages to measure a students ability to recognize, diagnose, and correct
problems of grammar and usage, punctuation, sentence structure, strategy,
organization, and style. In other words, the ACT English test measures how much
youve learned about the conventions of the English language and strategies for
good writing. There are four answer choices per question.

ACT READING uses four to eight passages and forty questions to measure your
ability to comprehend main ideas, significant details, implied and stated ideas,
specific vocabulary words, and text structures like comparison and sequence of
events. The passages might be single, long selections or paired, short selections. The
test promises that this is the type of reading required in first year college courses,
and the topics center on social studies, humanities, natural sciences, and literary
narratives. In other words, ACT Reading measures how well you have learned to
read a diverse array of texts over the past 11 years. There are four answer choices
per question.

Now that the SAT has collapsed its Reading and Writing sections into one integrated
section (Evidence-Based Reading and Writing) its structure is remarkably similar
to ACT English and ACT Reading. SAT Evidence-Based Reading and Writing is
now comprised of 44 Writing and Language questions and 52 Reading questions
about 4-6 passages (most long, but with some paired shorter selections) across
topics in history and social studies, science, and US and world literature. SAT
EBRW measures your understanding of words in context (no more of those pesky
SAT words like adamantine or vitreous), your command of evidence, and your
Take your S AT prep to the next level. Visit www.learnerator.com
13

14

ability to analyze text structure, assess implied and explicit meanings, and
understand purpose. The new SAT EBRW section will now have four answer choices
per question instead of five. Notably, the new SAT EBRW section will include charts
and graphs; understanding and analyzing visuals is a type of literacy, or reading,
too!

The ACT Reading section does not offer charts or graphs, because there is a whole
section dedicated almost exclusively to visual data. ACT Science is its own 40-
question section, comprised of Data Representation, Research Summaries, and
Conflicting Viewpoints. This section doesnt actually measure how much you know
about science. Instead, it measures how well you can understand, analyze, and
synthesize scientific data how well you can read a chart or a graph, how deeply
you understand two different ideas about scientific concepts, whether or not you
can think critically and make an inference about the data in front of you. Each
answer has four options.

Last, ACT Math has sixty questions with five options each (not four like on all the
other sections of the test) about topics that range from pre-Algebra through
Trigonometry. The most heavily represented subjects on the test are Pre-Algebra
and Plane Geometry. The ACT measures your mathematical reasoning skills as well
as much youve absorbed in your math classes over the past 12 years. You will be
able to use a calculator on every question on the test.

The SAT Math section has 57 questions total, 45 with four answer choices and 12
Student-Produced Responses (i.e. grid-ins), one of which involves showing your
work and demonstrating your thinking in response to a word problem. Of the four
content domains (Heart of Algebra, Passport to Advanced Math, Problem Data
Analysis, and Additional Topics), Heart of Algebra is the biggest, so make sure you
know your algebra before you take this test. Also, the new SAT will now present
item sets a group of related questions that ask you to respond to the same set of
data/graphs/stimuli. You wont be able to use your calculator for 20 questions on
Take your S AT prep to the next level. Visit www.learnerator.com
14

15

the SAT Math test.

Perhaps the biggest difference between the two tests is the Optional Essay. The
ACT will give you two perspectives on a common topic relevant to high school
students (such as curfews and drivers licenses) and ask you to contribute your
opinion. You will have to demonstrate that you understand both perspectives, that
your opinion is based in evidence and reasoning, and that your writing conforms to
standard English conventions.

The SATs optional essay will ask you to analyze a source text (which will have
very little to do with common topics relevant to high school students) and figure out
what the writer is saying and how s/he builds an argument. You will have to prove
that you understand how a writer constructs an effective argument, using evidence
from the text. The SAT is NOT looking for your opinion, but for how adroitly you
marshal and cite evidence, and how fluently you write.

These are very different types of essays, so be sure you have plenty of practice for
both.

Take your S AT prep to the next level. Visit www.learnerator.com


15

16

HOW WILL MY TEST BE SCORED?

SAT Scoring
The SAT is going back to its original 1600-point scale (#oldschool), which means
that the highest score you can earn is a 1600 (composite) with an 800 EBRW score
(area score) and an 800 Math (area score). However, it gets a little more
complicated after this:

You get 3 test scores (from 10-40) for (1) Reading, (2) Writing and Language, and
(3) Math.

You get 2 cross-test score (from 10-40) for (1) Analysis in Science and (2) Analysis
in History/Social Studies.

You get 7 subscores (from 1-15) for (1) Command of Evidence and (2) Relevant
Words in Context (Reading), (3) Expression of Ideas and (4) Standard English
Conventions (Writing and Language), (5) Heart of Algebra, (6) Problem Solving and
Data Analysis, and (7) Passport to Advanced Math.

Essays are scored from 2-8, with two readers who will assign you a score of 1-4.

ACT Scoring
On the ACT, you will receive a Composite Score (out of 36) that averages each of
your four test scores in Math, English, Reading, and Science. The highest score you
can receive on each section as well as on the test itself is a 36.

You will receive seven subscores: (1) Usage and Mechanics, (2) Rhetorical Skills,
(3) Pre-Algebra and Elementary Algebra, (4) Intermediate Algebra/Coordinate
Geometry, (5) Plane Geometry/Trigonometry, (6) Social Studies/Natural Sciences
reading skills, and (7) Arts/Literature reading skills.

Take your S AT prep to the next level. Visit www.learnerator.com
16

17

These are scored from 1-18, and have no mathematical bearing on your composite
score.
Your essay is scored out of 12, with two readers giving you a score between 1-6.

Take your S AT prep to the next level. Visit www.learnerator.com


17

18







Part I: Information & Ideas (Reading)

Take your S AT prep to the next level. Visit www.learnerator.com


18

19

Determining Explicit Meaning in Passages


Determining explicit meanings is the College Boards highfalutin way of
saying, reading the text closely and understanding what it says. All the SAT
wants you to do is:
1) Pay

attention to detail, and

2) Understand

all the information that is already there.

This applies both to the passage and to the question and all of the answer choices.
The beauty of the multiple-choice test is that all the answers are already there;
you just have to know where to look.
This means you have to consider every scrap of information the College Board
gives you. The little blurb before each passage is crucial they are giving you free
information about

the passage, which will help you understand the passage better

and answer questions about it.


There are two equally important ways to go about this.

1. Annotate the text.


Underline, star, or make notes next to all the information that seems like it might
be relevant. For example, take the blurb and the first paragraph of this passage,
taken directly from the SAT website:
This passage is adapted from a novel written by a woman in 1899. The novel was
banned in many places because of its unconventional point of view:

Take your S AT prep to the next level. Visit www.learnerator.com


19

20

It was eleven o'clock that night when Mr. Pontellier returned from Klein's hotel. He was in an
excellent humor, in high spirits, and very talkative. His entrance awoke his wife, who was in bed
and fast asleep when he came in. He talked to her while he undressed, telling her anecdotes and
bits of news and gossip that he had gathered during the day. From his trousers pockets he took a
fistful of crumpled bank notes and a good deal of silver coin, which he piled on the bureau
indiscriminately with keys, knife, handkerchief, and whatever else happened to be in his pockets.
She was overcome with sleep, and answered him with little half utterances.

The first thing youll want to do is circle all the important information:

a)

This is a novel (ok, so fiction that means youll have to be prepared for

questions on characterization, plot, literary devices)


b)

Written by a woman in 1899 (ok, so old-fashioned, probably be prepared for

some old-fashioned language)


c)

Banned in many places for being unconventional (which undoubtedly means

there will be a question about something related to this because the College
Board wouldnt drop that juicy morsel unless it had a reason so this might
contain themes considered unconventional in 1899)
See how much information you can glean just from the little blurb? Now turn
your attention to the passage. Take note of:

1)

What time it was when Mr. P came home (pretty late! 11pm, and his wife was

fast asleep!)
2)

Where Mr. P was coming from (a hotel why was he at a hotel at 11 pm

without his wife? Work event? Party?)


3) Mr.

Ps mood (clearly good something good must have happened. Take

special note of this because the SAT loves to ask about tone and mood!)
4) Other

thoughts: so his wife is in bed asleep at 11 pm but he decides this is the

Take your S AT prep to the next level. Visit www.learnerator.com


20

21

time to tell her about anecdotes and bits of news and gossip? Take note of how
she is reacting to him, too.
Annotations arent the only way to record your observations of the text. Its also
crucial that you adopt a second strategy:

2. Translate the text.


This wont always be necessary, but the more complex, old-fashioned, or
specialized a passage is, the more likely you will need to translate at least parts of
it into plain English. You can do this while you annotate, to save time.
Lets look at the passage again, this time with some translations and annotations
in red:

It was eleven o'clock that night when Mr. Pontellier returned from Klein's hotel. He was in an
excellent humor (good mood), in high spirits, and very talkative. His entrance awoke his wife, who
was in bed and fast asleep when he came in. He talked to her while he undressed, telling her
anecdotes (stories) and bits of news and gossip that he had gathered during the day. From his
trousers pockets he took a fistful of crumpled bank notes and a good deal of silver coin (lots of
money), which he piled on the bureau indiscriminately with keys, knife, handkerchief, and
whatever else happened to be in his pockets (piling all of his stuff up so hes messy). She was
overcome with sleep (tired), and answered him with little half utterances (halfsentences/incoherent).

When it comes to translating the text, sometimes youll just get stuck on a word.
Say you didnt know anecdotes in the fourth line. You probably know bits of
news and gossip, though, so its possible to figure out from context that hes
giving her a very chatty, thorough account of his day. You wont need to know
what every word means to figure out a general picture from the text, or even to
do a translation.
Take your S AT prep to the next level. Visit www.learnerator.com
21

22

Now that youve annotated and translated the text, youll want to turn your
attention to the question, and do the same, making sure you understand
completely what is being asked.
Question: Mr. Pontelliers conduct in this passage could most likely be
described as
A) Generous
B) Good-natured
C) Passionate
D) Self-centered

Remember to annotate and/or translate. What is the question asking you?


Mr. Pontelliers conduct (behavior) in this passage could most likely be
described as
So theyre asking about his behavior. Based on the few things we know about
him he got home late, hes very chatty though his wife is asleep, hes in a good
mood its pretty easy to answer this question, so lets consider each answer
choice.
Option A, Generous.

Is there any evidence to support this?

Yes, theres mention of the money in his pockets, which may serve to distract
you, but the passage doesnt say anything about him giving away or otherwise
doing anything with that money. Cross out A.
Option B, Good-natured.

Well, he is in an excellent humor, (good mood) but that

doesnt necessarily make him good-natured. And remember the question asks

Take your S AT prep to the next level. Visit www.learnerator.com


22

23

about his behavior that evening, not his general personality. This isnt a terrible choice,
but leave it for now and consider the others.
Option C, Passionate.

Is there any evidence that hes being passionate? Certainly he

seems happy and excited, but theres little evidence of passion especially as
passion generally tends to be directed towards something or someone, whereas a
good mood is general. Cross that one off, and then move on to the last answer.
Option D, Self-centered.

Remember that the question is asking about how heacted that

night. Think back to all the actions you noted when you were reading the text: he
came in late, woke his sleeping wife with his chatter, and piled all his stuff
everywhere in their shared room. Though he may befeeling good-natured (B),
hes acting pretty self-centered. Clearly, that is the only correct answer to what the
College Board is asking here. All you need to do is pay attention to detail.
-------------Thats a fairly simple passage, though, so lets do this once more with a slightly
harder passage.
The following passage is from the same book.
An indescribable oppression, which seemed to generate in some unfamiliar part of her
consciousness, filled her whole being with a vague anguish. It was like a shadow, like a
mist passing across her soul's summer day. It was strange and unfamiliar; it was a mood.
She did not sit there inwardly upbraiding her husband, lamenting at Fate, which had
directed her footsteps to the path, which they had taken. She was just having a good cry
all to herself. The mosquitoes made merry over her, biting her firm, round arms and
nipping at her bare insteps.

The little stinging, buzzing imps succeeded in dispelling a mood which might have held
her there in the darkness half a night longer.

Take your S AT prep to the next level. Visit www.learnerator.com


23

24

Again, lets start annotating and translating:

An indescribable oppression (really bad feeling), which seemed to generate (come


from) in some unfamiliar part of her consciousness (unfamiliar part of her consciousness
so she doesnt know why shes unhappy, or where the unhappiness is coming from
maybe her subconscious?), filled her whole being with a vague anguish.(She was sad) It
was like a shadow, like a mist passing across her soul's summer
day. (Imagery/sadness) It was strange and unfamiliar; it was a mood. She did not sit
there inwardly upbraiding (criticizing) her husband, lamenting (being sad at) at Fate,
which had directed her footsteps to the path which they had taken (so shes NOT sitting
there criticizing her husband and hating her fate? This is an interesting thing for the
narrator to tell us And its interesting that she would even think in terms of fate,
instead of her own decisions, directing her footsteps.) She was just having a good cry
all to herself. (as one does late at night) The mosquitoes made merry over her (mosquito
bloodsucking party), biting her firm, round arms and nipping at her bare insteps (feet).

The little stinging, buzzing imps (ie mosquitos) succeeded in dispelling a mood (they got
rid of her bad mood?!) which might have held her there in the darkness half a night
longer. (guess they made her so itchy she forgot to be sad?)

On the actual test, you wont necessarily have time to write yourself insanely
detailed notes like I did here. Thats OK! Instead of writing really bad feeling
or making notes about the unfamiliar part of her consciousness you can use
shorthand a little frowny face, a question mark, an underline, an exclamation, a
star.
And its OK if you dont know all the words, like upbraiding or lamenting.
The rest of the passage with its words like oppression, anguish, shadow,
mist passing across her soul, good cry should serve to cement the
impression that whatever upbraid or lament might mean, its not good.

Take your S AT prep to the next level. Visit www.learnerator.com


24

25

So now we can turn our attention to a question based on this passage:
By depicting in such detail Mrs. Pontelliers emotional state, the narrator is trying
to suggest that
A) Mr.

Pontellier is worldly and callous

B) Mrs.

Pontellier is clearly upset about something that happened before Mr.

Pontellier came back home


C) Mrs.

Pontellier does not enjoy being a mother

D) Mrs.

Pontellier is unused to confronting her own unhappiness

Remember to really think about what the test is asking you. The question asks
about the narrators intentions here in showing you Mrs. Pontelliers state
i.e., Based on this unhappy episode, what does the narrator want to tell you about this character?
You can easily answer this question based on explicit textual evidence.
Consider option A. Mr. Pontellier may indeed be worldly and callous (not very
nice), but there is no evidence from this particular part of the passage (which is
all the question asks about!) that this is why his wife is so upset. You can easily
cross this one out remember, you only want to look for what the text
says explicitly, and here the question asks only about one particular part of the
passage, in which Mr. Pontellier is NOT described in any detail. (In fact, all it
says about him is that Mrs. Pontellier did NOT use this time to criticize him in
her mind but it gives no specific descriptions as to his personality.)
Next, try option B. As with option A, this MIGHT be a possibility except that
there is absolutely no indication of this in the actual text. It says nothing about

Take your S AT prep to the next level. Visit www.learnerator.com


25

26

what happened before Mr. Pontellier came home, so you can cross this one out
immediately.
Look at option C. Again, remember to look only at the part of the text that the
question is asking about. There is no mention about being a mother in this part of
the text, and in fact no mention of children at all. This clearly cannot be the
answer.
Now we come to option D: Mrs. Pontellier is unused to confronting her own
unhappiness. Consider the evidence here, because the text has pretty much told
you that this is the case:
She feels indescribably oppressed (as though she lacks the words/experience
to describe it presumably, if she was very familiar with and deeply aware of
this feeling, she would know exactly how to describe it and what the reasons for
it are).
Her unhappiness generates from an unfamiliar part of her consciousness i.e.,
she has no idea where it came from. And if she doesnt know WHY shes
unhappy, then this is pretty good evidence that she doesnt think a whole lot
about her own unhappiness. Clearly, this is the only reasonable choice.
Now, a critical reader will no doubt make the connection between Mrs.
Pontelliers anguished crying and her inconsiderate husband. A good critical
reader might also connect this to the blurb before the passage, which tells you
that this book had an unconventional point of view in 1899 a point of view
that will no doubt have something to do with the fact that the marriage between
the main characters seems very unfulfilling.

Take your S AT prep to the next level. Visit www.learnerator.com


26

27

These critical reading skills will be very helpful in the next chapter, Determining
Implicit Meanings.

Take your S AT prep to the next level. Visit www.learnerator.com


27

28

Determining Implicit Meanings


The College Board defines determining implicit meanings as drawing
reasonable inferences and logical conclusions from the text. If determining
explicit meanings is figuring out what the text says, then determining implicit
meanings is figuring out what the text suggests or implies.
This sounds difficult, and in some cases it can be. But there are tricks to drawing
inferences and reaching conclusions, and they are:
1) Read closely.

Just as you do when youre trying to figure out an explicit meaning, youll
want to take notes, translate, and pay attention to what the text is saying.
Understanding the surface meaning of a text will help you understand it on a
deeper level, too.
2) Find evidence.

Every time you choose an answer, match up your choice with actual evidence
(meaning, a quote) from the text. If you cant do that, or if the evidence youve
chosen doesnt convince you, then you are probably not making a supportable
conclusion and you should pick another answer.
Try this strategy with the following passage from the Public Library of Science
blog:

As Ebola rears its ugly head in the U.S., there has been a lot of discussion about how afraid we
really should be. While health officials have remained relatively calm, insisting in a tense

Take your S AT prep to the next level. Visit www.learnerator.com


28

29

congressional hearing that there is no need to panic, some members of the American public have
gone in the opposite direction, proclaiming a state of emergency on social media channels such as
Twitter and Facebook. Donald Trump has been an especially noisy voice on the topic on Twitter,
demanding that Obama stop all flights to and from West Africa and proclaiming that Ebola is
actually much more contagious than the CDC says it is. At the end of September, with the
announcement of the first suspected case of Ebola in the U.S., Trump expressed panic on Twitter,
writing:

"The United States must immediately institute strong travel restrictions or Ebola will be all over
the United States--a plague like no other! Ebola is much easier to transmit than the CDC and
government representatives are admitting. Spreading all over Africa - and fast. Stop flights."

Trumps comments suggest not only a distrust of health authorities but also a kind of hysteria,
with the use of the word plague, that is reminiscent of the infectious disease panic seen in many
other outbreaks in American history, including SARS in 2003 and bubonic plague in 1924, the
latter an outbreak that famously led to the frantic firing of thousands of Latino workers and
destruction of homes in low-income Latino neighborhoods because they were seen as the
source of the disease.

The first thing you will want to do is annotate your text. Make notes next to
anything that seems interesting, unusual, or otherwise notable. And if theres
something you dont understand, translate it for yourself into plain English as
best as you can. My translations and comments are in red:

As Ebola rears its ugly head in the U.S., (negative) there has been a lot of discussion about how
afraid we really should be. While health officials have remained relatively calm, insisting in
a tense congressional hearing that there is no need to panic, some members of the American
public have gone in the opposite direction, proclaiming a state of emergency on social media
channels such as Twitter and Facebook. (debate here) Donald Trump has been an especially
noisy (negative-sounding) voice on the topic on Twitter, demanding that Obama stop all flights to
and from West Africa and proclaiming that Ebola is actually much more contagious than the CDC
says it is. At the end of September, with the announcement of the first suspected case of Ebola in
the U.S., Trump expressed panic on Twitter, writing:

"The United States must immediately institute strong travel restrictions or Ebola will be all over

Take your S AT prep to the next level. Visit www.learnerator.com


29

30

the United States--a plague like no other! Ebola is much easier to transmit than the CDC and
government representatives are admitting. Spreading all over Africa - and fast. Stop flights."

Trumps comments suggest not only a distrust of health authorities but also a kind of
hysteria, (negative) with the use of the word plague, that is reminiscent of the infectious disease
panic seen in many other outbreaks in American history, including SARS in 2003 and bubonic
plague in 1924, the latter an outbreak that famously led to the frantic firing of thousands of Latino
workers and destruction of homes in low-income Latino neighborhoods because they were seen as
the source of the disease.(Trump is panicking in the same way that people panicked in 1924,
which led to thousands racially-based firings negative).

These are just preliminary notes, but they will help us answer the next two
questions using the second strategy, matching up every option with a quote /
evidence from the text.

How does the author of this passage most likely feel about Trumps comments on Twitter?

A. She finds Trumps comments laudable and important.


B. She finds Trumps comments irrelevant and pointless.
C. She is distrustful of Trumps comments.
D. She is subtly disdainful of Trumps comments.

Now, the author never explicitly expresses an opinion about Trump. However, if
you read closely, you can answer this question by matching up each option with
evidence from the text.
Consider A: She finds Trumps comments laudable and important.
Even if you dont know that laudable means worthy of praise, you can
figure out that important is a positive thing. Comb over the text. Is there any
mention that the author finds Trumps contribution important, other than the
fact that she is mentioning him? Theres no evidence of this. In fact, all of the

Take your S AT prep to the next level. Visit www.learnerator.com


30

31

words that she uses to describe Trumps contributions are NEGATIVE she calls
him noisy and says he is demonstrating hysteria, neither of which are
positive words. Go ahead and cross out Option A.
Now, Option B Again, look at these words: irrelevant and pointless. Both are
negative, which matches with the negative tone the author uses to describe
Trumps comments. But is there any evidence that she finds Trump irrelevant?
Surely if he was pointless, the author wouldnt mention him at all. Nor would
she say that Trumps comments suggest a kind of hysteria reminiscent of
the infectious disease panic in 1924 that famously led to the frantic firing of
thousands of Latino workers and destruction of homes in low-income Latino
neighborhoods. In fact, Trumps comments are the opposite of irrelevant if
panics like his have led to people being fired and homes being destroyed, then
clearly words like his have some resonance, even if they are a negative rather
than a positive part of this national conversation.

Clearly, Option B cant be correct.

Check out Option C and try to find evidence for the fact that the author is
distrustful of Trumps comments. True, the word distrustful shows up in the
passage although the passage suggests that it is Trump who is distrustful of
authorities, not the author. True, the author seems like she does not agree with
Trump, calling him noisy and hysterical and subtly suggesting that his
comments could lead to widespread panic, firing, and destruction of homes. So
this might be a good answer there is some evidence here.
Lets consider Option D and see if there is more or better evidence: The author is
subtly disdainful i.e. she has a negative impression of Trumps comments.
Clearly, by describing Trump as noisy, hysterical, and having an attitude

Take your S AT prep to the next level. Visit www.learnerator.com


31

32

reminiscent of widespread panics that led to firings and destructions of homes,
the author is demonstrating a disdain of Trump.
So its between C and D. Which word better describes the authors attitude? Is
there evidence that she doesnt trust Trump, or is there more evidence that she
thinks negatively of him?
While C may be a possible answer, D is a better answer because there is
more evidence

for it. Which brings us to Strategy #3 for Determining Implicit

Meanings:

When in doubt, go with the option that has more evidence.

---------------------------------Lets try one more example from the same article:

At the same time, a good deal of the coverage of the Ebola crisis in West Africa has focused on the
irrationality and ignorance of West Africans, choosing to visit traditional healers instead of
doctors and ignoring warnings from health officials that traditional burial rituals can hasten the
spread of the disease. Indeed, this lack of faith in Western medicine is now being addressed by
local and foreign health officials who are running educational programs in places like Liberia
and Sierra Leone in order to ensure that the people exposed to the illness have the correct
scientific information.

But while certain cultural practices and lack of education have certainly played a role in the rapid
spread of Ebola in West Africa, the amount of attention we have paid to the ignorance and
irrationality of people living in these developing countries has not been balanced with an equal
amount of attention to our own developed nations manifestations of similar irrational health
beliefs. Are we really that much more knowledgeable and rational than our West African
counterparts, or do our irrationalities and psychological missteps simply take different cultural
forms? And do our ignorance levels and irrationalities have the potential to be just as dangerous
to our health as seeking out a local healer instead of a trained healthcare professional in response
to a life-threatening viral illness?

Take your S AT prep to the next level. Visit www.learnerator.com


32

33

Remember, annotate and translate anything that sticks out to you:

At the same time, a good deal of the coverage of the Ebola crisis in West Africa has focused on
the irrationality and ignorance of West Africans, choosing to visit traditional healers instead
of doctors and ignoring warnings from health officials that traditional burial rituals can hasten
the spread of the disease. Indeed, this lack of faith in Western medicine is now being
addressed by local and foreign health officials who are running educational programs in places
like Liberia and Sierra Leone in order to ensure that the people exposed to the illness have the
correct scientific information.

But while certain cultural practices and lack of education have certainly played a role in the
rapid spread of Ebola in West Africa, the amount of attention we have paid to the ignorance
and irrationality of people living in these developing countries has not been balanced with an
equal amount of attention to our own developed nations manifestations of similar irrational
health beliefs. Are we really that much more knowledgeable and rational than our West African
counterparts, or do our irrationalities and psychological missteps simply take different cultural
forms? And do our ignorance levels and irrationalities have the potential to be just as dangerous
to our health as seeking out a local healer instead of a trained healthcare professional in
response to a life-threatening viral illness?

As you can see, I bolded a lot of information all because it stood out to me in
some way. The words in quotes hold a lot of interest clearly, the author is
trying to communicate that SOMEONE believes that West Africans are
irrational and ignorant, but by putting the words in quotes, she is
demonstrating distance from that opinion. I also bolded now being addressed
because this lack of faith in Western medicine is clearly important enough that
local AND foreign health officials feel the need to address it.
Next, I bolded parts of the first sentence of the next paragraph, paying special
attention to those words but while transition words like BUT, HOWEVER,
WHILE, and YET (among others) always demonstrate a shift in thinking or a

Take your S AT prep to the next level. Visit www.learnerator.com


33

34

contradiction, and I wanted to be prepared to answer a question about this shift
in thinking.
In translation, the author is saying There has been some ignorance in West
Africa, which helps spread Ebola, but we (meaning, Americans) have not paid
attention to our own ignorance. Now the purpose of the article shifts slightly to
OUR ignorance and irrationalities, rather than West African ignorance and
irrationalities. She finished by saying that OUR ignorance levels can be
dangerous to our health.
See if you can use our strategies (read closely, annotate, find evidence) to answer
the following question:

Based on the passage, the author would most likely agree with which of the following aphorisms?

A. Dont count your chickens before they are hatched.


B. Dont throw stones if you live in a glass house.
C. Be the change you wish to see in the world.
D. Even if sickness distracts from thoughts, all that is needed is the will to love.

Even if you dont know what the word aphorism means (it is a fancy word for
common saying), chances are that you recognize these idiomatic English
expressions. This question will require you to make some inferences based on the
passage because clearly, the author never uses any of these sayings. So lets
examine them one by one:
Option A: Dont count your chickens before theyre hatched means dont
make plans based on things that havent happened yet. Skim the text. What
evidence is there that the author agrees with this advice? Other than the fact that
this is good general advice, the author makes no mention of plans or hopes for
the future. Cross this one off.

Take your S AT prep to the next level. Visit www.learnerator.com


34

35

Option B: Dont throw stones if you live in a glass house means Dont criticize
others for faults that you have yourself. Skim the text. Is there mention of
criticism? Yes! We have criticized West Africans for being irrational and
ignorant while ignoring our own missteps and irrationalities that are just as
dangerous to our health. There is certainly evidence that the author thinks that
the American public is throwing stones despite living in a glass house made of
our own irrationalities and ignorance. Keep this as an option and consider the
next option:
Option C: Be the change you wish to see in the world. This can be interpreted
as If you want to see a change in the world, then you yourself should try and
make this change. Look at the text. Is the author advocating for a specific
change? Maybe by asking And do our ignorance levels and irrationalities have the potential to
be just as dangerous to our health she

might be subtly advocating that we stop being

ignorant and irrational. But the general tone of the article isnt focused on
making changes. Instead, the point of the article seems to be more focused on
illustrating a phenomenon rather than giving a directive (in this case: stop or
change). Its still an option, but the evidence isnt quite rock-solid.
Consider Option D: This one mentions sickness (which is one of the topics of the
article). But the author never says anything about the will to love. You could
make a very weak argument that by saying that Americans can be as irrational as
West Africans when it comes to health, the author is advocating that Americans
start loving their West African counterparts because we all have the same
irrationalities.
You COULD make this argument. But read it back to yourself. It sounds pretty
flimsy. Cross off D.

Take your S AT prep to the next level. Visit www.learnerator.com


35

36

Now youre left debating between B and C. Which has the most evidence? Is the
author advocating for change here, or is she pointing out that Americans can be
pretty hypocritical in thinking West Africans are irrational when the American
public has the potential to be just as irrational? Clearly, B has the most evidence,
and therefore can be the only right answer.
---------------------------------Lets try this one more time, using the same passage.
At the same time, a good deal of the coverage of the Ebola crisis in West Africa has focused on
the irrationality and ignorance of West Africans, choosing to visit traditional healers instead of
doctors and ignoring warnings from health officials that traditional burial rituals can hasten the spread of
the disease. Indeed, this lack of faith in Western medicine is now being addressed by local and foreign
health officials who are running educational programs in places like Liberia and Sierra Leone in order
to ensure that the people exposed to the illness have the correct scientific information.

But while certain cultural practices and lack of education have certainly played a role in the
rapid spread of Ebola in West Africa, the amount of attention we have paid to the ignorance
and irrationality of people living in these developing countries has not been balanced with an
equal amount of attention to our own developed nations manifestations of similar irrational
health beliefs. Are we really that much more knowledgeable and rational than our West African
counterparts, or do our irrationalities and psychological missteps simply take different cultural
forms? And do our ignorance levels and irrationalities have the potential to be just as dangerous
to our health as seeking out a local healer instead of a trained healthcare professional in
response to a life-threatening viral illness?

Which of the following American practices would the author most likely deride as a psychological
misstep akin to an Ebola patient visiting a local healer?

A. Thinking that West Africans are irrational


B. Panicking about Ebola
C. Refusing a flu shot
D. Staying up-to-date on correct scientific information

Take your S AT prep to the next level. Visit www.learnerator.com


36

37

To answer this question, you first need to understand it. There are two blocks
here the fact that the question uses the word deride (which means criticize),
and the fact that the question is so long and could potentially be confusing. So,
even if youre not sure of the word deride, try your best to translate the
question. It is using terminology from the text (psychological misstep) to
ask: Which of the following American practices would the author most likely deride (criticize) as a
psychological misstep (irrational/bad thing) akin to (like) an Ebola patient visiting a local healer? I.e.,

Which AMERICAN practice would the author think is AS irrational as visiting a


local healer to cure Ebola? In other words, which action is irrational AND
potentially dangerous to public health? The key here are the words misstep
and akin clearly, the answer will be negative (misstep) in a similar way (akin
= comparison word) as thinking Ebola can be cured by a local healer.
Looking at the answer options, you can cross one off right from the start, because
the answer will clearly be negative and at least one of the answers (D) is positive.
Staying up to date on correct scientific information (which is a quote from the
text, but dont let that fool you!) is clearly a POSITIVE thing, not a dangerous,
irrational one.
Now look at Option A: Thinking that West Africans are irrational. The question
is asking: what is an irrational thing that Americans do? While there is evidence
from the text that Americans view West Africans as irrational, and that this view
is wrong, is there evidence from the text to support the second part of the
question that this behavior is akin to visiting a local healer to cure Ebola (i.e.
dangerous to public health)? Think back to the earliest part of the text, in which
the author subtly criticized Donald Trump for potentially inciting mass hysteria
that could lead to firing and destruction of homes. That is both irrational AND
problematic/destructive so for now, lets keep Option A. Theres some
evidence for it.

Take your S AT prep to the next level. Visit www.learnerator.com


37

38

Try Option B next: Panicking about Ebola. Is panicking about Ebola irrational?
The author suggests that it is, by saying in the very first paragraph health
officials have remained relatively calm, which suggests that the average
American should probably be a bit calmer too. So yes, the author would
probably agree that panicking is irrational. But is it as irrational as treating Ebola
incorrectly and allowing it to spread (as suggested by that akin)? Probably not.
You can go ahead and cross this off based on lack of evidence.
Now Option C: Refusing a flu shot. The author doesnt explicitly mention flu in
this article. But the flu, like Ebola, is a disease, and refusing to treat a disease is a
fairly irrational thing to do especially because like Ebola, the flu is contagious.
The key to determining whether this is the right answer lies in that akin
which of the options are actively dangerous in the same way as not treating Ebola
properly? Clearly, the best and most likely answer is C: both actions (not treating
Ebola OR the flu properly) are irrational AND allow a disease to spread in a
dangerous way.
True, we had some evidence for Option A, but the scenario in Option C
corresponds most directly with the scenario outlined in the text, so thats the best
answer there is more evidence for it based on what the text says.
This is a form of analogical reasoning, which is a skill you will develop further
in the next section.

Take your S AT prep to the next level. Visit www.learnerator.com


38

39

Analogical Reasoning
K, pop quiz:

Analogies : SAT :: VCRs : ____________________
A, Entertainment
B, DVDs
C, Theaters
D, Limited-edition boxed sets

Give up?

At the risk of sounding old, analogies were huge on the SAT Verbal section when I
was a wee lass, nervously sharpening my #2 pencil before the day of the big test. (By
the way, the answer to that quiz is A just as this form of analogical reasoning is no
longer relevant to the SATs, VCRs are no longer relevant to the wider world of
entertainment.)

Now that analogies have gone the way of pogs, dinosaurs, and the VCR, you may
think that you dont need to worry about them anymore.

Unfortunately, though the question format has changed, the SAT will still test you on
whether or not you can think analogically: whether you can deftly and accurately
compare one thing to another, whether you can draw conclusions based on
similarities and differences, whether you can logically apply information from one
scenario into another. (The word analogy comes from the Greek word for
proportion, which makes sense if you think of an analogy as setting up some sort
of relationship between two or more things.)

You wont have to memorize lists of words and figure out if chthonic: stygian ::
celestial : effulgent, because lets face it, those words are fairly useless unless youre
Take your S AT prep to the next level. Visit www.learnerator.com
39

40

a huge word nerd/studying for a PhD in mythology /a passionate H.P. Lovecraft
fan/an awesome combination of the three. (The analogy works, by the way).

But analogies themselves are a crucial way to organize information, to marshal
support for an argument, to understand a relationship, or to impose an orderly
framework on a narrative. In short, the College Board finally wised up, threw out the
thesaurus, and decided to test you on applicable skills, like whether you can use
critical reasoning to figure out what information can and should be usefully
extrapolated and applied to a new situation. A lot of this sounds more complicated
than it is; chances are, if youve ever had, made, or defended an argument, youve
already practiced this skill.
Now you just need to hone this skill in the context of reading. It may help to
remember that making an analogy is just another form of making an inference
extrapolating from the text to figure out how the information applies to a new
situation.

Ready to practice? The following passage from Platos Apology details the
conviction and execution of Socrates.
The question may be asked, Why will he persist in following a profession which
leads him to death? Why?--because he must remain at his post where the god
has placed him, as he remained at Potidaea, and Amphipolis, and Delium,
where the generals placed him. Besides, he is not so over wise as to imagine
that he knows whether death is a good or an evil; and he is certain that
desertion of his duty is an evil. Anytus is quite right in saying that they should
never have indicted him if they meant to let him go. For he will certainly obey
God rather than man; and will continue to preach to all men of all ages the
necessity of virtue and improvement; and if they refuse to listen to him he will
still persevere and reprove them. This is his way of corrupting the youth, which
he will not cease to follow in obedience to the god, even if a thousand deaths
await him.

Take your S AT prep to the next level. Visit www.learnerator.com
40

41

Mark up your text however you like. Heres how I annotated and translated mine:

The question may be asked, Why will he (Socrates) persist in following a
profession which leads him to death? Why?--because he must remain at his post
where the god has placed him, as he remained at Potidaea, and Amphipolis, and
Delium, where the generals placed him. (Socrates was a military man)
Besides, he is not so overwise as to imagine that he knows whether death is a
good or an evil; and he is certain that desertion of his duty is an evil. (Would
rather face death, which may or may not be evil, than desert his duty,
which he knows is evil.) Anytus is quite right in saying that they should
never have indicted him if they meant to let him go. For he will certainly obey
God rather than man; and will continue to preach to all men of all ages the
necessity of virtue and improvement; and if they refuse to listen to him he will
still persevere and reprove (reprimand) them. This is his way of corrupting the
youth, which he will not cease to follow in obedience to the god, even if a
thousand deaths await him. (Stands up for what he believes in despite what
other people say)

Your annotations and translations may look different; thats okay. Different things
jump out to different readers. Practice your analogical reasoning skills with the
following question:

1. Based on the excerpt, which of the following behaviors would Socrates most
disdain?
A. A teacher corrupting his students
B. The worship of false gods
C. Registered voters neglecting to cast their ballots
D. Students returning library books late

The passage doesnt mention any of these scenarios, so based on what you know
about Socrates, draw an analogy between the scenarios described in the answer
Take your S AT prep to the next level. Visit www.learnerator.com
41

42

choices and the positions outlined in the excerpt, and see which relationship fits
best.

Lets look at A. The words corrupting the youth appear in the passage in the
context of something that Socrates himself did not cease, so theres no need to
even draw an analogy here we are flat-out told that he engaged in corruption,
whatever this might mean, so we this cant be a scenario that he disdained.

Check out B, The worship of false gods. There is mention of gods and obedience to
gods in the passage, but nowhere is it mentioned that Socrates believed only in one
specific type of god and/or cared about other peoples beliefs. In fact, theres no
mention at all of Socrates as a critic of other peoples faiths, ideas, or dogmas, so this
cant be it we cant even draw an analogy between this answer choice and any
behavior outlined in the passage.

Look at C, Registered voters neglecting to cast their ballots. Think about the
relationship between a voter and the vote; once you have registered, it is arguably
your civic duty to cast your ballot. If you dont, you are neglecting your duty.
Though there is no mention of voting here, a sizable portion of the passage is
devoted to Socrates hatred of dereliction (the shirking of duty.) In fact, he would
rather die than neglect his duty which he views as evil. If we take the premise
that not voting = shirking your duty, then this relationship could fit. But lets view
Option D before we come to any decisions
Option D is a contender; one might argue that returning a library book in a timely
fashion is also a duty. However, the books still get returned in this scenario, even if
they get returned late, so the duty isnt necessarily shirked, just delayed. Though
this is somewhat plausible, it doesnt hold up as nicely as Option C, which provides
the best answer.

Lets try with another passage

Take your S AT prep to the next level. Visit www.learnerator.com


42

43

Barbara Cawdry is passionate about environmentalism and social protest. The
fifty-eight-year-old retired science teacher made waves recently when she tied
herself to an ancient oak tree slated to be cut down by the California Parks
Department. Though interfering with the agenda and activities of the state
government is considered a crime, Cawdry remained tethered to the tree,
refusing food or drink, until the state police arrived to forcibly remove her. She
is continuing her vigil in the North County jail, where she has threatened to
stage a hunger strike if the tree is cut down. At the time of this writing, Cawdry
faces a two thousand dollar fine and up to five months in jail. When contacted
for this article, Cawdrys message was brief: All Im doing is trying to save the
trees.

This is a fairly simple passage, so you may not feel the need to mark it up.

Take your S AT prep to the next level. Visit www.learnerator.com


43

44

2. Based on the passage, Cawdry would probably support which of the
following groups?
A. Loggers striking for better pay and better hours
B. Armed freedom fighters questing to resist an outside force by whatever
means necessary
C. Members of the Animal Liberation Front destroying labs where animal
testing is conducted
D. Anti-war protestors disrupting traffic during a protest

Now, none of those people or scenarios are mentioned in the passage. But based on
what we know about Cawdry, you can easily set up an analogy between her actions
and that of the groups mentioned in the answer choices.

Cawdrys relationship to social protest does not seem to be violent or destructive, so
lets see if we can eliminate any of the other choices. We dont know anything about
her politics other than the fact that she really, really likes trees, so we cant
necessarily set up an analogy between her passions and the passions of the people
in this group. With that said, if you look at choice A, its pretty obvious that Cawdry
would not sympathize much with loggers, as they literally cut trees down for a
living. She may espouse a similar form of protest non-violent disobedience but
her politics are too radically at odds with that of the loggers to make this a
compelling analogy. Strike answer A.

The armed freedom fighters in Choice B who resort to whatever means necessary
are probably too violent for Cawdrys tastes; we dont know how she feels about
their grievances, but we can extrapolate from her choice of protest (tying herself
peacefully to a tree) that she probably would not endorse whatever means
necessary, or else she might have resorted to violence to further her aims.
Choice C might look appealing at first; based on Cawdrys love for trees, we might be
able to extrapolate that animals are near and dear to her heart as well. But unlike
the members of the Liberation Front, she doesnt seem intent on destroying
Take your S AT prep to the next level. Visit www.learnerator.com
44

45

property; her protest seems mainly relegated to doing harm to herself, not to others.

Choice D is the only one left, and it fits best because just as Cawdrys behaviors are
non-violent yet disruptive, anti-war protestors take a similar tactic.
This question asks you to consider the different relationships between the ideas and
behaviors mentioned in the article and the ideas and behaviors outlined in the
answer choices; often, when the SAT asks you to make an analogy, what theyre
actually asking for is some kind of comparison. Make a brief list of the different
ideas in the passage; you will find the correct answer when you arrive at the best
match between the ideas in the passage and the ideas in the answer choices.

Try it one more time, using one of the sassiest passages in the English language,
Politics and the English Language, by George Orwell:

MOST PEOPLE WHO BOTHER with the matter at all would admit that the
English language is in a bad way, but it is generally assumed that we cannot by
conscious action do anything about it. Our civilization is decadent, and our
language--so the argument runs--must inevitably share in the general collapse.
It follows that any struggle against the abuse of language is a sentimental
archaism, like preferring candles to electric light or hansom cabs to aeroplanes.
Underneath this lies the half-conscious belief that language is a natural growth
and not an instrument which we shape for our own purposes.

Now, it is clear that the decline of a language must ultimately have political
and economic causes: it is not due simply to the bad influence of this or that
individual writer. But an effect can become a cause, reinforcing the original
cause and producing the same effect in an intensified form, and so on
indefinitely. A man may take to drink because he feels himself to be a failure,
and then fail all the more completely because he drinks. It is rather the same
thing that is happening to the English language. It becomes ugly and
inaccurate because our thoughts are foolish, but the slovenliness of our
Take your S AT prep to the next level. Visit www.learnerator.com
45

46

language makes it easier for us to have foolish thoughts. The point is that the
process is reversible. Modern English, especially written English, is full of bad
habits which spread by imitation and which can be avoided if one is willing to
take the necessary trouble. If one gets rid of these habits one can think more
clearly, and to think clearly is a necessary first step towards political
regeneration: so that the fight against bad English is not frivolous and is not
the exclusive concern of professional writers. I will come back to this presently,
and I hope that by that time the meaning of what I have said here will have
become clearer.

Ive annotated the passage like so, paying special attention to the IDEAS expressed,
because remember that when it comes to analogies, we want to see if theres an
equivalency between one idea / scenario and another:

MOST PEOPLE WHO BOTHER with the matter at all would admit that the
English language is in a bad way, but it is generally assumed that we cannot by
conscious action do anything about it. (Our language skills are
deteriorating.) Our civilization is decadent, and our language--so the
argument runs--must inevitably share in the general collapse. (People think
that because our civilization is waning so must our language). It follows
that any struggle against the abuse of language is a sentimental archaism, like
preferring candles to electric light or hansom cabs to aeroplanes. (Insisting on
standards makes you seem really old-fashioned.) Underneath this lies the
half-conscious belief that language is a natural growth and not an instrument,
which we shape for our own purposes. (People believe that we dont have
conscious control over the evolution of our language but we do!)

Now, it is clear that the decline of a language must ultimately have political
and economic causes: it is not due simply to the bad influence of this or that
individual writer.(Its no one persons fault that our language is
devolving.) But an effect can become a cause, reinforcing the original cause
Take your S AT prep to the next level. Visit www.learnerator.com
46

47

and producing the same effect in an intensified form, and so on indefinitely. A
man may take to drink because he feels himself to be a failure, and then fail all
the more completely because he drinks. (An analogy: this is a vicious cycle!
Our language is devolving, so bad writers feel god about writing bad
stuff, which makes the language devolve further) It is rather the same
thing that is happening to the English language. It becomes ugly and
inaccurate because our thoughts are foolish, but the slovenliness of our
language makes it easier for us to have foolish thoughts. The point is that the
process is reversible. (We can make our language better) Modern English,
especially written English, is full of bad habits which spread by imitation and
which can be avoided if one is willing to take the necessary trouble. If one gets
rid of these habits one can think more clearly, and to think clearly is a
necessary first step towards political regeneration: so that the fight against bad
English is not frivolous and is not the exclusive concern of professional writers. I
will come back to this presently, and I hope that by that time the meaning of
what I have said here will have become clearer.

Please note that Orwell himself makes use of analogies not once but twice! First he
says that struggling to maintain language standards makes one seem old-fashioned,
as if campaigning for the return of candles as opposed to electric lights. Then he
compares the devolution of the English language as the kind of vicious cycle that
alcoholics suffer. Orwell knows: analogies are a rich and complex way to express
ideas.

Take your S AT prep to the next level. Visit www.learnerator.com


47

48

3. Based on the passage, which of the following scenarios would Orwell find
most objectionable?
A. Doctors who use outdated methods of medical observation
B. Civilizations that become so decadent that their morals decay entirely
C. Students whose spelling has been corrupted by social media
D. Professional writers who struggle to communicate their intended meanings

I find it helpful to distill the answer choices into a single core idea, which may make
it easier to figure out which answer is most congruous with the ideas expressed in
the passage.

Option A. At the heart of this option is the idea of being old-fashioned or outdated.
While Orwell does talk about the idea of being old-fashioned (It follows that any
struggle against the abuse of language is a sentimental archaism, like preferring
candles to electric light or hansom cabs to aeroplanes. (Insisting on standards
makes you seem really old-fashioned.), he isnt disapproving of this. In fact, he
seems to either endorse the notion of being old-fashioned, or to disregard it entirely.
Either way, he isnt positioning himself firmly as being against old-fashioned
tendencies. We can discard Option A.

Option B. Distill this into a single idea decay. Its true that Orwell seems pretty
anti-decay (especially as regards to language); though he says little in this passage
about morals, theres no evidence that he WOULDNT find this scenario
objectionable. Keep it for now.

Option C. What is at the heart of this answer? The devolution of a type of language.
Orwell spends two paragraphs railing against bad writers; there is every indication
that he would find social-media spellings to be a travesty. In fact, this is so directly
parallels Orwells statement that its barely even an analogy its more a modern-
day extension of his opinion. Clearly this is probably going to be the best option, but
lets look at Option D as well.
Take your S AT prep to the next level. Visit www.learnerator.com
48

49


Option D. Distill this answer choice into an idea: the struggle of using language
properly. Orwells piece castigates bad writers and bad writing but nowhere does
he say that writing well is EASY. In fact, he would probably approve of a writers
struggle to communicate meaning clearly and cleanly. Certainly he makes no
statement that is similar / in harmony with the idea that writing should be easy.
Of all these choices, C is clearly the best it is the MOST similar to what Orwell is
saying.
Isolating the ideas in a passage so that you can answer questions about them is a
very good strategy for mastering reading comprehension questions; its going to be
especially useful for the next chapter of our review, Citing Textual Evidence.

Take your S AT prep to the next level. Visit www.learnerator.com


49

50

Citing Textual Evidence


To achieve success on the SAT and in the classroom, you must be in possession of all
the facts. Aside from a few math questions, the majority of the questions are
multiple choice meaning that the answers are already there. All that remains is for
you to make sure you are picking the best one, and the way to do this is to make
sure that youre paying attention to all the evidence. Every answer is the best
answer for a reason; you have to make sure you understand what that reason
actually is.
To do this, you must understand the passage thoroughly, so you know what actually
counts as evidence or as support for a claim.
Next, you must understand what the question is actually asking you so that you are
equipped to choose the absolute best answer for it.
Take this brief passage, for example:

Upon my entrance, Usher arose from a sofa on which he had been lying at full
length, and greeted me with a vivacious warmth which had much in it, I at first
thought, of an overdone cordiality -- of the constrained effort of the ennuy
man of the world. A glance, however, at his countenance, convinced me of his
perfect sincerity. We sat down; and for some moments, while he spoke not, I
gazed upon him with a feeling half of pity, half of awe. Surely, man had never
before so terribly altered, in so brief a period, as had Roderick Usher.

This passage is a bit old-fashioned, so lets take our time annotating it:

Upon my entrance, Usher arose from a sofa on which he had been lying at full
length, (the man was lying on a couch and got up when I entered) and
greeted me with a vivacious warmth which had much in it, I at first thought, of
an overdone cordiality -- of the constrained effort of the ennuy man of the
world. (his greeting seemed overdone; he put in too much effort). A glance,
Take your S AT prep to the next level. Visit www.learnerator.com
50

51

however, at his countenance, convinced me of his perfect sincerity. (But when I
looked at his face I could tell her was sincere.) We sat down; and for some
moments, while he spoke not, I gazed upon him with a feeling half of pity, half
of awe. (He didnt speak, and I stared at him, half with pity and half with
awe.) Surely, man had never before so terribly altered, in so brief a period, as
had Roderick Usher. (He had changed very quickly)

Now make a list of all the evidence you can cite from the text to support the
following statement:

There is something wrong with Roderick Usher.

Look at the text. What indicates that there might be something wrong?
1. Well, for one, Usher is lying on the sofa when our narrator walks in. Now, we dont
know what time of day it is, so by itself this cant be taken as indication that
something is wrong, but it could be a telling detail.
2. He has an overdone cordiality constrained effort of the ennui man of the world
i.e. he seemed at first to have the social mannerisms of a person who was bored
(ennui) with the world. This is a signal that Usher could be depressed or at the
very least full of existential boredom not a great state of affairs.
3. I gazed upon him with a feeling half of pity and half of awe. This is telling once
youre awed by the amount of pity you have for someone, you know something is
wrong with them.
4. Terribly altered to be changed is not such a bad thing. But to be terribly
changed; changed for the worst? Warning sign.

You never know what might be evidence, which is why its so important to read
closely and to mark up your text as thoroughly as possible, so when youre passing
back over it you can see at a glance which ideas end up being important.

Take your S AT prep to the next level. Visit www.learnerator.com


51

52

Lets try this again. Read the excerpt, which is from The Atlantic, and then see what
evidence you can find to support the statement.

Since the release of The Hunger Games in 2012, dystopian cinema has enjoyed
sustained interest in American culture. Popular young adult novels are being
turned into blockbuster Hollywood films every few months, it seems, and with
good reason: Beyond their built-in teen fan base, films like Divergent, The Giver,
and The Maze Runner draw on some of adult societys greatest fears of the
moment: Is technology tearing us further apart? Will global warming destroy
the planet? Will income inequality further create a world of haves and have-
nots?

Critics have worried that these particular films stoke an irrational fear of
technology, or a distaste for big government, but dystopian stories have long
been celebrated (and used in classrooms across the country) because of their
ability to push audiences to think critically about their actions.

Yet with the upcoming release of The Hunger Games: Mockingjay Part 1, poised
to be the biggest film of the year, its just as worthwhile to consider what these
films dont seem to fear. While recent dystopias warn youth about over-reliance
on computers, totalitarian rule, class warfare, pandemic panics and global
warming, very few ask audiences to think deeply about sexism and racism.

Once you have marked up your passage, find all the evidence you can to support the
following statement:

The author believes that most dystopian films and books are not doing enough
to promote critical thinking.

Read carefully, pulling out the strands of the authors argument in pieces:

Take your S AT prep to the next level. Visit www.learnerator.com


52

53

1. Dystopian cinema has enjoyed sustained interest
OK, this argues for dystopias popularity not for their promotion of critical thinking

2. Popular YA novels are being turned into blockbuster Hollywood films every few
months and with good reason.
Also argues for dystopias popularity not for their promotion of critical thinking

3. Draw on some of adult societys greatest fears
Fear does not necessarily lead to critical thinking, so we cant put this one in the
evidence column

4. Dystopian stories have long been celebrated (and used in classrooms across the
country) because of their ability to push audiences to think critically about their
actions
Finally, a mention of critical thinking so these films DO promote it. This does not
directly contradict the statement, however, that they do not do ENOUGH to promote
critical thinking. Remember, to answer these types of questions you must read
CAREFULLY!

5. Its just as worthwhile to consider what these films dont seem to fear.
Ah ha. The author is now asking you to consider what these films DONT do i.e. to
evaluate what is negative and/or missing about these films. S/he doesnt say anything
about critical thinking, but this might be the turning point for the article to start
discussing a specific failure of these films which may include the fact that they dont
do enough to promote critical thinking.

6. While recent dystopias warn youth about over-reliance on computers,
totalitarian rule, class warfare, pandemic panics and global warming, very few ask
audiences to think deeply about sexism and racism. We have a winner. Your first
clue is the world while, which indicates a shift from a previously held/described
opinion. The author spent the previous paragraph setting up a scenario (schools use
Take your S AT prep to the next level. Visit www.learnerator.com
53

54

dystopias to teach students certain lessons and make them think about things), and
now there is going to be a shift: Very few ask audiences to think deeply.

For an article like this, it is important to consider the structure of a piece the
author spends some time setting up a scenario, in which s/he lays out the
foundations of an argument. A good argument will always contain claims and
counterclaims, so dont be fooled by the parts of the argument that give an opposite
opinion; they should not be taken as evidence against the authors argument or
opinion, but as acknowledgments that alternate viewpoints exist (usually prior to
the author demonstrating why they are invalid). Its not always easy to tell which
part of a text is the authors argument and which contains the claims that s/he will
shortly dismantle; thats why transition words like yet, but, while, however,
and while are so important.

Lets try one more time; this time, you will have to answer a question with one of
four options. When answering, consider the evidence:


This passage is also from The Atlantic:

This week, one of the two Common Core assessment consortia announced its
cut scoresbenchmarks students need to clear to be considered proficient at
the content on which theyre tested. Just a third to roughly 40 percent of K-8
and high school students are projected to be proficient in math and English
Language Arts

And while tests before didnt challenge students enough, Minnich said, the
conversation in states where these Common Core-aligned tests are imminent
will be, Well, who decided this was the right thing for kids? Who decided that
these test scores were actually what kids need?

Take your S AT prep to the next level. Visit www.learnerator.com
54

55

For states to expand deeper-learning techniques, theyll likely first need to
overcome the blowback from parents whose children are suddenly deemed
unprepared for the challenges of college. Its a recipe as simple as it is
complex: Convince the masses the Common Core is on to something good, and
improve from there.

1. The author of this article probably feels most strongly that
A. Tests do not challenge students enough
B. Certain states are doing a better job than others of preparing students for
tests
C. The Common Core needs to undergo change to be truly meaningful
D. The Common Core is not the right thing for kids

Now, youll want to pay attention to this one because the answer choices all echo
sentences found in the text. The trick is to determine what the question is actually
asking you, so you can understand the correct evidence. Its not enough to simply
quote from the text your quote must support and be relevant to an assertion.

Lets look at the answer choices:

A. Tests do not challenge students enough.
Consider the evidence from the text just a third to 40% of students are projected
to be proficient in math and English. If only 40% of students are passing let alone
doing well there is very little compelling evidence that the tests are not adequately
challenging. And look at the next line And while tests before didnt challenge
students enough So while this may at one point have been an opinion, the
question is asking about tests NOW. Lastly, the question is asking about
the authors opinion. At this point in the article, the author has not yet stated an
opinion, but is rather quoting statistics and other peoples opinions. Pay attention to
the structure here its important to be able to differentiate the authors opinion,
and that of her sources.
Take your S AT prep to the next level. Visit www.learnerator.com
55

56


B. Certain states are doing a better job than others of preparing students for
tests.
Look closely. Are any specific states mentioned? Are there any statistics for the
breakdown of achievement by state? Is there a single indication that some states are
doing better than others? Theres absolutely nothing in this article to support this
opinion; scrap this answer.

C. The Common Core needs to undergo change to be really meaningful.
First, find all mentions of the Common Core:
This week, one of the two Common Core assessment consortia announced its cut
scores
OK, that has nothing to do with changing to be meaningful.

The conversation in states where these Common Core-aligned tests are imminent will
be, Well, who decided this was the right thing for kids? Who decided that these test
scores were actually what kids need?
A quote from someone not the author predicting that certain states are poised to
criticize the Common Core. This could potentially be a meaningful piece of evidence
as it predicts (the need for) a possible change in the Common Core standards.

Its a recipe as simple as it is complex: Convince the masses the Common Core is on to
something good, and improve from there.
OK, more possible evidence here: on to something good and improve from there.
Clearly, the author believes that the Common Core has SOME merit, and can be
improved on in order to rectify or address the issues mentioned in the article.

Lets keep this answer choice for now.

D. The Common Core is not the right thing for kids.
Looking at all the evidence we compiled for the answer above all the mentions of
Take your S AT prep to the next level. Visit www.learnerator.com
56

57

the Common Core its clear that the only people who are asking is this right for
kids? are states and parents, NOT the author. Additionally, questioning whether
something is right is NOT the same as declaring definitively that it isnt. Lastly, the
authors last words in this excerpt are The answer is simple: convince the masses
the Common Core is on to something good and improve from there. S/hes not
dismissing the Common Core out of hand, but saying instead that it has the potential
to be meaningful. It cant be entirely the wrong thing for kids if its on to something
good.

Clearly, the evidence best supports choice C.

Lets do this one more time, again with an excerpt from the Atlantic.
Animal behavior seems to reveal that animals don't want to be in enclosures,
either:
A giraffe who freaks out about men with large cameras, a brown bear whose
cage door is the subject of his obsessive compulsive disorder, a 5,000-pound
killer whale who shows her trainer who is boss by dragging him underwater for
just about as long as he can live, before letting him gothese episodes seem
like something more complicated than simple errors of confinement. It is hard
to avoid the conclusion that in some way the animals understand that the
world around them is an artificial one, that these phobias and psychotic
episodes represent reactions to that artifice, or subversions of it.

2. Based on the passage, what is the reason that animals in zoos can
potentially exhibit violent behavior?
A. These animals have psychiatric disorders
B. These animals are traumatized by the demands of their trainers and
keepers
C. These animals suffer from severe phobias
D. These animals seek to disrupt what they perceive as the unreal world
around them
Take your S AT prep to the next level. Visit www.learnerator.com
57

58


Look at the evidence for Option A. While the article mentions an animal freaking
out and another with obsessive compulsive disorder, it does not suggest that all
of these animals have psychiatric disorders. In fact, the article tries to investigate
the roots of these behaviors; to simply dismiss said violent behaviors as disorders
would not get at the heart of the articles argument.

Consider B. Again, while the article mentions freaking out about men with large
cameras, the obsessive compulsive fixation on a cage, and showing the trainer
who is boss, again, the article is trying to figure out why these things happen. To
use these incidents as evidence for themselves simply creates a circular argument.
Check out C. Again, the article does mention phobias, but there is no explicit link
drawn between the phobias and the violent behavior; additionally, the next
sentence seems to suggest that (rather than one creating the other), they stem from
the same source. To use the phobias as evidence for why animals behave violently
seems, again, to be circular, since the ultimate argument is that these behaviors and
attitudes stem from the same place.

Last, look at D. The last sentence of the excerpt says in some way the animals
understand that the world around them is an artificial one, that these phobias and
psychotic episodes represent reactions to that artifice, or subversions of it. In other
words, they understand they should not be in this condition, in this enclosure,
penned in, and all their behaviors (their disorders, phobias, feelings about trainers
and keepers, violent behaviors, etc) all come from the same place: the desire to
somehow disrupt this artificial, or fake, world that they are living in.

In essence, the distractors for this question are all incomplete they are all
incomplete thoughts that, taken together, create the best and fullest answer to the
question. In the next section, you will practice isolating the details that together
make up the central idea or theme of an excerpt.

Take your S AT prep to the next level. Visit www.learnerator.com


58

59

Determining Main Ideas


Perhaps the most important part of reading, and reading comprehension, is
knowing how to determine the main idea of a passage or selection. After all, this is
the real test of whether you understand what the passage is trying to convey. You
may not understand every single word, but the ability to ferret out the general
meaning of a larger selection whether its a paragraph, a chapter, or even a whole
book is crucial to the process of analyzing, deconstructing, and even enjoying what
you read.

But How?

Determining a main idea can be as simple as looking at the first and last sentences of
every paragraph. In a simple non-fiction work, these sentences will often introduce
or summarize the essential ideas in the selection. While this is NOT a strategy that
will work with literary or higher-level texts, its a good stepping-stone to
understanding where you can begin to look to gather the main idea.

Lets practice with a brief, simple selection from The New York Times:
Thursday morning comes quickly out of the dark, and with it the promise of
the Thanksgiving feast. Have you set the table yet? You should. Set it as if for a
sacrament. The Thanksgiving meal is Americas most holy secular ritual. There
are three things that people pick up on the instant they walk into your home on
Thanksgiving, said Danny Meyer, the New York restaurateur, who does not
celebrate the holiday at Union Square Cafe, Gramercy Tavern, North End Grill
or any of his other considerable number of restaurants, but always with family
at home. They will be able to feel the human energy. Theyll smell the food. And
they will see, instantly, the table. The combination if youre in a good mood,
and youve taken time with the table, and the turkey is cooking sends a
message that, no matter what, everything is going to be O.K.
Take your S AT prep to the next level. Visit www.learnerator.com
59

60


What is the main idea of this paragraph?

Look at the first sentence, which indicates that clearly, this passage will be about the
Thanksgiving feast. Then look at the last The combination (of good mood, good
food, and a set table) sends the message that everything is going to be O.K.

Now, Thanksgiving feast is not really an idea; its the subject. Whats the
difference? After all, a subject is what a passage is about and a main idea is also
what the passage is about. Think of it like this:

A subject is the broadest, most general way to describe something.
A main idea is much more specific.

A good rule of thumb when trying to determine main ideas is to put them in a full
phrase or sentence; after all, just saying that the main idea is the Thanksgiving
feast, gives no indication of what features or aspects of the Thanksgiving feast are
important. For all we know, a passage that is merely described as being about the
Thanksgiving feast could end up detailing all the ways in which the Thanksgiving
feast could go wrong! But the words no matter what, everything is going to be
okay are a pretty clear indicator that this isnt the case here. So to avoid ambiguity,
get yourself in the habit of scrawling the main idea (i.e. a full phrase or sentence) in
the margin of each paragraph.
Thus, taken together, the first and last sentences of each paragraph in a lower-
complexity informational text should present you with the main idea. Here it might
be something like:

With enough preparation, the Thanksgiving feast will go smoothly, or something
similar to that. (There are, of course, multiple ways to word this that will still be
correct!)
Lets try this once more with a similarly simple text, also from The New York Times:
Take your S AT prep to the next level. Visit www.learnerator.com
60

61


This season millions of Americans will celebrate with turkey on the table. The
turkey is, after all, the native North American animal that Benjamin Franklin
considered a much more respectable bird than the scavenging bald eagle. But
while the eagle landed on the countrys Great Seal and the turkey gets pride of
place at our holiday dinners, neither bird can claim to have changed American
culture more than their lowly avian (avian = bird) cousin, the chicken.

First sentence: Americans celebrate with turkey.
Last sentence: The chicken has changed American culture more than any other bird.
You dont even need the first sentence here to determine the main idea of the
passage. Clearly, its going to be about how chickens have changed America. (Pretty
thrilling stuff.)
In isolation, its pretty easy to determine the main idea of a single paragraph. But the
SAT will also ask you to determine the main ideas of PARTS of longer excerpts to
keep the longer excerpt in mind while choosing an answer that makes sense of the
different parts of longer pieces.

Lets practice with this 1965 speech on voting rights by Lyndon B. Johnson:

In our time we have come to live with moments of great crisis. Our lives have
been marked with debate about great issues; issues of war and peace, issues of
prosperity and depression. But rarely in any time does an issue lay bare the
secret heart of America itself. Rarely are we met with a challenge, not to our
growth or abundance, our welfare or our security, but rather to the values and
the purposes and the meaning of our beloved Nation.

The issue of equal rights for [African-Americans] is such an issue. And should
we defeat every enemy, should we double our wealth and conquer the stars, and
still be unequal to this issue, then we will have failed as a people and as a
nation. For with a country as with a person, "What is a man profited, if he shall
Take your S AT prep to the next level. Visit www.learnerator.com
61

62

gain the whole world, and lose his own soul ?" There is no [African-American]
problem. There is no Southern problem. There is no Northern problem. There is
only an American problem. And we are met here tonight as Americansnot as
Democrats or Republicans--we are met here as Americans to solve that
problem.

This was the first nation in the history of the world to be founded with a
purpose. The great phrases of that purpose still sound in every American heart,
North and South: "All men are created equal""government by consent of the
governed""give me liberty or give me death." Well, those are not just clever
words, or those are not just empty theories. In their name Americans have
fought and died for two centuries, and tonight around the world they stand
there as guardians of our liberty, risking their lives.

Those words are a promise to every citizen that he shall share in the dignity of
man. This dignity cannot be found in a man's possessions; it cannot be found in
his power, or in his position. It really rests on his right to be treated as a man
equal in opportunity to all others. It says that he shall share in freedom, he shall
choose his leaders, educate his children, and provide for his family according to
his ability and his merits as a human being.

A typical SAT question might ask you:
What is the main idea of the second paragraph?
A. Having equal rights will cause everyone to profit
B. Doubling national wealth and conquering the stars should be among
Americans top priorities
C. We will never defeat our enemies if we do not have equal rights
D. For the good of the nation, Americans must unite to fix unjust policies

Take your S AT prep to the next level. Visit www.learnerator.com


62

63

Again, this is a good place to look at the first and the last sentences of the paragraph
in question: The issue of equal rights is such an issue and We are met as
Americans to solve that problem.

Now, thats not particularly helpful because without the context of the previous
paragraph, we dont know what such an issue or that problem actually mean. But
we CAN understand that this speech deals with an important issue that Americans
must solve together. This may even be enough to help you answer the question
completely! But if youre not entirely sure yet of the answer, skim the first
paragraph again, and find out that such an issue (i.e. equality for African-
Americans) refers to an issue that lays bare the secret heart of America itself i.e.
a very important matter. So lets consider the answer options now:

A: Having equal rights will cause everyone to profit.
While in some (figurative) sense this may in fact be true, pay close attention is this
what Johnson is actually saying in the passage? Look at where he mentions profits:

a. should we double our wealth and conquer the stars, and still be unequal
to this issue, then we will have failed as a people and as a nation.
b. For with a country as with a person, "What is a man profited, if he shall
gain the whole world, and lose his own soul?"

These two sentences suggest that the current situation is untenable and immoral,
and while there is evidence that a reverse situation (equal rights may lead to profit)
its not compelling enough to be the main idea of the paragraph.

Look at B: While doubling American wealth and conquering the stars are good goals
to have, nowhere does Johnson say they are the most important; in fact, he seems to
suggest the opposite when he says that achieving those goals without achieving
equality will make us fail as a people and a nation. Clearly, this isnt the most

Take your S AT prep to the next level. Visit www.learnerator.com


63

64

important priority if there are other, better goals to worry about.

Check out C: Again, Johnson does mention defeating every enemy, but in context of
an achievement that will be meaningless if we do not first achieve equality for all of
our citizens. (We will fail as a people and as a nation.) This cant be the right
answer.

D is the only correct choice. Consider every part of the sentence:
For the good of the nation considering that other (extremely worthwhile)
goals are meaningless without equality, it can be inferred that achieving
equality will indeed be for the good of the nation.

The second part, Americans must unite finds support in the last few
sentences:
There is no Southern problem. There is no Northern problem. There is only an
American problem. And we are met here tonight as Americansnot as
Democrats or Republicans--we are met here as Americans to solve that
problem. Johnson lists some things that might divide us geographic region,
political affiliation before dismissing them as meaningless, demonstrating
that unity is key in solving essential problems.

Thus far this is all pretty straightforward. But what happens with a higher-level text,
or a literary text that doesnt do you the very nice favor of laying out main ideas in
the first and last sentences of each paragraph? When that happens, you must notice
the telling details and any repetitions of words and ideas. Try it with this (fairly
complex) excerpt from a David Foster Wallace essay about 9/11, written on
9/13/2001. Ive included my annotations below:

Everybody has flags out. Homes, businesses. It's odd:

Flags =

You never see anybody putting out a flag, but by Wednesday patriotism?

Take your S AT prep to the next level. Visit www.learnerator.com


64

65

morning there they all are. Big flags, small flags, regular
flag-size flags. A lot of home-owners here have those special
angled flag-holders by their front door, the kind whose brace
takes four Phillips screws. And thousands of those little
hand-held flags-on-a-stick you normally see at parades
some yards have dozens all over as if they'd somehow
sprouted overnight. Rural-road people attach the little flags
to their mailboxes out by the street. Some cars have them
wedged in their grille or duct-taped to the antenna. Some
upscale people have actual poles; their flags are at halfmast. More than a few large homes around Franklin Park or
out on the east side even have enormous multistory flags
hanging gonfalon-style down over their facades. It's
a total mystery where people get flags this big or how they
got them up there. My own next-door neighbor, a

Huge flags,
hanging like
banners
(gonfalon
heraldic flag, like
knights used to
have); making
quite a statement

retired CPA and vet whose home- and lawn-care are


nothing short of phenomenal, has a regulation-size
anodized flagpole secured in 18" of reinforced

-This guy sounds

cement that none of the other neighbors like very much

like he does

because they think it draws lightning. He says there's a very everything by the
particular etiquette to having your flag at halfbook:
mast: You're supposed to first run it all the way up

"phenomenal

to the top and then bring it halfway down.

homecare,"

Otherwise it's an insult or something. His flag is out

regulation-size

straight and popping smartly in the wind. It's far and away

official flagpole

the biggest flag on our street. You can also hear the wind in

-A good detail

the cornfields just south; it sounds the way light surf sounds

this guy cares

when you're two dunes back from it. Mr. N's flag's halyard

about

has metal elements that clank loudly against the pole when

procedures and

it's windy, which is something else the other neighbors don't

etiquette and

care for. His driveway and mine are almost side by side, and what is proper
he's out here on a stepladder polishing his pole with some
-"Or something"
kind of ointment and a chamois cloth and in fairness it's

Take your S AT prep to the next level. Visit www.learnerator.com


65

= our narrator is

66

true that his metal pole does shine like God's own

a bit flippant!

wrath. "He[ck] of a nice flag and display apparatus, Mr.


N.""Ought to be. Cost enough." "Seen all the other flags out
everywhere this morning? "This gets him to look down and
smile, if a bit grimly. "Something isn't it?" Mr. N is not

Biggest flag

what you'd call the friendliest next-door neighbor. I

because he thinks

really only know him because his church and mine are in the that makes him
seem like the
same softball league, for which he serves with immense
precision as his team's statistician. We are not

biggest

close. He's nevertheless the first one I ask: "Say Mr. N,

patriot? Clearly

suppose somebody like a foreign person or TV reporter were puts a lot of effort
into appearances.
to come by and ask you to say what the purpose of all these
flags everywhere after the Horror and everything yesterday Whats the
was, exactly what do you think you'd say?" "Why" (after connection
a brief interval of giving me the same sort of look he between Gods
wrath and
usually gives my lawn) "to show our support and
empathy in terms of what's going on, as Americans. "The

displaying an

point being that on Wednesday here there's a weird

American flag

accretive pressure to have a flag out. If the purpose

right after 9/11?

of a flag is to make a statement, it seems like at a

Think about this

certain point of density of flags you're making more idea


of a statement if you don't have one out. It's not

More details

totally clear what statement this would be. What if

about the

you just don't happen to have a flag? Where has

neighbor. Think

everyone gotten these flags, especially the little ones you can about the
put on your mailbox? Are they all from July 4th and people contrast between
just save them, like Christmas ornaments? How do they

the two men

know to do this? Even a sort of half-collapsed house down


the street that everybody though was unoccupied has a flag
in the ground by the driveway. The Yellow Pages have
nothing under Flag. There's actual interior tension:

Again, a very

Nobody walks by or stops their car and says, "Hey,

telling detail

your house doesn't have a flag," but it gets easier

contrast between

Take your S AT prep to the next level. Visit www.learnerator.com


66

67

and easier to imagine people thinking it. None of the

the two

grocery stores in town turn out to stock any flags. The


novelty shop downtown has nothing but Halloween stuff.
Only a few businesses are open, but even the closed ones are
displaying some sort of flag. It's almost surreal. The VFW

Accretive =

hall is a good bet, but it can't open til noon if at all (it has a

accumulated,

bar). The lady at Burwell's references a certain hideous Qik-

built up gradually

n-EZ store out by 1-74 at which she was under the


impression she'd seen some little plastic flags back in the
racks with all the bandannas and Nascar caps, but by the
time I get there they turn out to be gone, snapped up by
parties unknown. The reality is that there is not a flag to be
had in this town. Stealing one out of somebody's yard is
clearly out of the question. I'm standing in a Qik-n-EZ afraid
to go home. All those people dead, and I'm sent to the
edge by a plastic flag. Until in one more of the Horror's
weird twists of fate and circumstance it's the Qik-n-EZ
proprietor himself (a Pakistani, by the way) who offers

Pressure! He feels
pressure to put
out a flag, like
hes not being
patriotic
enough
Everyone has a
flag but him!

solace and a shoulder and a strange kind of unspoken

Feels very

understanding, and who lets me go back and sit in the

conspicuous

stock room amid every conceivable petty vice and


indulgence America has to offer and compose myself, and
who only slightly later, over styrofoam cups of a strange
kind of tea with a great deal of milk in it, suggests, gently,
construction paper and "Magical Markers," which
explains my now-beloved homemade flag.

How is the writer


internalizing,
understanding,
and processing
the tragedy?

Why would this


man offer
unspoken
understanding?
In what way are

Take your S AT prep to the next level. Visit www.learnerator.com


67

68

they both
outsiders?

How does the


writer resolve his
problem? Do you
think that his
neighbor would
approve of this
solution?


Look back at your notes and by the way, it doesnt matter if there are words whose
meaning you dont know; what matters is that you get the general gist.
So, what are the repeated ideas in this excerpt?
Well, theres flags and, more subtly, what these flags might mean. But certainly its
not enough to say the main idea of this essay is flags. Because again, that doesnt
tell us anything important.
Therefore, think about the people mentioned in this essay. How do they reactto the
issue of flags, and why do they react this way? This might make it easier to distill
this big essay into one idea.

1. Theres our unnamed narrator, who does not have a flag and feels bad / like an
outsider about it. (Remember, he says he feels pressure, and like he is making a
statement, and hes being sent to the edge over this issue.)
2. Theres Mr. N from next door, who has a flag and clearly takes lots of pride in it.
(Its regulation-size, raised and lowered with military precision, and as polished
as the wrath of God.)

Take your S AT prep to the next level. Visit www.learnerator.com


68

69

3. Theres the Pakistani owner of the shop, who listens to our narrator and
provides him with a solution to his flag problem. (Offers solace, gentle
suggestion, unspoken understanding.)
Clearly, the essay is about more than who has a flag. Its clearly about what the flag
represents to each person in this excerpt. Remember, finally, that this essay was
written two days after 9/11. The context here is crucial. Therefore:

What is the main idea of this passage as a whole?
A. The script by which different people process and symbolize complex
emotions, such as patriotism and grief
B. The proper procedures for showing off patriotism and reacting to a national
tragedy
C. The discrimination against Pakistanis after 9/11
D. The pressure that comes with not being adequately prepared to react to a
tragedy

Now, think about this carefully. Most of these answers are pretty plausible, because
they all have something to do with the passage. Lets start with A the script by
which people process and symbolize emotions like patriotism and grief.
Now, Wallace never mentions a script, but he doesnt have to; the majority of the
passage is about the fact that everyone seems to have a flag but him, as though they
are all following a memo that he didnt receive. (Where has everyone gotten these
flags, especially the little ones you can put on your mailbox? Are they all from July 4th
and people just save them, like Christmas ornaments? How do they know to do this?
Even a sort of half-collapsed house down the street that everybody though was
unoccupied has a flag in the ground by the driveway.) So the script part, while subtle,
is true. But what about this symbolize complex emotions? At this point, you should
realize that the flags here are not just objects but representations of a feeling. After
all, Wallace asks his neighbor:

"Say Mr. N, suppose somebody like a foreign person or TV reporter were to come by
Take your S AT prep to the next level. Visit www.learnerator.com
69

70

and ask you to say what the purpose of all these flags everywhere after the Horror and
everything yesterday was, exactly what do you think you'd say?"

Clearly, this question demonstrates that there is a larger purpose to these flags, that
they symbolize and demonstrate something big. And thats clear in Mr. Ns answer,
too: to show our support and empathy in terms of what's going on, as
Americans. Again, the flags are part of a script. They are the answer to a tragedy and
a means of showing support, which is demonstrated also in the different types of
flags on display the huge banner-style ones, the regulation-size ones on flagpoles,
the plastic ones, and even the authors homemade Magic Marker one.

Now, lets look at the other answer choices:
B. The proper procedures for showing off patriotism and reacting to a national
tragedy, is a worthwhile contender. However, remember that at first, Wallace does
not have a flag. Based on his characterization of himself and his flag-less-ness (If the
purpose of a flag is to make a statement, it seems like at a certain point of density of
flags you're making more of a statement if you don't have one out. It's not totally clear
what statement this would be. What if you just don't happen to have a flag?), is
Wallace the sort of person who would get hung up on proper procedures?
Especially when you contrast him with Mr. N, who gives him the same sort of look
he usually gives my lawn. Though there are points in this essay that are concerned
with procedure, these points are more about how other people seem to be acting
(again, following a script), rather than an actual guide on the best way to act.

C is perhaps the easiest answer to disregard; the Pakistani storeowner is only
mentioned at the very end of the essay, and though Wallace does mention the store
owners unspoken understanding (which alludes, in turn, to the outsider status he
probably feels in the wake of the tragedy), nowhere does he explicitly mention
discrimination. Additionally, this unspoken understanding is only mentioned once,
and a main idea is usually repeated and reinforced in multiple ways. This is clearly
not the correct answer.
Take your S AT prep to the next level. Visit www.learnerator.com
70

71


D is a contender clearly, Wallace feels pressure! This is an instance where there
are some good answers that pale in comparison to the best answer. While its true
that there is great pressure in feeling that you are not reacting appropriately to /
prepared adequately for a national tragedy, these reactions and preparations are
in turn just part of a larger script that tells people how they should act, how they
should deal with their lack of preparedness. D is a good answer because its really
part of answer A in many ways, a fragmented or incomplete choice that does not
fully express the main idea of the essay.

That was a long, high-level passage not so much because of the words or the
structure, but because it is full of subtly-expressed and complex ideas. In the next
section, you will learn how to break down and summarize these kinds of ideas.

Take your S AT prep to the next level. Visit www.learnerator.com


71

72

Summarizing
Ever tried to tell a friend a story? When youre recounting an event or a film, you
automatically make choices about which details to emphasize and which to leave
out. And though your friend probably wont mind if you leave in irrelevant details,
one of the most highly-rated skills in secondary and post-secondary education is the
ability to summarize key information, which explains why the College Board cares so
much about whether or not you can do it well.

There isnt a big difference between identifying the main idea and being able to
cogently summarize the text; after all, the main idea and the summary are
interrelated concepts. In a sense, summarizing just means that you are able to
restate the main idea of all of or part of a text and marshal key pieces of evidence to
support your claim.

Learning how to accurately and intelligently summarize information requires three
essential skills:
1. Comprehension of the material
2. The ability to identify and isolate key supporting details
3. The ability to describe, in general terms, what those details work together
to say
Your ability to do this may depend on the level, complexity, and type of text youre
reading. Luckily, every type of text will have certain signals that let you know
which pieces of information are important.

Take your S AT prep to the next level. Visit www.learnerator.com


72

73

Structure

Remember that text structure is paramount in simpler non-fiction texts, with each
paragraph introduced (or summarized) by a topic sentence that presents the main
idea of the paragraph. Its these bracketing sentences the first and last of each
paragraph that will work together to present a useful summary of the selection as
a whole.
For example, take this excerpt, from a lower-complexity non-fiction article titled
Traveling Teaches Students in a Way Schools Cant, recently published in the
Atlantic. I have bolded parts of the first and last sentences in each paragraph, which
should all help you to summarize the text of the passage as a whole.

When I turned 15, my parents sent me alone on a one-month trip to
Ecuador, the country where my father was born. This was tradition in our
familyfor my parents to send their first-generation American kids to the
country of their heritage, where we would meet our extended family,
immerse ourselves in a different culture, and learn some lessons on
gratefulness.

My familys plan worked. That month in Ecuador did more for my character,
education, and sense of identity than any other experience in my early life. And
five years later, my experience in Ecuador inspired me to spend more time
abroad, studying in South Africa at the University of Cape Town. These two
trips not only made me a lifelong traveler, but also a person who believes
traveling to developing countries should be a necessary rite of passage
for every young American who has the means.

Its often said that spending time in less affluent countries teaches
Americans never to take anything for granted. To some extent, this is true.
During my time traveling in these areas, I often traveled without access to hot
water, Internet, air conditioning, or even basic electricity. I slept in rooms with
Take your S AT prep to the next level. Visit www.learnerator.com
73

74

spiders, mosquitos, and bedbugs. I rode on public transportation that rarely left
on time and often broke down suddenly in remote areas. Stripped of my daily
habits and expectations, I was forced to surrender the idea that I have a right
to anythingincluding the luxury of convenience, or days when everything Ive
planned actually happens. And my minor travel hassles seemed even more
petty when I realized that they represented larger systemic problems
that locals must deal with every day.

But these trips didnt only teach me to appreciate what I had; they also
moved me to consider why I had it in the first place. I realized that much of
what I thought was necessity was, in fact, luxury and began to realize how
easily I could survive off of much less. I didnt necessarily need hot water or a
timely bus or a comfortable bed to be happy for the day. I didnt necessarily
need a jaw-dropping landscape or a famous archeological ruin or a stunning
beach to make my travels worth it. Instead, most of the time, that fulfillment
came from the people I interacted withnot the things I had or did. It came
from eating soup with locals at a rest stop on a 12-hour bus ride, sharing a
meal with Peruvian soccer fans while watching a match, or chatting with the
owner of my hostel during his lunch break. Discovering that my best travel
moments came from these subtle, personal moments instead of the grandiose,
materialistic ones made me understand that living contently required
little. What I originally thought I took for granted, I now rethought
taking at all.

1. What is the most accurate summary of this article?
A. The author is a lifelong traveler.
B. The author believes that minor hassles are meaningless.
C. The author advocates travel abroad as an educational tool.
D. The author never takes anything for granted.

Take your S AT prep to the next level. Visit www.learnerator.com


74

75

Now, all of these options were mentioned somewhere in the text; maybe even in the
bracketing sentences. But all the distractors are single details that work together to
create one cohesive theme, which can be best summarized in Option C.

Remember, when thinking about summaries, you can and should be as general as
possible; a summary isnt a blow-by-blow account chock-full of individual details,
but a general gist or zoomed-out picture of the main idea(s). In a simple non-
fiction text, its easy to identify these ideas if you look at the topic sentences of every
paragraph.
Repetition of Ideas

Whenever the writer repeats a string of words, a concept, or idea in the text, its safe
to say that these repetitions will be key in your general summary. Remember, the
writer doesnt have to use the exact same words each time in order to establish
repetition you can repeat ideas without repeating words. But its hard to diversify
the vocabulary when youre explaining the same thought over and over, so theres a
good chance that much of the repetitions will utilize at least some of the same
words, making it easy to skim the material and find the most heavily-emphasized
ideas.
Take the following example, from the Public Library of Science Blog. Here I have
bolded every instance of repetition:
The American Society of Magazine Editors awards are the Academy Awards of
magazinedom, and this year all the finalists in all the long-form, narrative
categories went to men. the dearth of women among the nominees can
perhaps be explained in part by the dearth of female bylines in the sort of
magazines that publish long-form narrative journalism.

At the New Yorker, Harpers, The New Republic and The Atlantic, for instance,
less than thirty percent of the stories published in 2011 were written by
women, according to this years VIDA Count, which did a gender breakdown of
bylines in each magazine.
Take your S AT prep to the next level. Visit www.learnerator.com
75

76


Ive long bemoaned the fact that there are so few women represented among
the contributing editors on many of these publications. (Contributing editors
are the writers whose work youre most likely to see in the mag; they either
have contracts for a certain number of words a year or just enjoy a privileged
relationship with the editors.) Its also true that many of the magazines that
publish narrative pieces are staffed largely by men. Part of the reason is
that a lot of the narrative journalism is published in magazines targeted at
men.

Take Esquire, for instance, which often scoops up ASME nominations and
awards: On the masthead of the March issue, of the 33 editorial staffers listed
including the photo, art, and fashion peopleonly nine are women. And from
what I can tell, only one or possibly two of those are in a position to assign
stories.

2. How would you summarize this article?
A. There are too many male journalists.
B. There are not enough female journalists.
C. Women are not accurately and adequately represented in long-form
journalism.
D. The magazines that publish long-form journalism are at fault for the
lack of female representation.

Remember, summarizing requires finding the general idea based on what the text
actually says. Unlike with other skills on the new SAT, the answers are already
there; all you need to do is rephrase them, rather than making inferences.

Check the bolded portions that repeat the main idea; none of them say that there are
too many male journalists, just that male journalists are overrepresented. Similarly,
this article doesnt say there arent enough female journalists, just that theyre not
Take your S AT prep to the next level. Visit www.learnerator.com
76

77

recognized (or even hired) in the same numbers as men. Nor does the piece
explicitly blame the magazines it just highlights the fact that a discrepancy exists
between male and female representation in journalism. Clearly, all the repetitions
serve to emphasize that female journalists exist, but are simply underrepresented,
as is presented in answer C.
Repetition of Details

Sometimes an author will use an abundance of details to continually demonstrate
one overarching idea. These details will work together to illustrate a fact or concept
that the author wants to emphasize. For example, take this non-fiction piece about
pupfish. Ive bolded every detail that the author includes in order to try and
understand she is trying to emphasize which in turn will lead me to a cogent
summary of this excerpt.

West of Pahrump, Nevada, in a corner of the Mojave Desert a couple
thousand feet above Death Valley, a warm aquifer provides a home for one of
the worlds rarest animals. Its a tiny silvery-blue fish, smaller than your
pinkie toe, and in the past 50 years it has survived real-estate speculators,
death threats, congressional battles, and human screwups. The Devils Hole
pupfishCyprinodondiabolisis nothing if not tenacious.

But the biggest existential threat to the pupfish comes from its own DNA. Once
upon a time, pupfish lived in a sprawling lake. Around 20,000 years ago,
water levels dropped, the landscape turned to desert, and the pupfish ended up
in disconnected ponds. Today, nine different species are scattered across
the Southwest, and half of them are endangered. Devils Hole is the worst case;
as of September 2012, there were 75 fish left. Thousands of years of
adaptation have left the Devils Hole pupfish able to live only in one very
particular environment: It needs 90-degree water, low oxygen, and a
shallow submerged ledge on which to spawn. Its hard enough being
endangered; being endangered and picky is a deadly combination.
Take your S AT prep to the next level. Visit www.learnerator.com
77

78


Think about what these details are doing in this piece: they are establishing a sense
of time and place (Mojave desert, September 2012), drawing a portrait of the fish
(small, tenacious), and establishing what kind of threats the endangered fish face
and why (environment, ability to breed only in very specific places). Though all of
these details are different, they work together to serve the same purpose. Together,
these details will help you answer the question:

3. What is the best summary of this selection?
A. Devils Hole pupfish are tenacious despite the odds they face.
B. Due to a combination of factors, Devils Hole pupfish face extinction.
C. It is crucial that the Devils Hole pupfish continue to survive.
D. Devils Hole pupfish are extremely rare.

Now, many of these details were mentioned in the article. But think about the big
picture that these details work together to illustrate. You need to be able to focus
on one general idea that doesnt extrapolate from the text, but simply gives a big-
picture restatement of what the text actually says. Option A is mentioned in the text,
but it doesnt do a thorough-enough job of summarizing the big picture. This detail
is only mentioned once and isnt explicitly reinforced throughout the text, so it cant
be the best summary of the selection. Option B does a much better job of providing a
summary it mentions a combination of factors (which is general enough to
encapsulate many of the details mentioned in the piece) and neatly summarizes the
point of the selection that, as the last sentence states, being endangered and picky
is a deadly combination; i.e., the pupfish face extinction. Option C is never explicitly
mentioned in the text; while you may be tempted to extrapolate from the text that it
is important for biodiversity that the pupfish survive, stick to what is explicitly
stated. Do not make inferences while trying to summarize. Choice D is true, but it
isnt the best summary its just one small detail that helps to build the bigger
picture of the main idea.

Take your S AT prep to the next level. Visit www.learnerator.com
78

79

Even when the details seem disparate, they often work together to create a cohesive
whole; when in doubt, think about which answer option is best SUPPORTED by the
details, and which options merely help BUILD a larger idea. This will help you better
determine an accurate and succinct summary for the piece.

Take your S AT prep to the next level. Visit www.learnerator.com


79

80

Signal Words and Text Features

When attempting to find key ideas, watch out for words that introduce important
concepts. Words like important, paramount, clearly, and obviously,
essentially, basically, in essence, thus, and ergo among otherscan signal
that the writer is introducing or summing up an essential idea. Words like for
example can help to demonstrate an essential concept. Bolding, italics, and CAPS
can also highlight important ideas, so watch out for those as well. Of course, this
isnt a hard-and-fast rule, but it can be helpful when youre trying to decide which
ideas to isolate.
For example, take this snippet on teaching literacy:

The concept of literacy is fundamentally misunderstood. There is a
misconception that literacy simply means knowing how to read, and while
that is partially true, it doesnt present the whole picture. Knowing how to
read is about much more than just matching up letters with the sounds that
they make; it is also about understanding how information fits together in
context. Students who excel in one context may fail in another because they
lack the multiple types of literacy that are necessary to succeed.

For example, there is a well-known anecdote about a high-level student who
continually failed her math homework. Finally, she was sent to a learning
specialist who tried to diagnose the problem. The specialist asked the student to
read and explain one of the math problems that was giving her so much
trouble. The student read fluently Find the product of two integers, but when
it came time to explain, she was at a loss. She did know how to explain what a
product was, in the mathematical sense, though she could perfectly explain the
concept of a product in a historical/social sciences setting.

Clearly, though she could read perfectly and scored high in all her English
classes, she lacked basic math literacy.
Take your S AT prep to the next level. Visit www.learnerator.com
80

81


There are a lot of signal words here, and they all work together to demonstrate a
concept. Just as in the last passage, there are also details that work together to
present a big picture, which you will be asked to summarize.

4. What is the best summary for this passage?
A. The definition of literacy is disputed.
B. Many people lack basic literacy skills.
C. There are multiple types of literacy.
D. Words can have different meanings, which makes literacy complicated.

Remember, think about how the details and the signal words work together to
create one cohesive idea that is explicitly stated in the text. Nowhere does it say that
the meaning of literacy is disputed, just that it is misunderstood. You can scrap
Option A. While B may be a tempting option, remember that it is never explicitly
stated in the text, and that it is too much of an extrapolation from the detail that
students may fail in certain contexts because they lack a specific type of literacy.
Option C is supported in the text; it is both explicitly stated in the phrase multiple
types of literacy are necessary to succeed and demonstrated in the example of the
student who had one type of literacy but not the other. Lastly, Option D may be true
it is illustrated in the anecdote but it is not the point of the selection; the
selection is there to present a general idea or main point about literacy, and Option
D is too specific.

Take your S AT prep to the next level. Visit www.learnerator.com


81

82

In Summary

Remember that there are multiple ways to come up with a good summary of a text.
Pay attention to structure, repetition of ideas and details, and signal words and text
features. Remember to keep your summary general and big picture instead of
focusing on the multiple smaller details that work together to create the general
idea of the text. Finally, dont extrapolate when presenting a summary, stick to
what is explicitly stated in the text. Utilizing these strategies will help you score high
on any summary questions, on both the SAT and in your classes.

Take your S AT prep to the next level. Visit www.learnerator.com


82

83

Understanding Relationships
Part of demonstrating reading comprehension skills is the ability to understand the
relationships between the individuals, ideas, and events mentioned in a text. This
understanding will help you better understand the selection.
There are multiple types of relationships that may appear in a selection. Here are
some of the most common.
Relationships between Individuals

Selections drawn from narrative pieces will usually feature characters; these
characters will interact in a variety of ways, which will demonstrate their
relationships. The SAT may ask you about the type of relationship that the
characters have to one another are they siblings? Coworkers? Friends? based on
context, or it may ask you to qualitatively describe the relationship amicable?
Acrimonious? based on the interactions that the characters have and the way that
they speak to one another.

Take this example, from Scarlet Stockings, a short story by Louisa May Alcott. As
you read, mark up the points in the passage where the characters reveal through
their speech either the NATURE of their relationship or a CHARACTERIZATION of
their relationship. Pay attention also to characters who are mentioned but not
present, as well as the characters relationship to their setting (where its
applicable).

"COME out for a drive, Harry?"

- One character spends a

"Too cold."

long time clearly trying to

"Have a game of billiards?"

amuse the other, who

"Too tired."

declines to be amused

"Go and call on the Fairchilds?"

- This second character is

Take your S AT prep to the next level. Visit www.learnerator.com


83

84

"Having an unfortunate prejudice against country

kind of a snob

girls, I respectfully decline."

- So these two are sister

"What will you do then?"

and brother

"Nothing, thank you."


And settling himself more luxuriously upon the couch,
Lennox closed his eyes, and appeared to slumber
tranquilly. Kate shook her head, and stood regarding
her brother, despondently, till a sudden idea made her
turn toward the window, exclaiming abruptly,
"Scarlet stockings, Harry!"

"Where?" and, as if the words were a spell to break

- Kate knows how to get

the deepest day-dream, Lennox hurried to the window, her brothers interest
with an unusual expression of interest in his listless

- We can reasonably

face.

extrapolate that Harry has

"I thought that would succeed! She isn't there, but I've

a little crush

got you up, and you are not to go down again," laughed

- Harry is not embarrassed

Kate, taking possession of the sofa.

in the least

"Not a bad maneuver. I don't mind; it's about time

- He clearly likes the red-

for the one interesting event of the day to occur, so

stocking girl, but is pretty

I'll watch for myself, thank you," and Lennox took the

passive about it

easy chair by the window with a shrug and a yawn.


"I'm glad any thing does interest you," said Kate,
petulantly, "though I don't think it amounts to much,
for, though you perch yourself at the window every
day to see that girl pass, you don't care enough
about it to ask her name."
"I've been waiting to be told."
"It's Belle Morgan, the Doctor's daughter, and my

Take your S AT prep to the next level. Visit www.learnerator.com


84

- Another relationship

85

dearest friend."

established

"Then, of course, she is a blue-belle?"

- A potential (and

"Don't try to be witty or sarcastic with her, for she

potentially cantankerous)

will beat you at that."

relationship established

"Not a dumb-belle then?"

- Harry asking if she has a

"Quite the reverse; she talks a good deal, and very well

boyfriend (ma belle = my

too, when she likes."

beauty); character detail

"She is very pretty; has anybody the right to call her

established

'Ma belle'?"

- More relationships

"Many would be glad to do so, but she won't have any

established

thing to say to them."

- Kate clearly cares for and

"A Canterbury belle in every sense of the word then?"

worries about her brother

"She might be, for all Canterbury loves her, but she isn't
fashionable, and has more friends among the poor
than among the rich."
"Ah, I see, a diving-bell, who knows how to go down into
a sea of troubles, and bring up the pearls worth having."
"I'll tell her that, it will please her. You are really
waking up, Harry," and Kate smiled approvingly
upon him.
"This page of 'Belle's Life' is rather amusing, so read
away," said Lennox, glancing up the street, as if he
awaited the appearance of the next edition with
pleasure.
"There isn't much to tell; she is a nice, bright,

- Think of the

energetic, warm-hearted dear; the pride of the

relationships being

Doctor's heart, and a favorite with every one, though established here
she is odd."

- Establishment of a

"How odd?"

possible relationship

Take your S AT prep to the next level. Visit www.learnerator.com


85

86

"Does and says what she likes, is very blunt and honest,

between two people who

has ideas and principles of her own, goes to parties in

havent met yet

high dresses, won't dance round dances, and wears red


stockings, though Mrs. Plantagenet says it's fast."
"Rather a jolly little person, I fancy. Why haven't we
met her at some of the tea-fights and muffin-
worries we've been to lately?"
"It may make you angry, but it will do you good, so I'll

- Clearly, Harry doesnt

tell. She didn't care enough about seeing the

care much for his society.

distinguished stranger to come; that's the truth."

(Dyspepsia = indigestion)

"Sensible girl, to spare herself hours of mortal

- Typical sibling

dullness, gossip, and dyspepsia," was the placid reply.

relationship trying to get

"She has seen you, though, at church and dawdling about a rise out of her brother
town, and she called you 'Sir Charles Coldstream' on the

- Kate criticizing her

spot. How does that suit?" asked Kate, maliciously.

brother (lovingly); Harry

"Not bad, I rather like that. Wish she'd call some day, and demonstrating he doesnt
stir us up."

really care (Im dying of

"She won't; I asked her, but she said she was very busy,

boredom!)

and told Jessy Tudor, she wasn't fond of peacocks."


"I don't exactly see the connection."
"Stupid boy! she meant you, of course."
"Oh, I'm peacocks, am I?"
"I don't wish to be rude, but I really do think you are vain
of your good looks, elegant accomplishments, and the
impression you make wherever you go. When it's worth
while you exert yourself, and are altogether fascinating,
but the 'I come -- see -- and -- conquer' air you put on,
spoils it all for sensible people."
"It strikes me that Miss Morgan has slightly infected you

Take your S AT prep to the next level. Visit www.learnerator.com


86

87

with her oddity as far as bluntness goes. Fire away, it's
rather amusing to be abused when one is dying of
ennui."
"That's grateful and complimentary to me, when I have

- Kate reveals her

devoted myself to you ever since you came. But every

annoyance with her

thing bores you, and the only sign of interest you've

brother

shown is in those absurd red hose. I should like to

- Harrys relationship to

know what the charm is," said Kate, sharply.

the town revealed

"Impossible to say; accept the fact calmly as I do, and be

- Kates relationship to the

grateful that there is one glimpse of color, life, and

town revealed, too --

spirit in this aristocratic tomb of a town."

clearly she feels defensive

"You are not obliged to stay in it!" fiercely.

about it

"Begging your pardon, my dove, but I am. I promised to - Circumstances revealed


give you my enlivening society for a month, and a

More relationships dead

Lennox keeps his word, even at the cost of his life."

parents, a sibling

"I'm sorry I asked such a sacrifice; but I innocently

relationship neglected for

thought that after being away for five long years, you

five years

might care to see your orphan sister," and the dove


produced her handkerchief with a plaintive sniff.

Now that you have read the annotated passage, try your hand at answering these
questions.

What is the best description of Kates feelings towards her brother?
A. Kate is disaffected and disinterested in her brother
B. Kate strives to entice Lennox to become more social, with uneven results
C. Kate is malicious towards Lennox, while he is bored with her
D. Kate is in awe of Lennoxs formidable ego

Take your S AT prep to the next level. Visit www.learnerator.com


87

88

When understanding relationships between characters, you will have to do a good
deal of extrapolating from what is directly stated in the text, but be careful that
every one of your inferences is based on actual evidence presented in the text. Look
at answer A; you should be able to see quickly that while Lennox could conceivably
be called disaffected and disinterested, Kate is clearly neither, as evidenced by her
constant attempts to interest Lennox in the outside world.

Option B is certainly supported by the text; the whole first paragraph is devoted to
Kates attempts to spur Lennox towards some sort of social activity, though he
seems disinterested and criticizes everyone in their town as dull. However, she
does manage to make him to go SOME social events (tea fights and muffin
worries), and to get him talking about his interest in Belle Morgan, so clearly her
efforts are not entirely futile hence why uneven is a good adjective.

Option C could also conceivably be supported by the text the selection does say
that Kate said maliciously and that Lennox complains of ennui, but taken as a
whole, the selection does not paint Kate maliciously. Instead, it is clear that she
cares about her brother and simply wants to try and enliven him. And while D may
also be an option, as Kate does speak about Lennoxs apparently impressive ego, she
does not appear to do so with awe, which is too positive an adjective for her
position. Therefore, B is the only truly defensible option.
Which quote best characterizes the siblings' relationship?
A. I promised to give you my enlivening society for a month, and a Lennox
keeps his word, even at the cost of his life."
B. When it's worthwhile you exert yourself, and are altogether fascinating,
but the 'I come -- see -- and -- conquer' air you put on, spoils it all for sensible
people."
C. Be grateful that there is one glimpse of color, life, and spirit in this
aristocratic tomb of a town."
D. "I thought that would succeed! She isn't there, but I've got you up, and you
are not to go down again," laughed Kate.
Take your S AT prep to the next level. Visit www.learnerator.com
88

89


The use of these quotes will force you to consider and marshal the evidence for your
claims. Lets examine the first clearly, Lennox feels himself duty-bound to his
sister, so while this quote is directly relevant to the relationship between the two
siblings, it is in fact more revealing of Lennoxs relationship to himself than to his
sister. He refers to his own company as My enlivening society (perhaps
facetiously), and makes a claim about himself (A Lennox keeps his word.) While
we could conceivably use this quote to understand Lennoxs relationship to his
sister (perhaps one of duty), it does not seem to accurately reflect the siblings
relationship as a whole, though if there are no better options, this one may be
suitable.
Next, try B; here, Kate is making an observation about her brothers personality.
This quote clearly reveals a feeling she has about one aspect of his personality,
especially as he relates to his society, but it seems to be more a comment about
Lennoxs public face than one about the siblings private relationship.
C reveals Lennoxs feelings about his town, as well as Lennoxs feelings about Belle,
but has nothing to do with Kate herself; it is too much if an extrapolation to suggest
that he finds Kate as boring as the town where they live.
Finally, D reveals both how well Kate knows her brother, as well as the obvious
sense of affection she feels for him. She teases him, rouses him from his stupor, and
makes him come to the window using a tactic only she would know (since it clearly
comes from her observations of him), and gives him a joking directive all signs that
point to an affectionate and close relationship between the siblings. Of all the
quotes, this one best encapsulates the relationship between Kate and Lennox; it is
not the most obvious choice, but it subtly paints the complex relationship between
cantankerous Lennox and affectionate Kate.

Relationships Between Ideas

The SAT will provide you with several nonfiction passages, many of which will
present claims and/or arguments that you will have to assess. Questions may focus
Take your S AT prep to the next level. Visit www.learnerator.com
89

90

on the relationship between one idea and the next. Do the ideas contradict each
other? Build on one another? Influence one another? Work in conjunction?
There are a few ways to understand the relationship between ideas. The first thing
to do is to try and differentiate between the ideas that are being presented. This may
sound simple, but a high-complexity text can present confusing parallels between
ideas, so you want to make sure that understand the distinction between one idea
and the next.

Chronological Signal Words

There are some signal words that should help you make these differentiations. Pay
attention to words that signal chronology:
First, before, next, later, afterwards, used to, recent(ly), now
For example, the following sentence uses signal words to clearly differentiate
between two ideas:

Though researchers used to believe that there could be no life on Mars,
therecent discovery of rocks that appear to be water-weathered has changed this
thinking.

It may be useful to label the ideas outlined here with numbers, so that you can figure
out what scientists used to believe, what they now believe, and what the
relationship is between the two ideas (clearly, the latest research now invalidates
the earlier theory.) Many relationships are often established, changed, or influenced
by TIME, so any words that signal a passage of time are certainly worth paying
attention to.

Relational Signal Words

Take your S AT prep to the next level. Visit www.learnerator.com


90

91

Pay attention also to words like:
While, although, but, however, despite and
In addition, moreover, also, due to, because, thanks to

These can also signal a shift or a change between two schools of thought, which can
be very useful in determining the different relationships between them. Relational
signal words can signal a compare-and-contrast and a cause-and-effect relationship,
which is very important to pay attention to.

Compare and contrast the relationships outlined in the following two sentences:

1.

While there was once widespread support for the existence of Bigfoot,
recent research has definitively disproved this theory. The signal words
should indicate to you that the relationship here is negative one theory
has since replaced the other.

2.

It is thanks to Newtons understanding of gravity that Einsteins notion of


relativity exists.

Clearly, this relationship is positive as one theory builds on or was inspired by the
other. This could in fact be broadly categorized as a cause and effect relationship.
Now that youve practiced with simple samples, try your hand at figuring out the
relationship between the ideas presented in the following blog post from the Public
Library of Science. This is a relatively higher-level text, so pay close attention to the
ideas that are being discussed here.
When I first ran across Asifa Majids article with - Olfaction = smell
Ewelina Wnuk in Cognition, about how speakers - Western society = cares more
of Maniq, a language indigenous to southern

about vision than other senses

Thailand, have a vocabulary for talking about

- Other cultures do not privilege

smell, I was taken aback. In anthropology,

sight over all, neglecting smell

especially since the work of people like David

Take your S AT prep to the next level. Visit www.learnerator.com


91

92

Howes, Constance Classen, and Andrew Synott,
we know very well that different cultures
privilege olfaction and other senses more than
Westerners do. The anthropology of the sense
has made it clear that the
ideological privileging of vision in the
West, and relative underdevelopment of sense
of smell is not matched elsewhere.
However, Wnuk and Majid were

- These two were trying hard to

attacking, with empirical observations and

disprove a theory that already

psychometric testing, one of the pillars of

existed: that evolution favored

Western philosophical accounts of how

other senses over smell, since

human senses evolved: the idea that human

smell was traditionally

evolution had tipped the

considered to be weaker and

balance decisively away from olfaction.

less precise. (This is the OLD

The alleged weakness and imprecision of

theory).

olfaction was taken for granted in perceptual


psychology.
Some of these theories of sensory evolution

- Old theory: our ancestors

hold that our ancestors had, in a way, paid for

evolved because they gave up

our distinctive cognitive and perceptual

on a precise sense of smell; they

development by sacrificing olfactory acuity.

favored vision, which grew

Vision increased precision at the expense of

stronger, as smell grew weaker

olfaction. In fact, some theorists of brain

- Acuity = strength

evolution go so far as to suggest that there

was a kind of neurological trade-off: language

- Also part of the old theory: it

use could only grow as our ancestors lost a

was necessary for us to sacrifice

capacity for smelling. The restraint and

this highly developed sense of

remove from the immediate sense-world

smell so we could develop

Take your S AT prep to the next level. Visit www.learnerator.com


92

93

necessary for logic and abstract thought was

language

opposed to the kind of complete immersion and


sensory triggering of behavior that other

- In other words, other animals

animals had because of the way aromas

were too distracted by smells to

dominated their perception. Were the senses in think clearly; we lost the ability to
a zero-sum exchange where visual acuity and a

smell the way animals do but we

distinctly human way of life made acute

can now think more clearly

olfaction impossible?

because we arent distracted by


our overly-strong senses

Research conducted by Asifa Majid, together - Transition -- new research


with her collaborators, suggests that

shows this OLD theory is untrue

language and olfaction are not at odds; the

the use of language makes our

right language can actually enhance the

senses STRONGER

perception of aroma, as language has also

enhanced, inflected and refined our other

- The fact that our sense of smell

senses. Rather than a fact of human being, the

is weak says more about Western

neglect of olfaction in the West is a result of our culture than the way that we
own cultural presuppositions and sensory

evolved physically

biases: smell suffers from neglect, not an


inescapable evolutionary trade-off.

There are just enough signal words here to help you differentiate between the two
distinct schools of thought.

The writer sets out a sequence of events that STARTS with her coming across a
certain scientists recent work (When I first). She then speaks about what we
know from anthropological research (non-Western cultures privilege smell; the
West cares more about sight), segues into the fact that Majid and Wnuk are trying to
disprove old theories (However), explains the old theories (Some of these
theories In fact), and explains Majid and Wnuks new research.
Take your S AT prep to the next level. Visit www.learnerator.com
93

94

The structure is a little bit confusing because it jumps between present (coming
across this new research) and past (explanation of older theories), but if you read
carefully and pay attention to signal words like First, However, Now,
Disprove, Rather, you should be able to ascertain the sequence of events as well
as the relationship between the ideas explained.

What was Majid and Wnuks aim in conducting this new research?
A. To support the theory that a degraded olfactory sense was evolutionarily
beneficial
B. To disprove the theory that a degraded olfactory sense was evolutionarily
beneficial
C. To demonstrate that language can enhance olfaction
D. To prove that visual acuity can only increase as olfactory acuity degrades

Now, these distractors all use high-level language to try and confuse you, but once
you differentiate between Majid and Wnuks aim and its relationship to older
theories, you will be able to answer the question easily. To begin with, remember
that Majid and Wnuk set out to attack the old theory. So they must be opposed to
it, or trying to disprove it. Remember also that the old theory states that humans
lost their strong, precise sense of smell in order to better develop the vision and the
language that was more evolutionarily advantageous. Majid and Wnuk instead
found that this WASNT true.

Knowing that, lets look at A. Next to degraded olfactory sense was evolutionarily
beneficial, you can write Old Theory. So A means that they wanted to support the
old theory, which simply isnt true, as the article says they wished to attack it.
Scratch A.

B is the exact opposite of A they wanted to DISPROVE this old theory, which is
indeed true. Keep it for now.

Take your S AT prep to the next level. Visit www.learnerator.com


94

95

C is true, according to their research, but it wasnt explicitly stated as the aim for the
scientists research; instead, it was a FINDING of their research. You can scratch C.
D is just a dressed-up version of the old theory (that its a fight to the death between
the sense of sight and the sense of smell, and that one can only grow at the expense
of the other). This is the opposite of what their research suggests as well as the kind
of claim that they sought out specifically to attack, so you can get rid of D.
This was a difficult question because it asks you to do a lot of work simultaneously
(figure out the text structure, figure out the relationship between two ideas, and
figure out the complex language in the distractors).

Take your S AT prep to the next level. Visit www.learnerator.com


95

96

Relationships between Events

Like individuals and ideas, events also have the ability to influence each other. As
you no doubt learned in your social science classes, events have the ability to give
rise to other events; for example, had the 1963 March on Washington never
happened, it is doubtful that the 2008 election would have had the same results. But
historical speculation aside, the SAT will no doubt ask you to identify and perhaps
analyze the relationships between different events. Here are four of the most
common kinds:

Cause and Effect
-Implies one event caused the other
-Signal words: due to, because, since, thus, therefore, hence, since, thanks to, as a
result, consequently
-Example: Due to the rain on Monday (cause), I was unable to walk to school (effect).

Chronology
-Indicates a sequence of events
-Signal words: first, second, third, since, then, now, recently, yesterday, last Monday,
for years
-Example: For decades, scientists have believed the existence of aliens is merely an
overused sci-fi trope; yesterdays surprise visit by friendly extraterrestrials has now
proved, beyond a shadow of a doubt, the existence of intelligent life on other
planets.

Compare and Contrast
-Describes two (or more) similar or disparate events
-Signal words: Also, additionally, moreover, in addition, furthermore (compare);
despite, while, however, yet, but, whereas (contrast)
-Example: The two wars were remarkably similar in their carnage and vastly

Take your S AT prep to the next level. Visit www.learnerator.com


96

97

inflated budget, but as one was effective and the other was not, they will be
remembered very differently.

Though there are other categories, such as Problem and Solution, these are the main
three that you need to worry about; the other categories could neatly fit in to
these. For more practice, read the following short passages and answer the
questions about them:

For years, researchers have disagreed about whether Pluto is in fact a planet.
Now, however, the diminutive ball of ice and methane might be staging its
comeback. The once-planet was reclassified a dwarf planet or plutoid (a
dwarf planet further out than Neptune) in 2003, leading to hundreds of
nostalgic protests, Facebook groups, and even t-shirts all mourning Plutos
demotion. One popular slogan read Pluto will always be a planet in our
hearts.

Based on the passage, which choice best describes the relationship between
Plutos reclassification and the reaction of the public?
A. The public reacted with glee to Plutos reclassification.
B. The public reacted with anger to Plutos reclassification.
C. The public reacted with confusion to Plutos reclassification.
D. The public reacted with sadness to Plutos reclassification.

This is a fairly straightforward cause-and-effect question: what was the effect of
Plutos reclassification on the public? As explicitly stated in the text, the
reclassification led to hundreds of nostalgic protests and mourning; these two
words should clearly establish that D is the only correct choice. You may not have
recognized this as a cause-and-effect relationship, but look at the phrase leading
to. That should clearly indicate exactly what kind of relationship this is.

Take your S AT prep to the next level. Visit www.learnerator.com


97

98

Having the essentially complete sequence of the human genome is similar to
having all the pages of a manual needed to make the human body. The
challenge to researchers and scientists now is to determine how to read the
contents of all these pages and then understand how the parts work together
and to discover the genetic basis for health and the pathology of human
disease. In this respect, genome-based research will eventually enable medical
science to develop highly effective diagnostic tools, to better understand the
health needs of people based on their individual genetic make-ups, and to
design new and highly effective treatments for disease.

Individualized analysis based on each person's genome will lead to a very
powerful form of preventive medicine. We'll be able to learn about risks of
future illness based on DNA analysis. Physicians, nurses, genetic counselors and
other health-care professionals will be able to work with individuals to focus
efforts on the things that are most likely to maintain health for a particular
individual. That might mean diet or lifestyle changes, or it might mean medical
surveillance. But there will be a personalized aspect to what we do to keep
ourselves healthy. Then, through our understanding at the molecular level of
how things like diabetes or heart disease or schizophrenia come about, we
should see a whole new generation of interventions, many of which will be
drugs that are much more effective and precise than those available today.

How will the sequencing of the human genome affect scientific research in the
future?
A. It will teach scientists how to make the human body.
B. It will lead scientists to sequence other organisms genomes.
C. It will increase awareness of public health issues.
D. It will contribute to great gains in preventative medicine.

This is a simple cause-and-effect question that asks about the effect that sequencing
the genome will have on scientific research. Look at all the answer options to
Take your S AT prep to the next level. Visit www.learnerator.com
98

99

determine which is the best one. Though A is clearly drawn from the text, it is an
obvious trick; the passage COMPARES the sequencing of the human genome to the
creation of a manual on how to make a human body. It does not state or imply that
building a human body will be an immediate effect or priority after the sequencing
occurs. Option B has no basis in the text, as there is no statement about scientists
plans for sequencing the genomes of other organisms. Though you may conceivably
be able to extrapolate C from the text, theres no real evidence for this option. Option
D is explicitly stated in the text, and is thus the best answer.

Take your S AT prep to the next level. Visit www.learnerator.com


99

100

From The Atlantic:

The history of clip art is in many ways the history of the World Wide Web
itself. Clip art was made possible because of the invention of desktop publishing
in the early 1980s. The first library of professionally drawn clip art was
provided by VCN ExecuVision and introduced in the IBM PC in 1983. It offered
images to be used in presentations and newsletters. Early electronic clip art
was simple line art or bitmap images. The introduction of the Apple Macintosh
program MacPaint, in particular, meant that consumers could use and edit bit-
mapped clip art for the first time. One of the first successful clip-art producers
was T/Maker Company, which had worked with Apple to develop an
alternative word processor, WriteNow. In 1984, T/Maker began publishing
small, retail collections of those bitmap images under the brand name
"ClickArt." The first version of "ClickArt" offered images designed for both
professional and personal uses; the professional were published in 1984 as
"ClickArt Publications." The widespread adoption of the CD-ROM in the early
1990s was also a boon to clip art: Many more companies began offering
electronic clip art on CDs. As this happened, clip-art makers began emphasizing
quantity over quality. Even T/Maker, which built its business on sales of small,
high-quality clip-art packages, entered the volume clip-art market. In 1995, it
became the exclusive publisher of more than 500,000 copyright-free images
to create what would be, at the time, one of the largest clip-art libraries in the
world.

Based on the information in the article, what could be considered the most
important turning point for clip art?
A. The invention of desktop publishing in the 1980s
B. The release of professional images for public consumption in 1984
C. The 1995 publishing of one of the largest clip-art libraries in the world
D. The adoption of the CD-ROM in the 1990s and the subsequent emphasis on
quantity, not quality
Take your S AT prep to the next level. Visit www.learnerator.com
100

101


This article is clearly structured sequentially, with multiple dates and events
recorded. Though the article never explicitly calls one development more important
than the other, it is possible to extrapolate an answer to this question. Look at the
words used most important TURNING POINT. This should immediately
disqualify option A though the existence of clipart is important, it isnt a turning
point because the beginning by definition cannot be a turning point. Strike A. While
B could be a good choice, its results are never discussed in the article, so it cant be a
turning point as we are never told what it CHANGED. C is highly tempting, but watch
out it is the RESULT of a turning point, rather than the turning point itself. We are
never told what effects of the existence of the largest clip-art library in the world,
but we ARE told that the adoption of the CD-ROM led to an emphasis on quantity,
not quality, which directly leads to the creation of one of the largest clip-art libraries
in the world. In order to answer this question, you must understand what it is
asking youto identify a point in the history of clip art that led to a change, or at
least to important results; only Option D fits this bill.
Understanding relationships between individuals, ideas, or events can be tricky, but
as long as you understand what the question is asking you and which signal words
will lead you to the answer, you will achieve success on this section of the SAT.

Take your S AT prep to the next level. Visit www.learnerator.com


101

102

Interpreting Words and Phrases in Context


One of the most important features on the new SAT is emphasis on vocabulary in
context. Gone are the days of recondite* SAT Words; now you will have to figure
out how high-level yet everyday words fit into the context of a piece. This isnt to say
that the words will all be familiar to you -- dont confuse frequently used with
easy -- but all the words on the new SAT will be relevant to the kinds of higher-
level texts youll be expected to read beyond high school, so practicing this skill now
will help you later on.

Here are some strategies for determining the meaning of words in context.
1. Pay attention to the context.

Sounds obvious, doesnt it? But in order to figure out the meaning of a word,
youll need to pay attention to how it is used. After all, the word product
means one thing in a math context, and quite another in a discussion of tribal
artifacts. So you dont only need to pay attention to the sentence directly
preceding the word in question; pay attention to the setting of the word. Read the
introductory blurb that indicates where the reading comes from. A narrative
piece will use certain words differently than an economics text or a science
article.
Read the following excerpt from a social sciences article about class distinctions,
and see whether you can assess what the words mean.
But its not only linguistic differences that make this transition hard. The way a
person speaks is just a small part of the social currency that determines who is
privileged and who is not. When Lisa Faison, 15, started at her elite private

Take your S AT prep to the next level. Visit www.learnerator.com


102

103

school, she quickly learned that it wasnt normal in her school community to
have no Internet at home. We cant afford it, she shrugs -- an unfamiliar
concept in this world of luxury cars, high-end vacations, and school tuition that
costs upwards of $20,000 a year.
Based on the context, the word currency most probably means
A. Money
B. Exchange
C. Community
D. Expression

Youve probably heard the word currency before in regard to money. But look
at the word before it, social. This modifier clearly indicates that this specific
use of the word currency has something to do with being a part of society. And
the sentence before it, about linguistic habits, also isnt directly relevant to the
issue of money. Money is clearly there as an option to try and confuse you, and
to assess whether you can differentiate between different uses of the same word.
(Plus, social money is a very awkward phrase -- when in doubt, try
substituting the answer option for the word. In some cases, it will be glaringly
obvious that the answer is wrong).
According to the article, currency, whatever it is, determines whether or not a
person is privileged. So currency is something that has a certain power, and its
something that is decided by an external source (social -- clearly, society
determines it).
Look at the other choices. We have exchange, which fits in with the money
idea -- after all, money is exchanged for goods and services. Maybe this type of
currency is also just an exchange -- when you have certain things, or grow up in
a certain community, you are seen in a specific way that others arent. This can
be thought of as a type of exchange -- the things that you HAVE or DO

Take your S AT prep to the next level. Visit www.learnerator.com


103

104

determine the way you are THOUGHT OF or SEEN. Plus social exchange
sounds like it could be an appropriate choice -- the article is about how a girl
from a lower-income background fits in with peers of a higher-income
background, which is certainly a type of social exchange.
Before we decide, lets try C. Social community is redundant -- in fact, its so
redundant that it cant possibly be right. It could have been a good choice, but in
this context it doesnt make any sense.
Lastly, D, expression, is a worthwhile contender. In fact, were it not for
exchange, D would certainly be the answer. After all, social expression -- the
way that people express themselves in society -- goes a very long way in
determining who is privileged and who isnt. But currency seems to be about
more than just expression -- expression suggests that people control the way
they are seen, but the article suggests that this isnt true. Lisa couldnt control the
way her peers saw her; because of one thing beyond her control, they
automatically saw her in a completely different way. This suggests that
currency and expression cannot be synonyms.
The best answer is B, exchange. Not only does it fit best as a synonym, it is also
related to the other meanings of the word currency. This will not always be the
case with words in context -- they will not always be related in meaning to the
same words used in other contexts -- but in this case, its a handy bonus that
helps to determine the best answer.
This was a difficult question because it illustrated a high-level concept; the SAT
will ask many questions like this, which ask you to figure out an ordinary word
in a complex setting.

Take your S AT prep to the next level. Visit www.learnerator.com


104

105

2. Replace, replace, replace.

As mentioned in the previous tip, replacing the word in question with an answer
choice can be a very useful way to determine if the word makes sense. For
example, try this excerpt from a science article.
The experiment had three steps. First, the researchers raised the visibility of the
fruit, moving it from its shadowy spot by the juice dispenser to a prominent
position near the cash register. Next, they lowered the price of the fruit from one
dollar to fifty cents. Last, they plastered images of happy-looking students eating
fruit all over the cafeteria walls, hoping to entice more students to share in this
activity. We did see an increase in fruit intake, said one researcher, but
because of our bundled experiment, were not sure why this campaign succeeded.

Based on the context, bundled most closely means


A. Rushed
B. Bungled
C. Together
D. Multi-step

First determine what this piece is about. Scientists are doing an experiment
involving several elements. Clearly, the experiment cant be rushed if it
contains multiple processes, and theres no mention in the article of a time
constraint on the research. Rushed is therefore not a good choice.
Bungled sounds like bundled. The SAT is relying on this fact -- and on the hope
that you may not be aware that bungled means messed up -- to confuse you.
But choosing a word solely on basis of its perceived similarity to another word
(rather than a shared root) is a terrible strategy, and one that the SAT capitalizes
on to trick you. In any event, it is highly unlikely that the scientist admit to
messing up the experiment that way, so bungled is most likely out.

Take your S AT prep to the next level. Visit www.learnerator.com


105

106

Together may make sense at first, since there were several elements to the
experiment, but read the sentence again: because of our together experiment
doesnt make any sense. Pay attention to parts of speech -- you can do an
experiment together, but you cant do a together experiment. At least not one
that makes sense. Scratch C.
Last, multi-step may not SOUND like its a synonym for bundled, but
remember the first sentence, the one that said the experiment had three steps?
Theres a dead giveaway for the fact that multi-step is the best answer in this
context. Bundled can mean packaged together, which is why together
could have been an attractive option -- but replacing the word with the answer
option will quickly make it clear that sometimes, the most attractive-seeming
options are not very feasible at all.
3. Visualize.

Research has demonstrated that visuals help you learn vocabulary. There will be
diagrams and charts on the new SAT, but no pictures -- which is why you can be
your own illustrator. You dont need to sketch out a perfect image to understand
the meaning of a word -- this is a strategy that will work just fine if you keep it
confined to your own head -- but taking the time to visualize what the article is
talking about will be very helpful as you answer questions designed to test your
comprehension.
For example, try visualizing this excerpt, from a social-science article.
But being a consumer and not a producer has its drawbacks. Those with
frustrated artistic inclinations often cite a general fatigue with consumption,
complaining of an exhausting glut that can only be counterbalanced by the hard
work of producing.

Take your S AT prep to the next level. Visit www.learnerator.com


106

107

This is a hard image to visualize in your head, as there isnt much imagery or a
sense of scene or setting. But there are some words that should help -- fatigue,
complaining, exhausting, hard work. Imagine a consumer -- someone who
consumes, or eats, or takes in something. (I am imagining a little Pac-Manlike creature.) Imagine this over-consumer getting tired -- consuming too much
and slowing down.
This exhaustion can only be counterbalanced or reversed, by the consumer
making something of his or her own (producing). Now I am imagining that
little Pac-Man creature staying in one place, creating something, losing all that
excess weight of consumption and getting energized again, thanks to the hard
work of producing.
The visual came to my head -- I didnt draw it or otherwise mark it down -- but it
helped me to better understand the piece by anchoring the abstract (all this talk
of producers and consumers and artistic fatigue) to an actual representation of
the concepts.
Try visualization as you answer this final question about words in context. The
following excerpt is from a literary piece.
She tensed, expecting the lights to blare and the Matrons voice to sound over the
loudspeakers wired into every room. But all was quiet and still. The night was a
neverending black and not a single bird sang or swooped in the trees. She took a
deep, quavery breath and stepped out into the uncharted night.

Based on the reading, what is the best meaning for the word uncharted?
A. Unknown
B. Wild
C. Frightening
D. Shadowy

Take your S AT prep to the next level. Visit www.learnerator.com


107

108

From the context, you can probably figure out that there is something a little bit
ominous happening here -- our protagonist is tensing in fear of a Matron
who can project her voice into every room (so the protagonist lives in some sort
of dystopian communal housing). The neverending black of the night and the
silence of the birds also contribute to this ominous impression. Based on this
reading, you may be tempted to choose C, frightening -- but despite this
ominous impression, the text does not say that the night is frightening. After all,
our protagonist is walking into it willingly. Nor do we have any indication that
the night is wild (there is no mention of tempestuous weather or wild
animals), and shadowy is an overly redundant synonym for neverending
black (plus, shadows need at least some light to exist, and there clearly isnt any
of that here.)
The best definition for uncharted is unknown -- uncharted territory is
territory that hasnt been put on a map yet, territory that isnt familiar.
Of all the choices, A and C are the strongest; in this case, the best you can do is
use context and common sense to eliminate the answer choices that are clearly
wrong and use your vocabulary knowledge (or your best guess) to determine the
correct answer.
Figuring out words and phrases in context can be tricky, and sometimes there
isn't a replacement for simply knowing what a word means. But if you use these
tips -- paying attention to every part of the context, replacing the word in
question with the answer options to see if it makes sense, and visualizing the
context as best you can -- this will be very helpful in figuring out the meaning of
vocabulary in context.


Take your S AT prep to the next level. Visit www.learnerator.com
108

109







Part II: Rhetoric (Reading)

Take your S AT prep to the next level. Visit www.learnerator.com


109

110

Analyzing Word Choice


Rhetoric is defined as the art of discourse, or (more informally) the art of
persuasion. The new SAT is preparing you to read, analyze, and internalize
information that you might see in college and the workplace -- mostly persuasive
writing that attempts to get an important point across succinctly and informatively.
In this section of the review, youll learn some strategies for:
1. Comprehending the structure of a persuasive piece
2. Analyzing the choices that a writer makes in order for a piece to become
persuasive
3. Understanding why a piece was written
4. Understanding how each element in the piece works within the larger context

This particular article will focus on that last issue; when analyzing rhetoric, you can
break down the argument into its smallest pieces, which are the words used to
effectively convey the idea in question. You may be familiar with Common Core
standards -- or with English teachers -- that ask you to determine how an authors
writing conveys or creates meaning and tone. This is the exact same skill, only
applied to a broader swath of prose than just literature.
This skill could also be called paying close attention. As you read, take note of the
words that create some sort of feeling or image. Does the author use bland, boring
words, or does s/he use intense, vivid language to evoke a particular mood or
scene? Take these two sentences as examples. Both say the same thing, yet one
evokes a mood much more vividly than the other:

I was tired and hungry when I came home the other day.
Famished and yawning, I trudged up the steps to my empty apartment.

The latter sentence clearly uses far more vivid language. Its not just the
replacement of famished for hungry or yawning for tired (though yawning,
Take your S AT prep to the next level. Visit www.learnerator.com
110

111

a verb, conveys a sense of action that tired does not) that creates such a strong
sense of emotion. Its also the inclusion of the word empty before apartment --
the first sentence tells us I came home, a rather neutral statement, while this
sentence describes the home. It is empty and isolated. It even gives us a glimpse at
the kind of home it is, evoking images of stairwells, the city, and isolation.

Lets try this one more time:
Stephanie Carmichael was a bully. There were no two ways about it.
Stephanie Carmichael liked to hit people. Unfortunately, her victims did not
share her enthusiasm.

Now, both of these sentences are equally vivid, but the tones are completely
different. Take the first sentence -- it is conversational, utilizing informal, everyday
language (no two ways about it.) The second sentence is more formal, almost
sardonic; it states the obvious (there arent many people who enjoy being hit), but
slightly dramatizes the situation by calling Stephanies targets victims and
referring to her penchant for hitting as an enthusiasm.
Even though the two sentences say the same thing in equally vivid language, the
tones are vastly different because of the words being used. Pay close attention to
distinctions like this and you will do very well on this section of the SAT. It can be
very helpful to visualize everything that the author describes, as this will help to
establish a sense of tone, mood, and place. When I saw the words no two ways
about it, I pictured a down-home, folksy type; when I read, her victims did not
share her enthusiasm, I pictured Benedict Cumberbatch as the narrator -- refined,
sardonic, and very British. Clearly, the words used can go a long way in evoking a
very particular image.

Extra Practice

As you read, think about how the writer uses words to convey information. Take a
look at this following example, paying attention to how s/he structures and words
Take your S AT prep to the next level. Visit www.learnerator.com
111

112

the argument. If possible, try to visualize the person who wrote this piece. Use their
words to get a sense for how they feel and what they think about the topic in
question:

The recent exclusion of the Harry Potter series from your newspapers Best
Books Ever Written list is an unqualified travesty. I am outraged and confused
that your magazine would choose to exclude such a seminal work of literature
from the Western canon. The Harry Potter books rank among the best ever
written, inspiring countless generations of children and inculcating them with
a sense of wonder and delight. Rest assured that this snub will not be ignored;
due to your lack of research and clearly unsound opinions, I am canceling my
subscription effective immediately.

What is the general tone of the article?
A. Incensed
B. Delighted
C. Unqualified
D. Distressed
Though this question does not directly ask you to pinpoint one word or phrase, it
asks you to consider the words used in this article as a whole. It should be fairly
easy to determine that this writer is clearly not delighted (B). Nor is s/he
unqualified -- though this word is used in the piece, it is an adjective (meaning
total or complete) for the travesty of the exclusion of Harry Potter from a Best
Of list. It is clear that the writer is D, distressed -- words like outraged and
confused make that clear -- but a better answer is A, Incensed, or angry. The
writer is clearly more than just distressed -- s/he is angry, threatening to cancel
his/her subscription to the magazine and threatening this snub will not be
ignored. This sounds like a lot more than just distress. As you read, make sure
that youre paying attention to the piece as a whole and considering the very best
answer.

Take your S AT prep to the next level. Visit www.learnerator.com
112

113

Which quote from the article best conveys the authors tone?
A. The Harry Potter books rank among the best ever written
B. Due to your lack of research and clearly unsound opinions
C. I am canceling my subscription
D. The exclusion is an unqualified travesty

This question is asking you HOW the writer has conveyed his/her tone of acute
anger. Though each of the quotes is lifted directly from the text, only one best
conveys a tone of rage. Clearly, A, while an important and closely held opinion, does
not convey the authors rage, but the authors opinion about the Harry Potter series.
B gets closer to the authors rage, as it insults the writers in the magazines, but it
does not contain any of the very strong language that the writer used in other parts
of the selection. C contains a threat, but no strong language. Clearly, D is the best
answer as it most effectively conveys the authors sense of rage.


Try practicing with a slightly less colorful selection from a history text:
The senator had little to say about the barbarism of the crime, bizarrely choosing
instead to direct his vitriol towards the victim and her family rather than the
perpetrators, who were released on bail just two days after the attack. I appeal to the
American public tonight, he stated in a press conference last Monday, to ask why this
girl was allowed to run amok after dark by herself.

What is the authors opinion of the senators remarks?
A. The author supports the senators appeal to the nation.
B. The author believes the senators remarks are a barbaric crime.
C. The author is bewildered and angered by the senators remarks.
D. The author has no opinion on the senators remarks.

In order to correctly answer this question, you must separate out the words that the
author uses to refer to the senator, and the words that the author uses to refer to the
Take your S AT prep to the next level. Visit www.learnerator.com
113

114

crime. You must understand also what the senators remarks WERE -- refer to the
quotes, which enclose the Senators words. (Understanding how this text is
structured will come in useful here.)

Be wary of answer choices that quote directly from the text -- they may be trying to
confuse you by drawing your attention to an irrelevant part of the selection. For
example, Answer A contains the words appeal to the nation, which are lifted
directly from the text. Yet there is absolutely no indication that the author supports
this appeal; in fact, it is the senator who used the word appeal, not the author.
Answer B also directly refers to a quote from the text, but again, the barbaric
crime has nothing to do with the senators words; the senators speech was a
response to the crime. Option C is directly supported by the text, as s/he says that
the senator bizarrely chose to direct his vitriol towards the victim. This use of
the word "bizarre in reference to the Senators choices showcases the authors
confusion about the politicians actions; the use of the word vitriol (cruel or bitter
criticism) also illustrates the authors position. After all, the Senator is
not actually being vitriolic; his language is not harsh enough for that. Yet clearly the
author feels that the Senators remarks are so unfair that they border on abuse -- a
position that is clearly if subtly demonstrated throughout the piece. You may be
tempted to say that D, the author has no opinion, but the word bizarre, again,
demonstrates that this isnt true.

Lets try once more; pay careful attention to how the author creates tone and mood
here. Which words are the most useful in the creation of this scene? Which words
best help you to visualize what the author is trying to convey?
As the long slog towards establishing fair trade practices in Equatorial Kundu
continues, there is a glimmer of hope on the horizon. The West African country has
long been the focus of controversy for its corrupt practices regarding the mining of
diamonds and the harvesting of coffee, but with the recent installation of reforming
President Mbala Mbame, new legislation could change the way miners and harvesters
are treated by internal and international trading entities. We will no longer stand by
Take your S AT prep to the next level. Visit www.learnerator.com
114

115

and watch the common people suffer, President Mbame announced in a historic
televised speech yesterday. Equatorial Kundu is facing the dawn of a new day, and
anyone who stands in our way will be dealt with summarily. There is no word yet
from the BPA, the rebel group that led riots in the streets in response to Mbames
election, on how these new agreements will impact their flourishing black market
economy, but Kundus army is on standby and the country is both tense and hopeful.

How does the author paint a vivid image for the reader?
A. S/he chose a strongly worded quote from the president of the country,
which establishes a display of strength.
B. S/he draws attention to the plight of the miners and harvesters, helping
the audience to understand the dire economic conditions of Kundu.
C. S/he speculates about the reprisals of the BPA, establishing a fearful tone
for the piece.
D. S/he uses multiple descriptive words, such as slog, glimmer, tense,
and hopeful to establish Kundus general mood.

Many of these choices seem as though they may be correct. For example, choice A is
at least somewhat true -- President Mbames speech was worded somewhat
threateningly (anyone who stands in our way will be dealt with summarily). Yet
the piece isnt about the display of strength of the new president; its about the
changing conditions in the country, and the possible hope of a revitalized economy
for the lower classes. While President Mbames quote certainly evokes a mood, it
does not help establish the mood of this piece.

Option B is also attractive, but clearly not correct; the writer does not detail the
plight of the harvesters or the miners at all. S/he mentions these low-income
occupations, but does not use any evocative language whatsoever to describe their
lifestyles or issues. C is also clearly incorrect; there is no speculation in the piece,
only a statement of facts (that the army is on standby and that there has not yet
been any contact from the rebel group). Finally, the author DOES use descriptive
Take your S AT prep to the next level. Visit www.learnerator.com
115

116

words that clearly paint an image of Kundu as being on a long road to recovery; the
word slog corresponds to the difficulties that the miners, harvesters, and new
President face as they stand up to violent rebel groups and take back what is
rightfully theirs. The word glimmer of hope corresponds to the hope mentioned
at the end of the article but emphasizes that the battle hasnt been won yet. The
tension is palpable, and the writer reiterates it by detailing the struggles that
President Mbame and his people will face in the future.

As always with these types of questions, it is crucial that you pay close attention to
what is stated, what can be reasonably inferred, and what is simply wild
speculation. This will help you determine which words are most important in
context.
Some types of passages will make analyzing word choice easier than others.
Different texts seek to do different things. For example, an objective piece that
clearly lays out a number of facts without editorializing will not necessarily have
lots of strong words you can point to as creating a tone, but a literary work or an
editorial will certainly use all manner of rhetorical devices from strong language to
evocative words in order to create meaning or mood. Remember that rhetoric
means argument, so when analyzing arguments, be on the lookout for strong
language and interesting, unusual, or particularly meaningful word choices.

Take your S AT prep to the next level. Visit www.learnerator.com


116

117

Analyzing Overall Text Structure


It may not seem like it, but all texts, like houses, are constructed carefully and in a
specific way. Even the most stream-of-consciousness modernist text has a structure
and a plan; a lot of work goes into polished, professional writing, and it is safe to
assume that there is a strong relationship between a texts aims (what it is trying to
do, say, convey) and the way it is organized.
It can be helpful to think of a text like a house. Just as houses are built to
accommodate the specific needs of the owner -- taking into consideration climate,
family size, and personal taste -- texts are tailored to best convey specific
information. For example, a narrative text that seeks to tell a story will be structured
very differently from a persuasive text that seeks to inform or argue a point. Here
are some text structures that may crop up on the new SAT:

Narrative Texts

There will be one reading from US and/or World Literature on the new SAT. It could
be structured in a variety of ways, including:
-a linear story, with a traditional plot arc (beginning, middle, end)
-a non-linear story, told in flashbacks or switching between past, present, and future
events
-stream-of-consciousness (this is unlikely, as these types of texts are quite high-level
and difficult to analyze, but its best to be prepared!)
Regardless of the structure, the resulting text will most likely be heavy with
description -- adjectives, imagery, and other strategies intended to get the reader to
think, feel, imagine, or visualize something. The author might center a piece directly
around dialogue, letting the reader understand the character dynamics and predict
the plot using only the characters words as a guidepost. For example, in Louisa May
Alcotts story Scarlet Stockings, the characters relationship and a probable sequence
of events emerge clearly from the following dialogue:
Take your S AT prep to the next level. Visit www.learnerator.com
117

118


"COME out for a drive, Harry?"
"Too cold."
"Have a game of billiards?"
"Too tired."
"Go and call on the Fairchilds?"
"Having an unfortunate prejudice against country girls, I respectfully decline."
"What will you do then?"
"Nothing, thank you."
And settling himself more luxuriously upon the couch, Lennox closed his eyes,
and appeared to slumber tranquilly. Kate shook her head, and stood regarding
her brother, despondently, till a sudden idea made her turn toward the window,
exclaiming abruptly,
"Scarlet stockings, Harry!"
"Where?" and, as if the words were a spell to break the deepest day-dream,
Lennox hurried to the window, with an unusual expression of interest in his
listless face.
"I thought that would succeed! She isn't there, but I've got you up, and you are
not to go down again," laughed Kate, taking possession of the sofa.
"Not a bad maneuver. I don't mind; it's about time for the one interesting event
of the day to occur, so I'll watch for myself, thank you," and Lennox took the
easy chair by the window with a shrug and a yawn.
"I'm glad any thing does interest you," said Kate, petulantly, "though I don't
think it amounts to much, for, though you perch yourself at the window every
day to see that girl pass, you don't care enough about it to ask her name."
"I've been waiting to be told."
"It's Belle Morgan, the Doctor's daughter, and my dearest friend."
"Then, of course, she is a blue-belle?"
"Don't try to be witty or sarcastic with her, for she will beat you at that."
"Not a dumb-belle then?"
"Quite the reverse; she talks a good deal, and very well too, when she likes."
Take your S AT prep to the next level. Visit www.learnerator.com
118

119

"She is very pretty; has anybody the right to call her 'Ma belle'?"
"Many would be glad to do so, but she won't have any thing to say to them."
"A Canterbury belle in every sense of the word then?"
"She might be, for all Canterbury loves her, but she isn't fashionable, and has
more friends among the poor than among the rich."
"Ah, I see, a diving-bell, who knows how to go down into a sea of troubles, and
bring up the pearls worth having."
"I'll tell her that, it will please her. You are really waking up, Harry," and Kate
smiled approvingly upon him.
"This page of 'Belle's Life' is rather amusing, so read away," said Lennox,
glancing up the street, as if he awaited the appearance of the next edition with
pleasure.
"There isn't much to tell; she is a nice, bright, energetic, warm-hearted dear;
the pride of the Doctor's heart, and a favorite with every one, though she is odd.
"How odd?"
"Does and says what she likes, is very blunt and honest, has ideas and principles
of her own, goes to parties in high dresses, won't dance round dances, and
wears red stockings, though Mrs. Plantagenet says it's fast."
"Rather a jolly little person, I fancy. Why haven't we met her at some of the tea-
fights and muffin-worries we've been to lately?"
"It may make you angry, but it will do you good, so I'll tell. She didn't care
enough about seeing the distinguished stranger to come; that's the truth."
"Sensible girl, to spare herself hours of mortal dullness, gossip, and dyspepsia,"
was the placid reply.
"She has seen you, though, at church and dawdling about town, and she called
you 'Sir Charles Coldstream' on the spot. How does that suit?" asked Kate,
maliciously.
"Not bad, I rather like that. Wish she'd call some day, and stir us up."
"She won't; I asked her, but she said she was very busy, and told Jessy Tudor, she
wasn't fond of peacocks."
"I don't exactly see the connection."
Take your S AT prep to the next level. Visit www.learnerator.com
119

120

"Stupid boy! she meant you, of course."
"Oh, I'm peacocks, am I?"
"I don't wish to be rude, but I really do think you are vain of your good looks,
elegant accomplishments, and the impression you make wherever you go. When
it's worth while you exert yourself, and are altogether fascinating, but the 'I
come -- see -- and -- conquer' air you put on, spoils it all for sensible people."
"It strikes me that Miss Morgan has slightly infected you with her oddity as far
as bluntness goes. Fire away, it's rather amusing to be abused when one is
dying of ennui."
"That's grateful and complimentary to me, when I have devoted myself to you
ever since you came. But every thing bores you, and the only sign of interest
you've shown is in those absurd red hose. I should like to know what the charm
is," said Kate, sharply.
"Impossible to say; accept the fact calmly as I do, and be grateful that there is
one glimpse of color, life, and spirit in this aristocratic tomb of a town."
"You are not obliged to stay in it!" fiercely.
"Begging your pardon, my dove, but I am. I promised to give you my enlivening
society for a month, and a Lennox keeps his word, even at the cost of his life."
"I'm sorry I asked such a sacrifice; but I innocently thought that after being
away for five long years, you might care to see your orphan sister," and the
dove produced her handkerchief with a plaintive sniff.
"Now, my dear creature, don't be melodramatic, I beg of you," cried her
brother, imploringly. "I wished to come, I pined to embrace you, and I give you
my word, I don't blame you for the stupidity of this confounded place."

Why does Louisa May Alcott choose to begin her story this way?
A. The dialogue helps establish the characters, setting, and plot without too
much exposition
B. The dialogue reveals the tragic pasts of her orphaned narrators and helps
establish a sense of sympathy
C. The dialogue reveals what a cad Harry is, setting him up for the failure he
Take your S AT prep to the next level. Visit www.learnerator.com
120

121

is likely to endure by the end of the story
D. The dialogue helps establish a love triangle that is likely to be the focus of
the storys plot

This question may not seem like its asking you about structure, but in fact that is
exactly what it is doing -- trying to get a sense for whether or not you understand
WHY an author made a certain choice in writing a story and whether or not that
choice is effective. (It also asks you to consider the authors purpose as well as the
relationship of one part of a story to the whole of the story -- three skills for the
price of one!)

Consider the answer choices. Answer A is certainly correct -- we do have a sense of
the characters interests, relationships, and personality (Harry is a grouch with a
crush, Kate is a sweetheart who loves and worries about her brother, and Belle is
apparently perfect). But the key here is the without too much exposition -- world-
building is difficult, and it takes skill to inundate the reader with information about
the characters and plot without being boring or redundant. In effect, Alcotts clever
in medias res (in the middle of things) dialogue strategy plops the reader directly
into the Lennoxes sitting room, eliminating the need for a tiresome introduction
that would have gone something like this: Harry and Kate were brother and sister,
but Harry had been away in France for five years and was pretty cranky about having
to be back in a boring old town that he hated. And so forth.

But lets consider B. The specificity of the other answer choices certainly makes
them tempting, but consider them closely: while its TRUE that Harry and Kate are
orphaned, this is in no way the purpose or entirety of their dialogue. They mention
it once, towards the end of the selection, and while the background helps establish a
sense of context, it is in no way the focus of the piece. Nor does Harry come off as
being particularly sympathetic, and Kate doesnt need the tragic orphan detail to
become sympathetic -- everything she does, from striving to set up her brother and
her best friend to her strategy of caring for her brother, establishes her as a
Take your S AT prep to the next level. Visit www.learnerator.com
121

122

sympathetic character. So beware of extreme specificity -- with questions like this,
which ask about a piece as a whole, generalized answers will often work better than
specific ones.

Cs flaw is similar to Bs; it is simply too specific, plus there isnt a ton of
support/evidence for it. Yes, Harrys kind of a cad -- but its highly unlikely that his
sister, who clearly loves and cares about him and her best friend, would be plotting
to set the two up if she didnt think they would work well together. Plus, theres
simply no evidence that the two wont hit it off -- in fact, the evidence is to the
contrary, since the chances are that youve seen some variant of this good girl
reforms bad boy trope before. Scratch C.

Ditto D. You may be tempted to pick this one because the dialogue does indeed
establish a love story that is integral to the storys plot. But please beware of this
expression love triangle. As of now, there are only two players in this potential
love story -- Harry and Belle. Kate certainly isnt part of the love triangle, as shes
Harrys sister and playing matchmaker, not striving for the romantic affection of
either party. Pay attention to small details like this.

Narrative texts may also be structured a-chronologically, utilizing literary devices
such as flashbacks, flash forwards/foreshadowing, or internal dialogue. For
example, look at this following excerpt, which is heavy on internal dialogue:

I hate this stupid place, thought Princess Bluebell to herself as she trudged up
the stairs to her new boarding school, The KAPS Academy for Young Heroines.
I hate horses, she thought murderously, eyeing the portraits of noble steeds
that lined the hallways. They smell terrible. I hate archery, she continued,
looking askance at the bust of Diana the Huntress that dominated the front
entryway. It gives me splinters. And I hate, hate, hate that the whole aim of
this stupid school is to teach me how to be my own hero instead of waiting
around for my handsome prince. At this, she sighed, pulling out her wallet-size
Take your S AT prep to the next level. Visit www.learnerator.com
122

123

photo of Charming, the handsome prince next door who was no doubt going to
stop loving her when she came back to the castle over summer break, all
tanned and strong and able to rescue herself instead of languishing pale and
malnourished in some attic tower guarded by a dragon. Everyone knew princes
had no use for strong girls. He would stop loving her in an instant! Well, I just
wont learn anything, she swore, and Mom and Dad will let me come home
after a semester and finally send me off to that witch whos been agitating to
keep me locked in a tower until I turn 16 and Charming can come rescue me.
Everyone wants me to be master of my own fate. Ill beat them at their game!
A wicked smile curved her lips, and she headed up the stairs to her first class,
Damsels, Maidens, and Other Helpless Twits: Defying the Stereotypes and
Daring to Be Your Own Hero, determined not to learn a single thing.

What is the primary effect of this internal dialogue on the structure of
the text as a whole?
A. The internal dialogue establishes Bluebell as a sympathetic character and
invites the reader to admire her master plan
B. The internal dialogue establishes Bluebell as a foolish character deserving
of mockery and invites the reader to laugh at her
C. The internal dialogue establishes an arc for plot and character
development
D. The internal dialogue establishes the passionate love story sure to ensue
between Bluebell and Charming

Again, remember that in questions about text structure, general is (generally) better
than super-specific. Pay attention also to the word primary -- what is the MAIN
effect, or the MAIN purpose of this internal dialogue? A may be half-plausible (its
possible that the author wants to portray Bluebell as a sympathetic character,
though from the gently mocking tone its fairly clear this isnt entirely the case) but
surely the second clause of the sentence (invites the reader to admire her master

Take your S AT prep to the next level. Visit www.learnerator.com


123

124

plan) will make it easy to scratch A, as Bluebells plan is fairly foolish and trades
heavily in antiquated stereotypes about femininity.

B is plausible too, but not entirely likely -- after all, the question asks about the
PRIMARY effect of the internal dialogue. We may be invited to roll our eyes a little
bit at Bluebell, but it is unlikely that the author would have written this piece
specifically to make fun of the main character as this is a fairly big waste of
everyones time and makes for a less than enjoyable reading experience. So while
this could have been a plausible choice had the question not been asking about the
PRIMARY effect of this dialogue, it isnt entirely likely to be the main reason the
author uses this literary device. C, again, is a nice general answer and entirely true --
this sets Bluebell up to probably revise her thinking, and also gives us a source of
conflict / tension, which help propel the plot forward. D is unlikely; there is
certainly a love interest, but its hard to say what will ensue between Bluebell and
Charming, especially considering that she is about to (probably) go through some
very intense changes to her worldview. So again, the general answer that focuses on
the CRAFT -- the underlying structure -- of a story rather than a specific element of
the story.

Expository Texts
Expository texts will make up the bulk of the passages on the new SAT, so its
important to understand how their structures will affect their content. As discussed
in a previous article, there are multiple types of expository text structures, and
several ways to determine what they are. Sometimes texts will fall into easy-to-spot
structures (chronological, cause and effect, compare and contrast), all of which are
briefly reviewed here. But there are multiple devices that authors use to frame
arguments and ideas that dont necessarily fit into these three neat categories, or
that utilize elements from one or more of them in order to effectively establish a
claim. Those will be reviewed as well.

Take your S AT prep to the next level. Visit www.learnerator.com


124

125

Chronology
A piece that is structured chronologically or sequentially will have all sorts of time
words to indicate the sequence of events: first, last, second, last Tuesday, then,
afterwards, before. Writers often use this type of structure to frame a journalistic
piece whose purpose is to accurately convey information in the order in which it
happened. This is useful in pieces about current events (or historical events), where
dates and times are very important as they influence whatever will happen next. For
example, check out this article on the history of space exploration, from
aerospace.org. As you read, make note of all the chronological signal words:

Humans have dreamed about spaceflight since antiquity. The Chinese used
rockets for ceremonial and military purposes centuries ago, but only in
the latter half of the 20th century were rockets developed that were powerful
enough to overcome the force of gravity to reach orbital velocities that could
open space to human exploration.

As often happens in science, the earliest practical work on rocket engines
designed for spaceflight occurred simultaneously during the early 20th
century in three countries by three key scientists: in Russia, by Konstantin
Tsiolkovski; in the United States, by Robert Goddard; and in Germany, by
Hermann Oberth.

In the 1930s and 1940s Nazi Germany saw the possibilities of using long-
distance rockets as weapons. Late in World War II, London was attacked by
200-mile-range V-2 missiles, which arched 60 miles high over the English
Channel at more than 3,500 miles per hour.

After World War II, the United States and the Soviet Union created their own
missile programs. On October 4, 1957, the Soviets launched the first artificial
satellite, Sputnik 1, into space. Four years later on April 12, 1961, Russian Lt.
Yuri Gagarin became the first human to orbit Earth in Vostok 1. His flight
Take your S AT prep to the next level. Visit www.learnerator.com
125

126

lasted 108 minutes, and Gagarin reached an altitude of 327 kilometers (about
202 miles).
The first U.S. satellite, Explorer 1, went into orbit on January 31, 1958. In
1961 Alan Shepard became the first American to fly into space. On February
20, 1962, John Glenns historic flight made him the first American to orbit
Earth.

Landing a man on the moon and returning him safely to Earth within a
decade was a national goal set by President John F. Kennedy in 1961. On July
20, 1969, Astronaut Neil Armstrong took a giant step for mankind as he
stepped onto the moon. Six Apollo missions were made to explore the moon
between 1969 and 1972.

Recognizing these signal words will help you better understand the structure of the
text -- and will also help you to answer questions that ask specifically about a
sequence of events. For example, a question may ask you about the first person to
enter space, the first American to orbit Earth, the first person to step on the moon --
all distinct characters. You will have to pay attention to the signal words and the
dates in order to answer those questions accurately. A question about the structure
of the text may ask you something like:

Why does the author choose to introduce the article with the example of
ancient Chinese ceremonial rockets?
A, These were the first rockets into space, and as such they are notable and
relevant to an article about space exploration
B, The article is about Chinese space exploration, and so these rockets are a
valuable precursor to the article as they help to better understand the
timeline
C, These rockets are a successful framing device that help to introduce the
history of space exploration, from rockets that tried and failed to reach space

Take your S AT prep to the next level. Visit www.learnerator.com


126

127

to rockets that landed human beings on the moon
D, The use of the Chinese rockets example is irrelevant to the text as a whole

Of course, using what you already know about text structure -- go general, think
about craft and purpose, and pay close attention -- it is pretty clear that C is the best
answer. A is incorrect as the article clearly states that these ancient rockets didnt
reach space; B is clearly incorrect because the Chinese are not mentioned as being
pioneers of space exploration; and D simply isnt true because the example is indeed
relevant in kicking off the article.

Take your S AT prep to the next level. Visit www.learnerator.com


127

128

The new SAT may also focus on questions such as:

What is the effect of using multiple dates in this article to introduce
different events?
A. The use of dates imparts information in the simplest, most objective, and
most historically accurate way
B. The use of dates is meant to demonstrate how many years it took for
Americans to achieve space flight
C. The use of dates is meant to highlight Soviet achievement at the expense of
American achievement
D. The use of dates is meant to demonstrate that it has been a long time since
the American economy could support exploratory space flight

This question asks you to analyze why an author chose a specific structure, and
potentially to evaluate whether this choice was the most effective one. Again, using
our strategy of being GENERAL and focusing on CRAFT and STRUCTURE rather than
specific pieces of content, it should be fairly clear that A is the only correct answer.
The objective tone of the article (it uses no laudatory adjectives and offers no
opinions, just lays out facts) makes it obvious that the author is not interested in
criticizing or lauding either the Americans or the Soviets, eliminating B and C. And
there is simply no support in the article for D, as there is no mention of the economy.

Take your S AT prep to the next level. Visit www.learnerator.com


128

129

Cause and Effect
Use of the cause and effect structure is especially prevalent in science or history
articles that seek to explain or understand specific phenomena. This structure
quickly becomes evident with words like due to, thanks to, because of, leading
to, resulting in and the like. This article about diversity on television uses a very
subtle cause and effect structure; as you read, pay attention to words that signal a
cause and an effect.

Though the demographics represented on television have shifted -- with greater
representation of blended, mixed, or minority families -- there is still work to be
done. Blonda Thimes, the creative mastermind behind such long-running
blockbuster hits as Shays Anatomy, Disaster, and How to Get Away With A
Terrible Crime, has often been credited for bringing minority actors to the
forefront of American television. Lauded for her color-blind casting, Thimes
had a lot to say when we contacted her on Thursday. Its pretty sad, Blonda
says, that in 2014 its still somehow notable or strange that a black woman is a
heroine or a main character instead of a sassy friend or sidekick. Thimes
casting has also led to a minor revolution in the representation of gay and
lesbian characters on television, as her shows focus on diverse representations
that stray quite far from stereotypical depictions of this minority group. Thimes
called these stereotypes lazy, pathetic writing and claimed that they are
responsible for mass social ignorance.

Now, there are multiple causes and effects here, so pay close attention to how the
author chooses to present the multiple factors that went into this article.

According to the article, what is the main effect of Thimes color-blind
casting?
A. Her work has received acclaim and success
B. She has been lauded by critics

Take your S AT prep to the next level. Visit www.learnerator.com


129

130

C. American television is more diverse than ever before
D. Stereotypical depictions of the GLBT community have lessened

Now, pay close attention to the word main effect, because while A is true, it clearly
isnt the focus of the article. B is also true -- in fact, it is nearly identical to A -- but it
also isnt the focus of the article. C is clearly true (the article states she has led to a
minor revolution and she has been credited with -- i.e. she has caused -- greater
representation of minority actors on television), so this is clearly the best answer. D
is tricky, because while Thimes has been credited with positively representing the
gay community, this does not mean that other shows have suddenly stopped
utilizing reductive stereotypes. An increase in positive representation does NOT
equal a decrease in negative representation, so be careful of false effects like this
and take care to pay attention to the underlying structure of the article.

Compare and Contrast
Many articles will utilize comparison and contrast in order to effectively convey a
point. Compare and contrast can be quite subtle; while signal words like worst,
best, like, dissimilar, and others can set up parallels, authors can signal a
compare and contrast structure by using disparate examples that ask the reader to
think about the elements that are implicitly being compared. Take a look at this
article from the New York Times and note the multiple instances of compare and
contrast:

Teenagers might not be needing their social networks in the way that
you or I as adults might really need our social networks, Emily White,
author of the book Lonely: Learning to Live With Solitude, told Op-
Talk. When she and her spouse separated, she said, I needed help in all sorts of
ways, and if that help hadnt been there I wouldve felt lonely, whereas if I was
16 and you asked me after drama class whether my social support network
mattered a lot to me, my answer might have been no.

Take your S AT prep to the next level. Visit www.learnerator.com
130

131

She also floated a potential explanation for the finding that students self-
reported loneliness dropped even as social network isolation increased.
Environmentalists, she said, sometimes use the term environmental amnesia.
What it means is that every generation is born into a more degraded
ecosystem, she explained, and they take that degraded ecosystem as their
baseline, and they adjust to it. Maybe teenagers today have a sort of social
amnesia Im wondering if these high-school students have
had weaker support networks their whole lives, and so theyre reporting less
loneliness because thats what theyre used to.

The first paragraph contains a fairly obvious type of comparison and contrast;
teenagers and adults clearly process loneliness in different ways and rely on social
networks in different amounts at different points in their lives. But its the second
paragraph that interestingly and implicitly compares teenagers and adults. The
word weaker should have you thinking Weaker than WHAT? and striving to
answer that question -- what are the elements being compared here? Relational
words like this are always a signal that SOMETHING is being compared and
contrasted. The degraded ecosystem should also be a clue -- clearly, this
researcher is highlighting a difference in the Social ecosystem a teenager in 2014
and a teenager in, say, 1994. Understanding that this text has a cause and effect
structure will help you to answer questions about it with greater accuracy.

How does Emily White explain the fact that teenagers face increased
social isolation yet report less loneliness?
A. She argues that there is no connection between social isolation and
loneliness
B. She gives the example of separating from her spouse to demonstrate that
teens dont feel loneliness the way adults do
C. She claims students do not need social networks to get over loneliness
D. She uses a cross-disciplinary example to demonstrate that teens dont
understand loneliness and support the way older generations do.
Take your S AT prep to the next level. Visit www.learnerator.com
131

132


In essence, this question asks you to figure out WHY this argument was structured
this way -- why did White first lead with her example about being 16, and then go
into an example? What, in essence, is she trying to establish by presenting her
argument in this fashion?

A clearly cannot be correct as White is obviously not claiming that there is no
connection between social isolation and loneliness; in fact, she strives to explain
their relationship by giving the example of the degraded ecosystem. If there were no
relationship between the two, why would White try to explain it? Clearly, A is
incorrect.
B could be plausible, but read carefully -- she didnt say teens dont feel lonely like
adults do. She said teens dont need social networks like adults do. Those are two
separate issues, according to the article.

C is also incorrect -- there is no support for this in the article. She never once says
that teens do not need social networks to get over loneliness, only that they view
social networks differently than adults do.

D is the best answer by process of elimination, but also because it is true -- she DOES
use a cross-disciplinary example that illustrates that the older generation has a
better ecosystem in terms of relationships, loneliness, and social networks, which
helps to explain why teens view these differently from adults and why their
loneliness has dropped as their isolation has increased. The way those two factors
interact is different today (she claims) than it was twenty years ago, before the rise
of the internet.
The comparison and contrast is subtle here, and often the new SAT will rely on this
subtlety in order to potentially confuse you. Be on the lookout for words that subtly
signal relationships (weak ER, strong ER, etc.) as they should lead you to look for
and try to understand the elements that are being compared.

Take your S AT prep to the next level. Visit www.learnerator.com
132

133

Use of Quotes and Dialogue
Chronological and relational signal words are simple to spot, but not every piece of
journalism will come with a helpful string of dates, times, causes, effects, and
comparisons. To make a piece interesting and informative, writers will often
interview experts, eyewitnesses, and other people of interest who will bolster (or
contradict) whatever claims the article is making. Articles like this are certainly
more interesting than a dry string of dates and facts, but they can be confusing to
follow as different experts can have different opinions and you need to differentiate
between who is saying what, and in what context.

For example, look at the following article from the Atlantic, on the importance of
field trips:
What if those field trips actually had a proven, tangible benefit to student
learning? Thats the premise set out in a new study by Jay Greene, a professor of
education reform at the University of Arkansas.
Greene evaluated 670 students, who were divided into two groups. The first
group of students was chosen at random to see a live theater performance of
either Hamlet or A Christmas Carol. The second group either read the texts of
the plays or watched film versions.

When compared with their peers in the second group, the students who
attended live theater scored significantly higher on a vocabulary test that
incorporated language from plays, and they were also better able to answer
questions about the plot and characters, according to Greene's findings.

The live theater group also scored higher on tests that measured their
tolerance of diverse points of view and ability to detect emotions in other
people. Those gains were still measurable six weeks after students attended the
live theater performance, Greene said.

"Schools are increasingly focused on the things that weve told them to be
Take your S AT prep to the next level. Visit www.learnerator.com
133

134

focused on, namely improving math and reading test scores," Greene told me.
"Anything that isnt directly related to that doesnt attract as much of their
attention, their resources, or their time."

In metro Atlanta, field trips are being used as both a means of reinforcing
classroom instruction and providing students with new experiences.
"Its important for [students] to learn the standards and perform well on these
standardized tests," Jason Marshall, a principal at an elementary school in the
Atlanta area, told the Atlanta Journal-Constitution. "But I think the way weve
always approached it is the day they take a test is really just a snapshot of what
they learn. Were interested in them learning much more about their
community, their state, country, world and how all those things are
interconnected."

A question about text structure for an article like this may ask you what a specific
expert says, forcing you to ensure that you understand and can differentiate
between one experts opinion and anothers (or the experts opinion and the opinion
of the writer). Keep your eyes peeled for quotes and make sure that you mark down
who is saying what, as the use of multiple quotes from multiple sources can often
confuse students about who is saying what and why they are saying it. For example,
try this question:

Based on the quotes from educators in this article, how would a high-
school principal approach field trips (leaving aside concerns about
cost)?
A. There is not enough information in this article to answer this question
B. A high school principal would most likely be opposed to a field trip, as it
would distract students and take away instructional time
C. A high school principal would only support field trips to go see
Shakespearean plays

Take your S AT prep to the next level. Visit www.learnerator.com


134

135

D. A high school principal would be invested in the concept of an educational
field trip as it encourages different forms of learning.

The first thing you must do when answering this question is find the quotes from
educators so that you can make a reasonable inference about how a principal might
feel. (Yes, questions about text structure often require some inferences -- authors
generally do not spell out WHY they make certain choices in writing, leaving it to the
reader to figure it out for themselves.)

Leave A for now -- lets consider the other answer choices and whether or not they
are supported by the article before we decide whether or not there is enough
information in the article to answer the question. There is support for B in the quote
Schools are focused on the things that weve told them to be focused on anything
that isnt directly related doesnt attract as much of their attention. But if you
continue reading (and paying attention to the way this article is structured, with
quotes introducing and FRAMING an issue), then it becomes clear that this isnt the
be-all, end-all of quotes when it comes to how principals feel about field trips. Look
at the quote from the principal in the last paragraph: The day they take the test is
just a snapshot; were interested in them learning much more about their
community This doesnt sound like the perspective of a person opposed to
educational field trips, plus it goes hand in hand with the previous statements about
field trips being educationally useful. So while B is somewhat plausible if you refer
only to one quote from the article, the end quote from an actual school principal,
coupled with the statements about the positive effects of field trips, work together
to make it clear that B cannot be the answer.

C is too specific; while the research focused on plays, the principal quoted in the last
paragraph makes no distinction between the types of field trips and the specific
content knowledge that his school cares most about imparting. D is best supported
by the principals statement in the last paragraph -- if his school cares most about
supporting different types of learning, then a field trip with multiple educational
Take your S AT prep to the next level. Visit www.learnerator.com
135

136

outcomes is actually the ideal. Clearly, there was enough information in the article to
answer the question, so A cannot be correct.

The Structure of an Argument
Finally, many of the texts on the new SAT will focus on rhetoric: claims,
counterclaims, and arguments. Later articles will focus with greater specificity and
detail on these issues, but as they are closely related to structure, it is important to
be able to identify them.

Writers will often set up an argument by:
1. Establishing a claim
2. Defending that claim
3. Potentially addressing other claims / opinions
4. Demonstrating why those other opinions are incorrect

Steps 1 and 2 are essential. Steps 3 and 4 are optional and you will not always see
them. But it is essential that you learn to identify the structure of an argument and
understand how a claim is established and defended. Look at this article and try to
understand what this writer is agitating for, and whether the structure of this
argument is effective:

But if the University wants us to take its strictures seriously, the least it can do
is pretend to listen to our demands. We are being asked to fork over
increasingly high tuitions each year, to partake in the mandatory (and
expensive) dining and boarding plan for the sake of communal class bonding,
and to participate in community-building exercises that benefit no one and
alienate a significant percentage of the student body. The University would do
well to consider that it endangers the future contributions of its alumni base
with its attitude towards its students, which is one of shameless financial
wrangling and gross neglect.

Take your S AT prep to the next level. Visit www.learnerator.com
136

137

How does the structure of this argument help to create a tone of
indignation?
A. The author clearly establishes her indignation by ending the letter with a
subtle threat to the university
B. The author clearly establishes her indignation with the use of informal
language such as fork over
C. The author clearly establishes her indignation by placing certain words in
quotations to indicate disdain
D. The author clearly establishes her indignation by making a specific
complaint and enumerating the reasons for it

This is an interesting question because it doesnt ask you to evaluate the claim or
even to evaluate whether the structure helps establish the claim; instead it asks you
to assess how the structure helps to create a tone. This is a double whammy
question that gets at skills like analyzing word choice as well as analyzing structure.
Have a look at the answer choices.

A is tempting because it is partially true -- the author DOES end the letter with a
subtle threat -- but it is clear well before this threat in the last sentence that she is
unhappy. Therefore she doesnt ESTABLISH her indignation in the last sentence, just
CEMENTS it. Pay attention to distinctions like this because they could be the
difference between choosing the right answer and choosing a plausible distractor.
B is implausible; use of words like fork over makes the author sound a little bit
folksy, but doesnt necessarily establish a sense of indignation. Its not like fork
over is the exclusive phrase of people who are unhappy with a specific financial
situation.

C is interesting; its clear from the quotes that the author doesnt think much of this
so-called communal class bonding, but she doesnt spend much time critiquing it,
nor does she establish her indignation by using this phrase -- her indignation was

Take your S AT prep to the next level. Visit www.learnerator.com


137

138

established well before, in the first sentence. So C cant be correct.

D is the only obvious choice -- not only is it the most all-encompassing/general, it
also best reflects the structure of this article and argument.

There are multiple types of text structures; this is just a sampling. But if you take
away anything from this article, it should be to pay attention to the subtleties and
signal words. They are often the key to unlocking the entire piece.

Take your S AT prep to the next level. Visit www.learnerator.com


138

139

Analyzing Part-Whole Relationships


As discussed in the previous article, rhetoric relies heavily on a well-constructed
argument. Its important that you can analyze the structure of the text as a whole,
but its equally important to be able to pull out specific parts of the text and analyze
how they relate or fit in with the rest of the structure.

Think of texts like tapestries, or Jenga towers: every element needs to be balanced
and well-considered, or else the finished whole can unravel completely. There are
essential functions that each small part of the text must fulfill:

1. Logic / setup every element of the text must fit together logically and
clearly.
2. Focus every element of the text should stay on point; this is especially
important in an expository text.
3. Support every element of the text should be doing something:
introducing
a topic, supporting a claim, countering a claim, establishing evidence,
describing a feature, outlining
4. Inform Is this sentence informing you about something? A setting, a
character, a piece of research, an opinion? If it isnt, consider its purpose. It
may not belong there.

The new SAT may ask you about the function of a word within a sentence (as
discussed here), a sentence within a paragraph, or a paragraph within a larger text,
so its important to understand the function of the larger element as well as the
function of the smaller element. Here are some common types of part/whole
questions the new SAT will focus on:

Take your S AT prep to the next level. Visit www.learnerator.com


139

140

Sentence-to-Paragraph Relationships

Logic / Set-up
In order to ensure that you fully comprehend structure, the new SAT will often ask
you what a sentence is doing in the middle of a paragraph. Youll have to analyze
the surrounding context in order to correctly answer these, as not every sentence
functions in the same way within a paragraph. For example, an expository essay
may start with an introductory or topic sentence that reveals the purpose or subject
of the paragraph, while a narrative selection will kick off with a first sentence that
helps to characterize or describe someone. These bits of text are doing the hard
work of set-up, and often they set the tone for the rest of the piece. For example,
look at the first sentences of these two separate paragraphs and try to figure out
their functions and relationship to the paragraph as a whole:

Lupita didnt much care for bees, but she wasnt going to sit idly by and watch
as Bobby Valente -- by all accounts the biggest jerk in the entire seventh grade -
- smashed this one with the edge of his Understanding New Horizons! science
textbook. You cant just kill something just because you dont like it, she
informed him, edging close and blocking the bee from his reach. Bobby eyed her
strangely. But its a bee, he said. It might hurt us first.
----
(from Wikipedia) Bees may be solitary or may live in various types of
communities. The most advanced of these are eusocial colonies found among
the honey bees, bumblebees, and stingless bees. Sociality, of several different
types, is believed to have evolved separately many times within the bees. In
some species, groups of cohabiting females may be sisters, and if there is
a division of labor within the group, then they are considered semi social.

It should be pretty obvious that these two paragraphs do NOT have the same
purpose. Their tones are different, their subject matter is different, and their
organization is different. Therefore, if we break down these paragraphs into their
Take your S AT prep to the next level. Visit www.learnerator.com
140

141

component parts, we wont find that every sentence corresponds in function and
type. Therefore, beware of falling into traps like the first sentence always
introduces a topic. This may be true SOMETIMES, or for CERTAIN types of
passages, but it all depends on the surrounding context.

What is the purpose of the first sentence in the first selection?
A. To provide a thesis statement for a larger argument
B. To demonstrate Lupitas feelings about bees
C. To establish a conflict between bees and humans
D. To flesh out a character

It should be fairly clear from the selection that this selection is not making an
argument, and as such does not require a thesis statement to establish a claim. You
can get rid of answer A. While B is semi-plausible, this does not seem to be the focus
of the selection -- the sentence states that Lupita didnt care for bees, yet she
campaigned for the bees rights/safety anyway. This says more about Lupita than
about Lupitas actual feelings about bees. Keep B until you can eliminate C and D. C
is clearly incorrect -- there isnt actually a conflict here between bees and humans; if
there were, Lupita wouldnt be agitating to save the bee. D is the best answer,
because what actually matters in this paragraph (and this sentence) is the
establishment of a character who cares about the helpless and isnt afraid to stand
up to the biggest jerk in the entire seventh grade in order to defend her beliefs,
even though these beliefs might be unpopular. You get more information about
Lupita here than about Lupitas feelings about bees or the conflict between bees and
humans, which is a pretty good indicator that D is the best answer. When in doubt,
consider the answer choices in terms of the information they impart; the one that
seems like it imparts the most relevant information will probably be the right
answer.

What is the function of the first sentence in the second paragraph?
A. To introduce various specialized terms and vocabulary that will be used in
Take your S AT prep to the next level. Visit www.learnerator.com
141

142

the article
B. To lend support for an argument
C. To set up the purpose and topic of the paragraph.
D. To counter a claim

If you read the first sentence of the second article carefully, youll see that there is
no specialized vocabulary until the second sentence; here, the question-writers are
hoping youll see the word introduce, stop reading, and pick A. This is a bad
strategy. Read the whole question and the whole answer. B cant be right since there
is no preceding argument, so the first sentence cant support an argument that
doesnt exist. C is obviously true -- the second paragraph is all about the different
types of communities in which bees live, and the first sentence clearly indicates this.
D, like B, cant be correct since there is no preceding argument or claim to support
or refute.

Focus
Every text has (or should have) a focus -- in essence, a point. And every component
of the text (sentences, paragraphs, etc) should reinforce the central point -- or
introduce new points for consideration. Think of texts almost like interlocking
puzzle pieces -- every sentence should fit in some way with the one before it, so
even if a sentence is introducing a new point, it shouldnt come out of nowhere. For
example, here are two examples of excerpts, one of which keeps its focus and one of
which doesnt:

1. The books were organized alphabetically. There was a gap on the shelf
between Melville and Milton, where Mencken had been pulled out.
2. The books were organized alphabetically. Cheese is delicious.

Its not too difficult to see that retaining focus is important, or else the reader will
give up. The excerpt below contains a good example of an excerpt that shifts
focus subtly, without descending into incoherence.
Take your S AT prep to the next level. Visit www.learnerator.com
142

143


Vampires -- in all their various incarnations -- are dead. The pale, the tortured,
the lonely, the en-souled, the menacing, even the sparkling: as a romantic trope
or an object of nightmares, vampires have had their moment in the sun, and
their long dark night has finally come. They are dead, but our obsession with
the supernatural isnt. In short order, hopeful Young Adult writers and cynical
movie producers have foisted upon us every half-baked example of the liminal,
undead, or unknown: zombies, ghosts, half-demons, fallen angels, werewolves,
rockstars masquerading as college students What these varyingly successful
attempts to breathe new life into the creature story fail to understand,
however, is that it is the human monster that is the scariest, the darkest, the
most intense and ultimately the most interesting.

How does the first sentence establish the tone of the rest of the article?
(logic/set-up)
A. By dispassionately stating a fact, therefore creating an objective,
journalistic tone
B. By passionately declaiming, therefore creating a fiery tone
C. By including a pun, therefore creating a dry tone
D. By utilizing a dependent clause within the two hyphens, therefore creating
a digressive tone

This question asks you to consider the first sentence in context of the rest of the
article; you must pick out specific words and phrases that establish a tone in order
to accurately answer the question. Consider Option A. The sentence does, in some
way, include a fact -- vampires are indeed dead. But the tone in the remainder of the
article is hardly objective; the author uses a lot of strong language (foisted upon
us, half baked, varyingly successful, fail to understand,) to establish a clear
and obvious opinion. Answer A can therefore not be correct. (This is why it pays to
read the entire answer choice!) B is clearly incorrect: the author does not
passionately declaim in the first sentence, even though parts of the selection do
Take your S AT prep to the next level. Visit www.learnerator.com
143

144

indeed utilize fiery language. C is correct -- there is a pun here (vampires are dead in
the literal sense, but they are also dead in the sense that they are culturally
irrelevant), and the tone of the article is indeed dry (the quote vampires have had
their moment in the sun and the inclusion of rockstars masquerading as college
students in a list of undead/unknown creatures both indicate that the writer has a
somewhat wry sense of humor about this topic.) D is half-correct as well -- there is a
dependent clause hidden within those two hyphens -- but it does not necessarily
create a digressive tone, as the article stays on focus the whole time, demonstrating
that vampires were once culturally relevant and now no longer are, or should be.
Dont be impressed by Ds grammar-speak; this is the SATs transparent attempt to
try and trick you into choosing an answer just because it sounds like it might be
right.

Which sentence in the selection introduces a shift in focus?
A. The pale, the tortured, the lonely... because it shifts the focus of the
article back to when vampires were culturally relevant.
B. They are dead because it shifts its focus from vampires to other
creatures.
C. In short order because it shifts the focus of the article to writers and
movie producers
D. What these varyingly successful attempts because it introduces the
authors premise

This is a difficult question because it asks you both to understand what the focus of
the article is as well as how the author creates a subtle shift in focus. From reading
the paragraph, it should be clear that the focus of the article isnt necessarily going
to remain on vampires, so you need to be able to make a logical inference about
where the focus of the article will later rest.

Look at A. The second sentence of the article is still about vampires, and in fact
categorizes all these different types of vampires in order to demonstrate that yes, all
Take your S AT prep to the next level. Visit www.learnerator.com
144

145

of them are now culturally irrelevant (their long dark night has finally come), so A
cant be correct.

B reinforces the idea that vampires are dead, and introduces creatures who, like
vampires, command our attention as a culture -- perhaps signaling a shift from
vampires to other creatures. B is actually a good answer, so lets keep it in our back
pocket until we read the other answer choices. C is an example of a thought that B
introduces, so it cant be the shift, because its dependent on another sentence. D,
however, is the best answer -- it shifts away from vampires and all the other
supernatural creatures that our culture is obsessed with, and establishes a premise -
- that all of these books and films about supernatural creatures are missing a
treasure trove of literary inspiration (from humans). D indicates (correctly) that the
preceding example of vampires just serves to introduce or set up the premise -- that
the human psyche could be a darker/better inspiration for literature than made-up
creatures. Though B is a good answer, D is better.

Support
Expository texts will make claims; a good expository text will support those claims.
The SAT will test your understanding of how these supports / evidence work. For
example, it is pretty clear from the following two examples that the second sentence
supports the first by giving some sort of reason or explanation for it:

I hate animals. They smell bad.


Dont be fooled by sentences like this, though, which look like they are making a
well-supported claim:

The person who wrote the above sentence is clearly a sad, sad human being.
Animals are awesome.

While both sentences are true, the writer doesnt take the time to demonstrate the
relationship between them, leaving the reader to make the connection. But in truth,
Take your S AT prep to the next level. Visit www.learnerator.com
145

146

these are two disparate clauses that have nothing to do with one another -- so make
sure that the sentences actually do support each other.

Test your understanding of supports with the following excerpt:

Team Voldemort is not exactly what it sounds like. This group of 12 fifteen-
year-old girls from British Columbia has no interest in magic, spells, or world
domination -- though they do often enter the field with two red slashes painted
under their eyes. It symbolizes the blood of our enemies, says Katie Canuckie,
the cheerful, pigtailed tenth-grade captain of the team. Were preparing for
victory and we will win. She flashes a smile that glints with heavy orthodontia.

The award-winning girls lacrosse team, which has broken three international
records with its 15-year winning streak, was renamed in 2002 to symbolize
that we should be taken seriously, quoth Brenda Ballyhoo, the teams coach
and a former Olympic hopeful, who was behind the name change.

Look, she says, speaking frankly, people dont take girls sports seriously.
That is a documented fact. How much attention does the WNBA receive as
compared to the NBA? How about womens soccer in comparison to football, or
to the World Cup nationwide? Womens hockey as opposed to mens? Face it:
people dont care about girls sports. Well, were here to change that. Our name
might be cutesy, but were dead serious about our mission. When asked
whether she was concerned about the message that the name Team Voldemort
sends, Brenda laughed darkly. Well see whos concerned when were done
with our season.

Which sentence provides the best support for Brendas claim that
people dont take girls sports seriously?
A. Our name might be cutesy, but were dead serious about our mission.
B. Well see whos concerned when were done with our season.
Take your S AT prep to the next level. Visit www.learnerator.com
146

147

C. Thats a documented fact.
D. How much attention does the WNBA receive as compared to the NBA?

Ballyhoos assertion is that no one cares about girls sports. Claims made in A and B
about the dead serious-ness of Team Voldemort are a response to the idea that no
one cares about girls sports, NOT a refutation of it. C is a plausible choice -- but it
doesnt actually support the claim because without actual statistics or verification,
its an easy claim to make, and it doesnt need to be proven in any way. The best
answer for this question is D, because it provides an example of what Ballyhoo is
talking about -- she says that no one cares about girls sports and gives a clear
example of a type of sport in which women are underrepresented or neglected by
the public.

How does the writer of the article support the assertion that Team
Voldemort is not what it sounds like?
A. By asking the coach whether she is concerned about the message implied
by the teams name
B.By giving statistics about the teams victories and awards
C. By subtly contrasting the aims and attitudes of the team with Voldemorts
aims
D. By describing Katie Canuckies orthodontia

Often with support questions, the support for the assertion will be directly
after the claim itself (which gets back to text organization/structure and
focus -- having the support placed too far from the claim would lead to
confusion!) Look at A -- at the end of the article (far away from the actual
claim itself, which is made at the very beginning of the article), the writer
asks the coach if she is concerned about the teams name. But this question
has little to do with the fact that the team isnt what it sounds like. Giving
statistics about the teams victories and awards does paint the team
positively (in contrast to the team name, which could be construed
Take your S AT prep to the next level. Visit www.learnerator.com
147

148

negatively), but this contrast isnt strong or explicit enough to provide
support for the fact that Team Voldemort isnt what it sounds like. C is
plausible, however, as the author does contrast the attitudes of the team with
Voldemorts; the author states: This group has no interest in magic, spells,
or world domination; s/he may as well say unlike Voldemort. Describing
Katies orthodontia does somewhat subtly set up a contrast between this
innocent teenage girl and the menacing Voldemort, but again, as in B, this
contrast isnt strong or explicit enough to really be construed as support for
this claim.

Paragraph to Whole-Text Relationships
The previous questions asked about the relationship between a sentence and
a paragraph. Just as each sentence does its work within a paragraph, each
paragraph has a specific function within the text as a whole. Obviously,
however, paragraphs within literary texts will perform very different
functions than paragraphs within expository texts. Still, each paragraph
should be doing one or more of the following:

Setting up a situation or claim

Supporting OR refuting a claim

Maintaining Focus

Informing

Each function will come with signal words. Some words that might be associated
with greater support or refutations would be:

because, in support of, therefore, thus, ergo, hence, due to this, moreover, additionally,
as well, also, and, further, an example of this is

Pay attention to these words; they will most likely show up in non-fiction expository
texts, as opposed to narrative works, but even narrative works have signal words
Take your S AT prep to the next level. Visit www.learnerator.com
148

149

that indicate a paragraphs function(s). As you read below, underline or annotate
the words that seem to indicate the function of the paragraph. The following excerpt
is a narrative piece.
1. I never considered myself an immigrant, though I was with my parents on the
ten-hour Iberia flight from Casablanca to Newark that took us farther from
home than any of us had ever been before. But a five-year-old is already an
immigrant anyway, confronting confusing new rules and codes every time she
leaves the house; maybe this was why I took to America so much more quickly
than either of my parents. Or maybe I was always going to be like an
American, trying desperately to find my way home. My parents wanderlust --
not to mention the lack of economic opportunity in the Old Country -- just took
the guesswork out of it.

2. By the age of ten, my accent had melted away, my French was rusty enough
to make my aunts click their tongues at my mother whenever I attempted
transatlantic conversation, and though twice-weekly Arabic school had kept
me functionally literate, my spelling was a disaster.

3. On the other hand, I excelled at my role as a mediator between my parents
and the phalanx of bureaucrats who controlled seemingly every aspect of
American life. From parent-teacher conferences (Maman, my teacher wants to
speak with you on Tuesday. Why? What have you done?! Nothing, I swear!
Theyre speaking to everyones parents!) to doctors visits (Baba, he says I
need another shot. What, only one? Surely youre at risk for more than one
disease), I became an expert navigator of the foibles, interests, and various
idiocies of the adult world.

4. By the time I was sixteen, my parents English was, if not flawless, then at
least less fractured than before. Certainly it was sufficient to embarrass me at
every college fair we attended (naturally, my parents were obsessed with
getting me into a good college, which to them meant Harvard and Harvard
Take your S AT prep to the next level. Visit www.learnerator.com
149

150

only, watching me graduate from medical school, and then marrying me off to
another doctor who had also graduated from Harvard; naturally, I was
obsessed with piercing as much of my body as I could and studying
photography. Thus far I had not made these desires known to either of them,
but as we had learned in English class, things fall apart. If I were forced into
another AP Biology or Chemistry class, the center would not only no longer
hold, but would implode completely.)

What is the relationship between the first and the fourth paragraph of
this text?
A. The fourth paragraph provides a counterexample to the first
B. The fourth paragraph gives a thorough analysis of principles outlined in
the first
C. The fourth paragraph refutes the first paragraph
D. The fourth paragraph expands in detail on the general principles outlined
in the first paragraph

Literary or narrative texts dont often utilize the same rhetorical devices and
strategies as other non-fiction texts, which may make your job of figuring out these
questions much easier. After all, its pretty clear that this writer isnt trying to
persuade the reader, but instead to tell a story. Therefore, words like thorough
analysis, counterexample, and refute wont necessarily apply to works like this.
You can automatically scratch A (there is no example, let alone a counterexample, to
analyze here), B (there is no thorough analysis here) and C (this kind of narrative
wouldnt be interested in refuting itself -- the author is telling a story, not trying to
make a claim. It would be counterintuitive for the writer to try and refute
herself!)Though D may not seem immediately obvious, its certainly the best answer
-- the general principle outlined in paragraph one is the idea of immigration, of
being a foreigner in some sense, and of the narrator being an American by
disposition. By the fourth paragraph, in which the narrator is disillusioned with her
parents ideals and interested in expressing her individuality, it should be clear that
Take your S AT prep to the next level. Visit www.learnerator.com
150

151

this is just a further example of the general idea expressed in paragraph one.

Taken in context, what is the purpose of Paragraph 3?
A. Paragraph 3 provides examples that vividly illustrate the narrators point
B. Paragraph 3 further reinforces the points made in Paragraph 2
C. Paragraph 3 provides an analysis of the various ways that being an
immigrant has changed the narrator
D. Paragraph 3 provides a set-up for Paragraph 4

Look closely at these answer options and read carefully. As per Option A,
paragraph 3 certainly is full of examples, as evidenced by the multiple
parenthetical asides that humorously illustrate the authors interactions with
her parents as well as the other adults in her world. Keep answer A for now.
Look at B. B requires that you also be familiar with paragraph 2, but
Paragraph 2 simply states that the narrator no longer speaks French and
Arabic with native proficiency. This does not mean, however, that the
narrator would automatically be better equipped to deal with the adult
American world than her parents; it simply means that she no longer speaks
good French. In fact, Paragraph 3 goes in a completely different direction
from Paragraph 2 -- it gives examples about an entirely different part of the
narrators life, so it cant be taken as reinforcement of anything stated in
Paragraph 2.

C could be a good answer, but again -- this isnt an analysis so much as a list
of examples. Additionally, we dont know if this is a change in the narrators
personality -- all she has ever been is an immigrant -- so C is in fact a fairly
worthless answer to begin with.

Lastly, look at D; while its tempting to assume that all paragraphs will be
linearly linked, as they often are in expository texts, this is obviously not
always the case, especially with literary narratives. Additionally, Paragraph 3
Take your S AT prep to the next level. Visit www.learnerator.com
151

152

does not set up Paragraph 4, as 3 documents how helpful the narrator is to
her parents, and how she acted as a go-between, while Paragraph 4 discusses
the fact that she now wishes to be her own person instead of playing a pre-
ordained part. Moreover, Paragraph 4 starts off with a transition (by the
time I was 16), which indicates that theres a shift in perspective. A is the
best answer.

When answering questions like this, you must think critically about what each
paragraph strives to do, not just what each paragraph says. Pay attention to the
words that the writer uses; transition words that indicate the passing of time (as in
Paragraph 4) or a change of opinion or topic (on the other hand, in Paragraph 3)
will help you figure out whether a paragraph is setting up, expanding, extending,
refuting, or shifting focus completely.

Expository Texts
There are multiple signal words that help indicate the purpose of each paragraph.
As you may remember from the review of text structure, signal and relational words
can indicate the type of text as a whole, which will help you understand what each
paragraph is doing in service of the larger whole. For example, look at this
chronologically structured text from the Public Library of Science Blogs Network,
and figure out from the signal words what purpose each paragraph holds within the
text as a whole:

1. Within 5 years I think people will know what an exome is, because
analyzing it will be as common as a CBC or blood lipid profile is today before
visiting the doc. As costs decrease and gene discoveries increase, weve reached
a tipping point, by definition when a series of small changes or incidents
becomes significant enough to cause a larger, more important change. Until
exome becomes a household world, clever studies are illuminating pioneering
applications of the technology.

Take your S AT prep to the next level. Visit www.learnerator.com
152

153

2. The exome, the part of the genome that encodes protein, harbors 85% of
disease-causing gene variants (were not supposed to say mutation anymore,
but thats what I mean). Results from several large studies have been published
over the past 3 years, but a paper in last weeks Science Translational
Medicine from Stephen Kingsmores group at Childrens MercyKansas City
offers the most promising results yet.

3. It heralds the dawning of the new age of clinical genetics. Weve been
waiting for this to come around for 10 to 15 years, and its finally here, says
Robert Marion, MD, chief of the division of genetics at The Childrens Hospital
at Montefiore and a developmental pediatrician at the Albert Einstein College
of Medicine, about the paper (hes not part of the team). I devoured his book
Genetic-Rounds: A Doctors Encounters in the Field that Revolutionized-
Medicine.

4. Last month, the Journal of the American Medical Association published
findings of two ongoing prospective exome sequencing studies of individuals
with symptoms suggesting an inherited condition. A group
from UCLA diagnosed 213 of 814 (26%) cases that hadnt been diagnosed
clinically or with single-gene tests or panels. The 26% rose to 31% if parents
had their exomes sequenced too. The second report, from Baylor College of
Medicine, diagnosed 504 of 2000 (25.2%) patients. Both studies werent just
children.

As you may have noticed, there are multiple words here that signal that this is a
chronological text (within 5 years, over the past 3 years, heralds the dawning of a
new age, last month, etc.); knowing that this is a chronological text may help you
understand the authors purpose and therefore why she chose to structure this
essay the way she did.
What is the function of paragraph 2 within the larger text?
A. Paragraph 2 illustrates the authors larger point
Take your S AT prep to the next level. Visit www.learnerator.com
153

154

B. Paragraph 2 provides a thesis for the rest of the text
C. Paragraph 2 introduces a change in focus from Paragraph 1
D. Paragraph 2 summarizes the rest of the article

Look at option A. To rule it out, you need to know what the authors larger point is.
Because this is an expository text, there is a good chance that this thesis statement
resides somewhere in the first paragraph; sure enough, the first paragraph is home
to the authors claim that exome will soon become a household word thanks to its
importance to medical science. Therefore, we now need to see whether Paragraph 2
illustrates this claim. Paragraph 2 gives a definition for the word and highlights its
importance to medical science based on the past few studies on exomes. This does
help illustrate that exome may become a household word (there have been many
studies and apparently many medically important findings), but before we settle on
this answer, lets examine the other options.

B is clearly untrue; the thesis is found in Paragraph 1. Paragraph 2 provides
illustration for a thesis. Scratch B.
You can scratch C, too. There is no shift in focus -- its a continuation of the focus.
Nor can D be true; the second paragraph in a four-paragraph article certainly cant
summarize the rest of the article, since the article has barely begun by Paragraph 2.
Clearly, A is the best answer.

Whether or not the chronological signal words helped you assess the function of
each paragraph, its still a good idea to make sure you understand text structure as
this can potentially influence the rest of the text.

The new SAT may not necessarily word its questions so bluntly; rather than ask you
the purpose or function of each paragraph in a text, the test may ask about the
way the author creates an argument, structures his/her claim, reinforces
his/her position, builds on the argument, refutes the argument, or similar.
These are really just forms of asking you What is the purpose of this paragraph
Take your S AT prep to the next level. Visit www.learnerator.com
154

155

since it is through the organization and structure of each individual paragraph that
writers most effectively structure and word their arguments.

Take your S AT prep to the next level. Visit www.learnerator.com


155

156

Analyzing Point of View


A texts point of view is the perspective from which it is written. Tone, stance,
opinion: all of these are part and parcel of the concept of point of view. Not every
piece takes a perspective or point of view; think about articles that strive for
journalistic objectivity, whose strength is in the fact that they present ideas and
events without extraneous commentary or obvious emotion.

Perhaps in middle school you learned that every text is written from a specific
point of view (first person, second person, third person). This is an extension of
that concept, focusing on the idea that HOW a piece is written (i.e. its form -- the
perspective it takes, the point of view it espouses) will clearly influence its content.

Each point of view can be classified into three broad categories:

1. Positive
2. Negative
3. Neutral

Of course, these arent particularly nuanced designations; there are a multitude of
ways in which a point of view can be positive or negative. However,
understanding the broad category into which a point of view fits will help you
understand it on a more nuanced and detailed level.

As discussed in this article, signal words will help you understand what an author is
trying to impart. Perspective is closely related to tone, so pay attention to signal
words that illustrate how an author feels about the topic. Try to assess the following
piece, from Vox.com, based on specific signal words that demonstrate how the
author feels about the topic at hand.

Take your S AT prep to the next level. Visit www.learnerator.com
156

157

Every day seems to turn up opportunities to abuse science in new and perverse
ways, especially when it comes to health. You open a newspaper or news site,
and you read about a health claim making the rounds: a diet that will give you
the energy of a teenager, an exercise routine that will elongate your legs, a
policy that will protect Americans from scary viruses. Many of these claims
even the ones that come from the lips of the most esteemed doctors and public
officials aren't backed by any good evidence. Some even run in the opposite
direction of what the best-available evidence tells us. In the interest of the
correcting the record, we rounded up the most egregious abuses of health
science in 2014.

What are some words that signal the authors perspective on these health claims?
Well, we have the sarcastic phrases a diet that will give you the energy of a
teenager," "an exercise routine that will elongate your legs," "a policy that will
protect Americans from scary viruses. All of these are semi-sarcastic indications
that the author thinks that these claims are silly -- clearly, if you use logic, it is easy
to see that no diet can actually elongate your legs. But perhaps more obvious are the
following phrases arent backed by any good evidence run in the opposite
direction of what the best-available evidence tells us. And finally, most egregious
abuses of health science should indicate pretty clearly that what is about to follow
is, in the authors opinion, arrant nonsense. All of this is established by certain signal
words, which establish the tone, which make it pretty clear what the authors
perspective is. Broadly categorized, its obvious that the perspective is negative. But
within that negativity lie shades and nuances, as indicated in the following question:

The authors perspective in this piece is that of
A. A social reformer on a crusade
B.A medical professional dispensing advice
C.A scholar exposing bedrocks of hypocrisy
D. An observer interested in scientific integrity

Take your S AT prep to the next level. Visit www.learnerator.com
157

158

Now, it becomes quite clear from these answer choices that these answer choices
reach far beyond the negative, positive, and neutral designations. So examine
them closely. Option A is tempting -- after all, this writer is very unimpressed with
the egregious abuses of health science committed in 2014. And one could argue
that correcting these misimpressions is a type of social reform. So though this isnt
a perfect analogy, keep it for now. B is tempting too; however, there is no evidence
that this writer is a medical professional, nor is there any advice being dispensed --
only facts (presumably). Be careful of choices like these and again, pay attention to
the nuance. The third option is also tempting -- however, there isnt any hypocrisy
here, just misinformation. And again, theres no indication that this person is any
sort of scholar. Finally, D is tempting -- observer is general enough to be true
(clearly, this person did observe what s/he feels to be egregious abuses of health
science) without inserting any background or credential that isnt stated in the text
(i.e. medical professional or scholar), and the second part works as well --
correcting this misinformation is indeed a form of pursuing scientific integrity. So
between A and D, think about which is more accurate -- theres more evidence for
the pursuit of scientific integrity than for the crusade for various social reformers.

The SAT will ask you about the perspectives of multiple kinds of texts. Try this one,
from a New York Times entitled Should You Have Things:

...the loss of possessions, ones deeply associated with the self, can cause real
grief.
At The New Yorker, Allen Kurzweil writes about one such possession an
Omega Seamaster watch that had belonged to his father, who died when he
was 5. The watch fell victim, he believes, to the influence of his boarding-school
bully, Cesar Augusto Viana:
Within the week, his henchman admitted that hed hurled my watch off a
balcony on a dare. I ran down the stairs, dashed outside, and dug through knee-
deep snow until my fingers turned white and tingly. The watch never surfaced.
The loss left me more than bereft. I felt annihilated.
Take your S AT prep to the next level. Visit www.learnerator.com
158

159

Much later, after hes confronted Mr. Viana as an adult, Mr. Kurzweil realizes:
My fathers Omega turned out to be more than a talisman. It was a time
machine that had transported me back to a moment when my family was intact
and I was happy. And, he writes: When I told my wife and son that I was
banishing Cesar from our lives, they celebrated his eviction by giving me an
extravagant gift. I am wearing it on my wrist.
Experiences, good or bad, may become stories, but Mr. Kurzweils account
suggests that things may have a narrative power, too. They may help us tell the
stories of our lives, of what weve lost and what weve gained.

This is a slightly more challenging text to parse, as the author does not necessarily
offer his or her own perspective -- at least not a strongly-worded one. But if you
read carefully, paying attention to text structure (i.e. differentiating between the
quotes from outside sources and the perspective of the writer his or herself), and
highlighting important signal words, youll be able to accurately answer this
question:

The writers perspective on material objects is that
A. Attachment to material objects signals an underdeveloped sense of self
B. Attachment to material objects encourages a sense of victimhood
C. We must be careful of the power of our attachment to material objects
D. Attachment to material objects is psychologically powerful for a reason

This will be easy to answer if you pay attention to the fact that the writer is careful
not to express a strongly worded opinion; instead, s/he allows the sources and the
quotes to speak for themselves. Nowhere does the writer indicate that s/he believes
that A is correct, and while B clearly plays on the story of bullying described in the
third paragraph, this story of bullying itself was not motivated by the material
object, nor was the attachment to the object the reason for Mr. Kurzweils sense of
victimhood. C is somewhat plausible, as the author does state in the last paragraph
that material objects have a narrative power, but the author doesnt indicate that
Take your S AT prep to the next level. Visit www.learnerator.com
159

160

this is a power that should cause us to be wary or careful. Instead, D is clearly the
best answer, as it indicates that these material objects do have power and that this
power does have a reason (they may help us tell the stories of our lives.) As you
can see, perspective isnt always indicated in strongly worded rants; it is just one of
the ways that writers can frame or illustrate their ideas.

Understanding point of view requires a fair amount of reading comprehension. You
must be comfortable with analyzing word choice and text structure, with isolating
details from the text, and with citing textual evidence for the correct answer.
Additionally, you must read the text carefully in order to differentiate between the
authors perspective and that of the figures involved in the narrative. Finally, the
SAT will present a multitude of seemingly plausible choices when it asks you to
parse the particular perspective of a piece. Try assessing the perspectives found in
the following article, excerpted from The Atlantic, which presents multiple
perspectives:

And yet, on Tuesday morning, the nuns and priests who presented the findings
of [a report about American nuns] seemed hopefulfor the future of American
nuns, but also for the relationship between U.S. sisters and the Vatican, which
has been strained. The report recognizes some women's "perception of not
having enough input into pastoral decisions which affect them or about which
they have considerable experience and expertise." And the Vatican let [a nun]
fully design and execute the report, which other [nuns] appreciated: When
representatives visited her congregation, Holland said, "it was evident that
these were sisters like us, to whom we could speak openly and honestly."

Holland's reaction is particularly revealing, because she speaks for an
organization of American sisters that has long been criticized by the
Vatican. [Nuns] in the United States are represented by two organizations: The
Leadership Conference of Women Religious, or LCWR, an association of
congregation leaders that represents about 80 percent of American sisters; and
Take your S AT prep to the next level. Visit www.learnerator.com
160

161

the Council of Major Superiors of Women Religious, or CMSWR, which
represents the other 20 percent. The Council is typically characterized as more
"conservative," or traditional: Most of the women it represents wear habits, and
some live in cloisters. The LCWR, on the other hand, represents many
congregations whose members wear street clothes and work in their local
communities.

The chairwoman of the more conservative CMSWR, Mother Agnes Mary
Donovan, was very positive about the report, saying the information-gathering
process "was overwhelmingly a beautiful experience." She noted that the
average age of women religious represented by the CMSWR is 53, which is "well
below the average trend." Nearly 1,000 women are preparing to take their final
vows into CMSWR organizations, which she said was a good sign for
recruitment.

But the report warned that the less traditional lifestyles of the majority of U.S.
congregations might be a turn-off for women who are interested in religious
life. "Candidates often desire the experience of living in formative communities
and many wish to be externally recognizable as consecrated women [by
wearing habits]," it says. "This is a particular challenge in institutes whose
current lifestyle does not emphasize these aspects of religious life."

Among the women who live in those less traditional congregations, the report
will likely be met with mixed reactions. As Holland said during the press
conference, "The visitation was met by some ... with suspicion and
apprehension." Not all congregations agreed to participate in the study, which
representatives of the Vatican called "a painful disappointment." And this
report says nothing about the ongoing investigation into the activities and
teachings of the LCWR, which "often contradict or ignore magisterial
teaching," as a 2012 Church document put it.

Take your S AT prep to the next level. Visit www.learnerator.com
161

162

Although there may be hope for healing between American nuns and the
Vatican, and although there's clearly a strong will to address the issues facing
women religious, this report mostly surfaces issues, rather than resolving them.

"I'm concerned about those who may still be angry," Holland said."

Now, this piece presents multiple perspectives -- as well as a shifting timeline -- on
one report. Because this is the excerpt to a larger piece, and because this text details
the lifestyles of a particular subculture that may not be known to you, this may be a
difficult text to parse. But if you annotate the text, keeping the different players
straight, then you should be able to answer the following questions without an issue.
First, read the questions and see whose perspectives the question is asking about;
then, go back to the text and underline, star, or color-code the particular players that
the article is asking you about. This should make it easier to keep track of who is
speaking, and what their particular perspective is.

Based on the article, the majority of American nuns would take which
perspective on the results of this report?
A. Most American nuns would react with painful disappointment
B. Most American nuns would have no unified perspective on this report
C. Most American nuns would be angry about this report
D. Most American nuns would be very positive about this report

In order to answer this question, you must understand who this majority of
American nuns is. After all, there are two major groups mentioned: "[Nuns] in the
United States are represented by two organizations: The Leadership Conference of
Women Religious, or LCWR, an association of congregation leaders that represents
about 80 percent of American sisters; and the Council of Major Superiors of Women
Religious, or CMSWR, which represents the other 20 percent. The Council is typically
characterized as more "conservative," or traditional: Most of the women it represents
wear habits, and some live in cloisters. The LCWR, on the other hand, represents many
Take your S AT prep to the next level. Visit www.learnerator.com
162

163

congregations whose members wear street clothes and work in their local
communities."

Clearly, the article is asking you about that 80%, i.e. the slightly less traditional
group of sisters who wear street clothes (as opposed to habits) and work in their
local communities. This automatically eliminates any answer choice that gives the
perspective of that 20% of more traditional nuns. Now that you recognize this,
read answer A. The article does mention the words painful disappointment, but
this isnt part of the perspective of the nuns -- instead, its the reaction of the Vatican
to some congregations unwillingness to participate in this survey. Scratch A.

B claims that most nuns would have no unified perspective. If you read the article
closely, youll see that Holland, who seems to speak for that 80% of less traditional
sisters, at times is hopeful (other nuns appreciated; it was obvious these were
sisters like us and we could speak honestly and openly; the visit was met by some
with suspicion and apprehension; Im concerned about those who may still be
angry.) Most tellingly, the article states that these less traditional nuns -- i.e.,
eighty percent of nuns in America, i.e. the majority that this question is asking about
-- will have mixed reactions to this report. Taking all of these factors into
consideration, it is clearly difficult to find a unified perspective here that all
American nuns would espouse. B is clearly the correct answer.

C claims that most American nuns would be angry, but theres no evidence for that
from the text -- the text does claim that some might be angry, but this isnt the
majority of American sisters. D claims that these nuns would feel very positive about
the report, but again, there is no evidence for this in the text; it is the minority of
American nuns (that 20% of more conservative nuns) who would probably feel
very positive about the report, considering that their spokeswoman called the
experience of gathering information for the report overwhelmingly a beautiful
experience.

Take your S AT prep to the next level. Visit www.learnerator.com
163

164

This was a hard question to answer, because it asks you to understand each part of
the text as well as to rely on inference and nuance rather than the more obvious
triggers of strong language, description, and tone.

Based on the article, most American nuns take which of the following
perspectives on the Vatican?
A. They are above all devoted to the Vaticans strictures
B. They have entirely broken from the Vaticans decrees
C. They have an uneasy relationship to the Vaticans authority
D. They remain objectively neutral to the Vatican

Remember, this question asks you about most American nuns, so be sure that you
are paying attention to that 80% of nuns instead of the 20%. If you read the article
carefully, youll see that the relationship [between the two] has been strained, and
that many American sisters have been criticized by the Vatican for contradicting
and ignoring magisterial teachings. Moreover, some congregations refused to
participate in this Vatican-sponsored fact-finding mission, clearly indicating that
they have little interest in helping the Vatican. All of this helps to get rid of Option A.
Option B clearly cant be true since many of these women DID participate in this
Vatican survey, and had mixed feelings towards it -- if they have entirely broken
from the Vatican, surely they wouldnt have cared about healing their relationship
to the institution. C surely seems to be true, whereas D is clearly false. Clearly, C is
the only correct answer.

Determining point of view is about more than just figuring out if the article is
written in first, second, or third person. Signal words, adjectives, and other
descriptions can help you figure out the perspective espoused by the writer or by
one of his/her sources, but its also important to remember that you must bear in
mind text structure and organization when reading to discover perspectives, as
these will help you follow along and understand the multiple perspectives that may
be contained in a single text.
Take your S AT prep to the next level. Visit www.learnerator.com
164

165

Analyzing Text Purpose


Every text youll find on the SAT has been written for a purpose. Perhaps you
learned in middle school that all texts were written for one of three purposes:

1. To inform
2. To persuade
3. To entertain

This is true, broadly speaking, but again, it isnt a very nuanced way of thinking
about text purpose. Within to inform or to entertain lie a multitude of other,
more precise reasons why a writer may have chosen to write a text. Since sadly
most texts dont come with a disclaimer stating THIS IS MY PURPOSE, it is up to
the reader to determine the authors purpose in writing a text or including a specific
detail in said text. In fact, you will be using the same skills in parsing an authors
purpose as you did while analyzing part-whole relationships and assessing the
purpose of a particular sentence or paragraph within a larger text.

As always, signal words that demonstrate a personal connection to the text are
especially important. Additionally, pay attention to contextual information, either
included in the text of directly preceding it that may reveal the texts purpose. For
example, just the fact that this particular article is hosted on an education blog,
under a page called SAT Review, should tell you that the purpose of this article is to
help you study for the SATs. Using contextual information as well as tone, assess the
following texts purpose, paying attention to the tone as well as to any contextual
information:

We Shall Fight on the Beaches, by Winston Churchill, June 1940:
I have, myself, full confidence that if all do their duty, if nothing is neglected,
and if the best arrangements are made, as they are being made, we shall prove
ourselves once again able to defend our Island home, to ride out the storm of
Take your S AT prep to the next level. Visit www.learnerator.com
165

166

war, and to outlive the menace of tyranny, if necessary for years, if necessary
alone. At any rate, that is what we are going to try to do. That is the resolve of
His Majestys Government-every man of them. That is the will of Parliament and
the nation. The British Empire and the French Republic, linked together in their
cause and in their need, will defend to the death their native soil, aiding each
other like good comrades to the utmost of their strength. Even though large
tracts of Europe and many old and famous States have fallen or may fall into
the grip of the Gestapo and all the odious apparatus of Nazi rule, we shall not
flag or fail. We shall go on to the end, we shall fight in France, we shall fight on
the seas and oceans, we shall fight with growing confidence and growing
strength in the air, we shall defend our Island, whatever the cost may be, we
shall fight on the beaches, we shall fight on the landing grounds, we shall fight
in the fields and in the streets, we shall fight in the hills; we shall never
surrender, and even if, which I do not for a moment believe, this Island or a
large part of it were subjugated and starving, then our Empire beyond the seas,
armed and guarded by the British Fleet, would carry on the struggle, until, in
Gods good time, the New World, with all its power and might, steps forth to the
rescue and the liberation of the old.

The purpose of this speech was most likely:
A.A declaration of independence
B. The setting of a new agenda for the nation
C. An inspirational battle cry
D. The pillorying of a public enemy

From context, you should have figured out that this speech was given sometime
during WWII. Knowing this, you can easily scratch A -- this isnt a declaration of
independence so much as a call to war. B is also uninspiring -- while Churchill is
setting an agenda of war for the nation with his insistence that we shall not flag or
fail. we shall fight on the seas and oceans his passionate declamation is at odds
with the dry, corporate tone of B. C is certainly correct, as evidenced by Churchills
Take your S AT prep to the next level. Visit www.learnerator.com
166

167

obvious passion and fiery language (we shall never surrender even if this
Island were subjugated then our empire beyond the seas would carry on the
struggle) Clearly, he is attempting to inspire his people and assure them of their
victory over the Nazis. And while he clearly has no love for the Nazi regime, this
speech is less concerned with pillorying (criticizing) this public enemy than
inspiring the British population.

Every text, even the driest one, has some kind of purpose; sometimes, as with this
text, paying attention to the intended audience will also help you to understand the
purpose of a text. This speech was given at the House of Commons but likely
broadcast on the wireless, so its safe to assume that the audience was the entire
population of England. When the audience is that large, the writers purpose will
clearly be different from the his/her purpose when addressing a small, intimate
gathering.

For example, try to assess the audience and the purpose of the following excerpt,
given by First Lady Michelle Obama at the Brookings Institute in December 2014:

And then there are the issues of quality and value, which are similarly complex.
As you know, when deciding whether its worth sending their girls to school,
parents arent just asking themselves, will this be a good experience for my
daughter, theyre calculating what those school fees will mean for their familys
food budget, theyre contemplating the loss of household help that is critical to
the survival of that family. So they want to see real evidence that their
daughter is learning real, marketable skills - things like literacy, numeracy,
vocational skills that will help her provide for herself and, ultimately, her
family.

Thats the kind of bar that we need to clear as we move forward. Because in our
work to educate girls, especially adolescent girls, were often asking families to
do what seems to be in the exact opposite of their daughters and their families
Take your S AT prep to the next level. Visit www.learnerator.com
167

168

best interests. Often, were asking them to change or disregard some of their
most strongly held values and traditions.

So, yes, we need more infrastructure. We need more resources. And, yes, we
need more good laws and policies - those are absolutely the necessary building
blocks for change. But we also need buy-in from those families and those
communities. We need parents to actually believe that their daughters are as
worthy of an education as their sons, and that sending girls to school is a good
investment for their future.

And that might take some real effort on the ground to actually understand
peoples concerns, to gain their trust, to determine what resources they need to
make the sacrifice of educating their daughters.
What is First Lady Obamas purpose in giving this talk?
A. To inform the nation about a major problem facing girls and women
worldwide.
B. To criticize the mishandling of an international crisis.
C. To persuade the members of this institute to contribute funds for the
amelioration of this issue.
D. To spur greater involvement among key players poised to redress the
issue of girls neglected educations worldwide

Remember, the context of this speech is given to you: First Lady Obama is speaking
at the Brookings Institute. Even if you dont know that the Institute is a think tank
based in Washington D.C., it should be clear that this isnt a State of the Union
address aimed at the entirety of the nation, but a talk given to a limited number of
members of the American public.

Therefore you can scratch A immediately -- the First Lady is not informing the
nation but instead addressing a group of people who are likely already acquainted
with the depths of this issue. Nor does B have any basis in the text -- there is no
Take your S AT prep to the next level. Visit www.learnerator.com
168

169

criticism, nor any indication that the First Lady believes that there has been a
mishandling of the crisis. C isnt particularly convincing either -- though the First
Lady does detail some of the challenges that girls worldwide face, nowhere does she
ask for more funds to address these issues. Clearly, D is the only correct answer --
the First Lady succinctly highlights some of the issues that girls worldwide face, as
well as possible avenues to take when addressing those problems. And the fact that
she is giving this speech to a small number of people at a think tank clearly indicates
that these are the people who have some sort of influence and ways to deal with this
problem. So while this certainly falls under the broad umbrella of informing, with
a slight tinge of persuading, this speech is clearly far more nuanced than either of
those designations can adequately capture.

The SAT will also test your understanding of the purpose of literary narratives.
Obviously, a literary narratives purpose will always be, on some level, to
entertain. But when reading, try to go beyond the idea of literature as
entertainment. Delve into the context and pay attention to signal words that will
help you to understand the true purpose of the authors writing. With questions
about purpose, the answer choices are your friend -- youre not being asked to get
into the authors mind, necessarily, just to pick the most plausible out of a range of
options. In essence, the work has already been done for you; you just need to
recognize the right answer.

Try to figure out the purpose of this brief literary passage, Snow, by Julia Alvarez:

Our first year in New York we rented a small apartment with a Catholic school
nearby, taught by the Sisters of Charity, hefty women in long black gowns and
bonnets that made them look peculiar, like dolls in mourning. I liked them a lot,
especially my grandmotherly fourth grade teacher, Sister Zoe. I had a lovely
name, she said, and she had me teach the whole class how to pronounce it. Yo-
lan-da. As the only immigrant in my class, I was put in a special seat in the first
row by the window, apart from the other children so that Sister Zoe could tutor
Take your S AT prep to the next level. Visit www.learnerator.com
169

170

me without disturbing them. Slowly, she enunciated the new words I was to
repeat: laundromat, cornflakes, subway, snow.

Soon I picked up enough English to understand holocaust was in the air. Sister
Zoe explained to a wide eyed classroom what was happening in Cuba. Russian
missiles were being assembled, trained supposedly on New York City. President
Kennedy, looking worried too, was on the television at home, explaining we
might have to go to war against the Communists. At school, we had air raid
drills: an ominous bell would go off and we'd file into the hall, fall to the floor,
cover our heads with our coats, and imagine our hair falling out, the bones in
our arms going soft. At home, Mami and my sisters and I said a rosary for world
peace. I heard new vocabulary: nuclear bomb, radioactive fallout, bomb shelter.
Sister Zoe explained how it would happen. She drew a picture of a mushroom
on the blackboard and dotted a flurry of chalk marks for the dusty fallout that
would kill us all.

The months grew cold, November, December. It was dark when I got up in the
morning, frosty when I followed my breath to school. One morning as I sat at
my desk daydreaming out the window, I saw dots in the air like the ones Sister
Zoe had drawn random at first, then lots and lots. I shrieked, "Bomb! Bomb!"
Sister Zoe jerked around, her full black skirt ballooning as she hurried to my
side. A few girls began to cry.

But then Sister Zoe's shocked look faded. "Why, Yolanda dear, that's snow!" She
laughed. "Snow."
"Snow," I repeated. I looked out the window warily. All my life I had heard
about the white crystals that fell out of American skies in the winter. From my
desk I watched the fine powder dust the sidewalk and parked cars below. Each
flake was different, Sister Zoe had said, like a person, irreplaceable and
beautiful.

Take your S AT prep to the next level. Visit www.learnerator.com
170

171

What is the authors purpose in recounting this story?
A. To illustrate the hardships that new immigrants face when assimilating to
their new country
B. To provide a searing indictment of Americas Cold War policy
C. To give a poignant snapshot of the various joys and challenges of learning
to be an American
D. To demonstrate the fear that many children felt during the Cold War

Though the excerpt is brief, you can easily isolate its main subject, which is
immigration and assimilation into American society. Yet Option A is clearly
incorrect -- though immigrating to the United States is no doubt a difficult
enterprise, especially without an adequate command of English, this brief excerpt
details none of the hardships of assimilation. You may have highlighted the
holocaust was in the air line to support this answer, yet remember that any
hardships that came with being in America during the early 1960s were shared by
all Americans, not just immigrants. (We know that the story takes place in the early
1960s because of the mention of President Kennedy, who served as President from
1961 - 1963.)

B is also incorrect -- this is clearly not a polemic against American policies since no
policies are even mentioned here. The author presents the holocaust in the air as a
fact of life, not something to rail against. C seems to be the best answer, as the piece
as a whole is largely about the different aspects of learning to be an American --
everything from learning a language to experiencing snow for the first time is part
and parcel of these joys and hardships. D isnt a terrible answer, as this passage does
clearly demonstrate fear, yet considering the upbeat ending and the matter-of-fact
way in which the author discusses the fear that she and her family and classmates
felt, this is a less convincing answer than C, which more fully encompasses every
aspect of the passage.

Take your S AT prep to the next level. Visit www.learnerator.com


171

172

In learning to parse purpose, you must consider the text as a whole. What is its main
focus? What most concerns the author? Are there ideas or themes that are
consistently repeated? Who is the intended audience? All of these considerations
will help you to better understand purpose and to accurately answer questions
about an authors purpose in writing and publishing a text.

Take your S AT prep to the next level. Visit www.learnerator.com


172

173

Analyzing Claims and Counterclaims


One of the most important features of the kinds of texts youll see on the SAT (as
well as in any academic setting) is argument. This isnt to say that all texts contain
fightin words; instead, most expository and some narrative texts contain a central
thesis that they try to prove with details, explanations, and other forms of evidence
and support. The ability to parse an argument is absolutely crucial to your
understanding of the text. But understanding the thesis isnt the only aspect of
understanding the argument; you must also be able to break down an argument into
its component claims and counterclaims.

Claims:
A claim is the central argument of the text. It can also be called a thesis, a
proposition, or -- if there is only one -- simply the argument. Assess the claim of
the following brief excerpt:

One of the ways in which ordinary people can prevent gingivitis is by gargling twice
daily with a dentist-approved mouthwash.

Broadly stated, the claim of this passage is that mouthwash will prevent gingivitis.
Presumably, the rest of the passage will go on to explain why mouthwash works, in
an effort to back up its central claim. Unfortunately, because the nature of claims is
so variable -- all texts will look different, and all texts will make different types of
claims -- there is no immediately obvious signal word that will help you to ascertain
the nature of a claim. In order to figure out what a claim is, you must simply read
carefully. Does the writer provide lots of supporting details? Are the paragraphs
organized effectively, with topic sentences that introduce the central focus of each
paragraph? If so, the claim is often found in the last sentence of the first paragraph.
If youre in a pinch and cant find the claim, skimming the first and second
paragraphs of an expository text may help. However, it is not enough to simply
Take your S AT prep to the next level. Visit www.learnerator.com
173

174

identify the claim, as a sophisticated argument will also include what is called a
counterclaim.

Counterclaims:
Counterclaims provide an opposing viewpoint to the central claim. Including a
counterclaim in your writing helps you to prove that yes, your opinion is the best-
substantiated / most widely-supported one. By giving the opposition a voice and
then knocking it down, the writer demonstrates a deep competence and familiarity
with the topic -- provided that s/he can effectively address the counterclaim. Look at
the way this author structured his counterclaim:

One of the ways in which ordinary people can prevent gingivitis is by gargling twice
daily with a dentist-approved mouthwash. However, in a recent survey of American
dentists, some questioned the use of mouthwash as a tactic to prevent gingivitis.

As you can see, the second sentence indicates that the author is presenting a
counterclaim -- a refutation to the central claim that using mouthwash is an
effective tactic for preventing gingivitis. One easy way to spot a counterclaim is to
look for signal words like however, but, on the other hand, yet, in contrast,
otherwise, notwithstanding, nevertheless, on the contrary -- any words that
signal a contrast will help you find the counterclaim. In fact, if youre having a hard
time finding the actual argument in the essay, work backwards. Look for the
transitional words that may signal a counterclaim, and assess what the counterclaim
says. Then figure out the claim it is attempting to refute.

Countering the Counterclaim:
Sophisticated essays counter their counterclaims in order to illustrate how
thoroughly they have considered the topic and how effectively they can argue.
Writers include counterclaims either to knock them down, as in a persuasive essay,
or to maintain objectivity by showing the other side of the story, as in a piece of
journalism. What is the aim in this piece?
Take your S AT prep to the next level. Visit www.learnerator.com
174

175


One of the ways in which ordinary people can prevent gingivitis is by gargling twice
daily with a dentist-approved mouthwash. However, in a recent survey of American
dentists, some questioned the use of mouthwash as a tactic to prevent gingivitis. The
survey, which was released on Tuesday, apparently polled a total of three dentists,
none of whom had completed their dental studies.

Clearly, this writer used the counterclaim in order to further bolster his own
argument -- by discrediting the opposition.

Read the following passage -- a commencement address by Ursula K. LeGuin,
delivered in May of 1983 -- and determine how LeGuin establishes claims and
counterclaims. As you read, evaluate the effectiveness of the writers strategy, and
determine the purpose of setting up a claim, counterclaim, and refutation.

Intellectual tradition is male. Public speaking is done in the public tongue, the
national or tribal language; and the language of our tribe is the men's
language. Of course women learn it. We're not dumb. If you can tell Margaret
Thatcher from Ronald Reagan, or Indira Gandhi from General Somoza, by
anything they say, tell me how. This is a mans world, so it talks a mans
language. The words are all words of power. Youve come a long way, baby, but
no way is long enough. You cant even get there by selling yourself out: because
there is theirs, not yours.

Maybe weve had enough words of power and talk about the battle of life.
Maybe we need some words of weakness. Instead of saying now that I hope you
will all go forth from this ivory tower of college into the Real World and forge a
triumphant career or at least help your husband to and keep our country
strong and be a success in everything -- instead of talking about power, what if
I talked like a woman right here in public? It wont sound right. Its going to
sound terrible. What if I said what I hope for you is first, if -- only if -- you want
Take your S AT prep to the next level. Visit www.learnerator.com
175

176

kids, I hope you have them. Not hordes of them. A couple, enough. I hope theyre
beautiful. I hope you and they have enough to eat, and a place to be warm and
clean in, and friends, and work you like doing.

Well, is that what you went to college for? Is that all? What about success?

Success is somebody elses failure. Success is the American Dream we can keep
dreaming because most people in most places, including thirty million of
ourselves, live wide awake in the terrible reality of poverty. No, I do not wish
you success. I dont even want to talk about it. I want to talk about failure.

Because you are human beings you are going to meet failure. You are going to
meet disappointment, injustice, betrayal, and irreparable loss. You will find
youre weak where you thought yourself strong. Youll work for possessions and
then find they possess you. You will find yourself -- as I know you already have --
in dark places, alone, and afraid.

...So what I hope for you is that you live there not as prisoners, ashamed of
being women, consenting captives of a psychopathic social system, but as
natives. That you will be at home there, keep house there, be your own mistress,
with a room of your own. That you will do your work there, whatever youre
good at, art or science or tech or running a company or sweeping under the
beds, and when they tell you that its second-class work because a woman is
doing it, I hope you tell them to go to hell and while theyre going to give you
equal pay for equal time. I hope you live without the need to dominate, and
without the need to be dominated. I hope you are never victims, but I hope you
have no power over other people. And when you fail, and are defeated, and in
pain, and in the dark, then I hope you will remember that darkness is your
country, where you live, where no wars are fought and no wars are won, but
where the future is.

Take your S AT prep to the next level. Visit www.learnerator.com


176

177

What is LeGuins central claim?
A. That men abuse their power
B. That victimhood is a physically and emotionally untenable situation
C. That every person should strive to define themselves as they want
D. That to be human is to be doomed to fail

LeGuin makes a few claims over the course of this argument, and they are all subtly
constructed. Though she would probably not disagree with choice A -- the language
of our tribe is the mans language -- this does not appear to be her central claim;
remember, a central claim is the most important or main one that an author makes.
Though LeGuin discusses the language of the tribe she does not focus on male
abuse of power. Nor does she focus on victimhood -- she mentions it once in the last
line of her speech. C is a well-supported claim, as she devotes the majority of the
speech to talking like a woman -- refuting traditional (ie male) ideas of success by
encouraging women to do or be what they want (including mothers, their own
mistresses, doing whatever [theyre] good at. And though D seems correct -- she
does talk about failure quite extensively -- the word doomed suggests that LeGuin
sees failure as something negative, to be avoided, instead of natural and to
embraced. Clearly, C is the best answer.

Which line from the text provides the best counterclaim to LeGuins
central assertion?
A. No, I do not wish you success. I dont even want to talk about it.
B. You will find yourself in dark places, alone and afraid.
C. I hope you have no power over other people.
D. Well, is that what you went to college for? Is that all?

Remember, LeGuins central assertion is that people should define themselves how
they want, without recourse to external benchmarks about what constitutes
happiness. Were looking for an effective counter to that. A is simply an extension of
her argument, not in any way a counter of it. B is also an extension of her argument
Take your S AT prep to the next level. Visit www.learnerator.com
177

178

(that no one should be afraid of ending up in dark places.) C, too, is an extension of
her argument. Only D is another voice, providing another perspective on her
argument. D asks well, what about success? What about why you even went to this
college in the first place? D makes a counterclaim to LeGuins assertion that its just
fine to make motherhood your only ambition. This question what about success /
why did you go to college is only important to LeGuins argument because she
knocks it down immediately, saying it is irrelevant. Clearly, she only includes a
counterclaim in order to demonstrate that her argument takes the best perspective.

What is LeGuins tone in this commencement address?
A. The tone of an optimist who believes that everything will turn out the way
it should
B. The tone of a pessimist who believes that everyone is destined to fail
C. The tone of a realist who acknowledges the various possibilities and
opportunities that exist in this world
D. The tone of an idealist who places her trust in a higher power

This is a review question designed to jog your memory about strategies used to
analyze tone. It should be clear from reviewing LeGuins claim and counterclaim that
she acknowledges the realities of the world -- she talks about failure and darkness
too much to be considered an optimist. But nor is she a pessimist. Clearly, she
believes in the power of possibility as well as the strength of the human spirit. There
is no evidence that she places her trust in a higher power, which leaves only C as
the best option -- she does acknowledge that various opportunities and possibilities
exist in this world, as evidenced by her obvious hope that each woman is her own
mistress and her insistence that darkness is your country.

The structure and content of this argument is somewhat unusual. Try your hand at
another narrative text, and this time consider the author's purpose in including the
various claims and counterclaims found in the article.

Take your S AT prep to the next level. Visit www.learnerator.com
178

179

We sent you to college to learn THIS? That was the reaction that Matilda
Flanagan, 19, received from her incensed mother after she came home during
winter break of her sophomore year of college, flushed with pride and relief at
finally having chosen a major. Flanagans mother, Brunhilde, a nurse, had
hoped her daughter -- a straight A student who won a national science fair in
the eleventh grade for her rather gory project on the life cycle of a tapeworm --
would go on to major in chemical engineering or biology, something really
practical and lucrative, as Brunhilde bluntly puts it. But instead, the 19 year
old Flanagan, who describes herself as a huge nerd, had chosen to take
another path: Fandom Studies. More specifically, her concentration was on the
collective fascination with cultural artifacts of the 1990s, i.e. Buffy the
Vampire Slayer and My So Called Life.

Shes majoring in television! Brunhilde Flanagan raged during a recent
interview. I mean, why are we paying 45k a year for her to write some essays
on a TV show? She could have easily done that while living at home and
working at Walgreens! (Matilda worked 15 hours a week at Walgreens during
high school.)

The elder Flanagan isnt alone in her ire. Increasingly, the parents of students
at expensive private liberal arts colleges are throwing up their hands and
asking: Where exactly is my money going?

Bob Litchfield, a litigator from Denver, echoed Brunhilde Flanagans
frustration. Why exactly is Yale charging my son -- or more accurately, me --
over fifty thousand dollars a year so that he can study Pokemon? Please, explain
this to me, he said in a phone interview, the anger palpable in his voice.
Litchfield had hoped that his son, a 20-year-old classically trained violinist,
would follow his footsteps and enter the rarefied (not to mention
remunerative) world of corporate litigation. But Humperdinck Litchfield is far
more interested in demonstrating how Charmander represents the shattered
Take your S AT prep to the next level. Visit www.learnerator.com
179

180

dreams of a nation. Since matriculating at Yale, he has become fluent in
Japanese, and has committed the contents of every Pokemon card ever released
to memory.

This is an outrage, says Brandywine Buckley, a stay-at-home mother of
quadruplets who attend the University of Florida. Why exactly are my four
daughters studying pop music? One of them recently wrote a term paper on
what Britney Spears means for our culture. Ill tell you what she means: she
means youll never get a job like this! We wanted our daughter to become a
statistician. Why is she wasting her time?

Money is certainly a central concern for these apoplectic parents, but these
students have a quick response. In a way, this is the definition of academics,
Matilda said, animated. I mean, what could be more academic than pursuing
knowledge that is by definition completely and entirely useless in the context of
the real world? I dont see anybody questioning history nerds who choose to
spend 7 years of their lives in the library, pursuing pointless degrees in
medieval history. Why is that allowed -- heck, even celebrated -- when a study
of our own pop culture is demonized? To which her mother responded drily,
medieval historians dont exactly make any money, either.

The battle between practical parents footing impressively (some might say
frighteningly) high tuition bills and passionate students who have chosen to
pursue their own untraditional paths wont end anytime soon, as more and
more universities add Cultural Studies classes to their course offerings. It is
impossible to say whether these impractical courses of study will outlive the
current decade, as more and more graduates of cultural studies programs
graduate with serious debt and no immediately applicable job skills -- unless
you count the ability to name every single episode of Buffy the Vampire Slayer
in a single breath.

Take your S AT prep to the next level. Visit www.learnerator.com
180

181

This is an interesting piece to parse, because -- unlike the previous straightforward
text -- this article offers multiple perspectives and has a somewhat untraditional
structure with seemingly no central argument. But if you read closely, youll see
two sides of an argument emerge, with the author taking a position of her own. Its
important that you are able to apply the ability to assess claims and counterclaims
even to texts with an untraditional structure, as you will be learning to assess all
forms of informative texts is an essential skill on the SAT, in college, and later in life.

What is the central claim of the text as a whole?
A. That cultural studies are a pointless waste of time
B. That cultural studies are the purest form of academic inquiry
C. That cultural studies are a recent phenomenon that has sparked strong
feelings
D. That cultural studies are an important part of college

If you read the question closely, youll see that you are being asked to assess the
central claim of the whole text, NOT any individual claim espoused by one person.
This question is basically asking you what the text is ABOUT. While answers A and B
are opinions held by the various players in this article, neither is the point of the
text, or the central claim that the author makes. All the author is trying to do is
illustrate a recent trend, making C the best answer; there is no evidence for D.

Based on the article, what is the primary underlying motivation for
these irate parents anger?
A. Parents are concerned that cultural studies will interfere with their
childrens social skills
B. Parents are concerned that cultural studies will not adequately prepare
their children for the job market
C. Parents are concerned that cultural studies are shallow and unproductive
D. Parents are concerned that cultural studies are extremely expensive

Take your S AT prep to the next level. Visit www.learnerator.com
181

182

This question asks you to assess the claims made by the parents interviewed in the
article and evaluate the reasoning behind them. It is fairly obvious that parents
think that cultural studies are a waste of time, but though obsessive fandom may
interfere with students social skills, there is no real evidence for this in the article
or in any of the parents arguments, so you can scratch A. B is a strong contender,
especially considering that every single parent interviewed expressed hopes for
their childrens future in the job market (see: the father who wanted his son to be a
lawyer, the mother who hoped her daughter would be a statistician). And though C
is a reasonable inference, it doesnt seem to be the primary motivator for these
parents anger. D is also a reasonable contender, as parents mention money often --
but if you read very closely, its clear that they are willing to spend a lot of money on
their childrens education provided their children actually acquire usable skills in
the job market. So its not necessarily that parents are reticent about spending
money -- the argument has more to do with disappointment that they will not
recoup this investment. Clearly, B is the best choice.

How does the author subtly demonstrate her own opinion in this
article?
A. By quoting more irate parents than fandom-loving students
B. By calling Matilda Flanagan a huge nerd
C. By quoting Matilda Flanagan extensively, demonstrating an acceptance of
Matildas opinion
D. By subtly insinuating that cultural studies are impractical

Again, this really is a question about claims -- what is the author subtly claiming, and
how? A may be tempting -- it is true that the author quotes three irate parents and
only one cultural studies student. But this is not necessarily indicative of an obvious
bias; its possible that the writer did not have the same access to students as she did
to parents. Keep this answer for now, but consider the other options. B is clearly
incorrect, because Matilda calls herself a huge nerd (seemingly as a point of pride).
And C cant be true because quoting someone extensively does not imply an
Take your S AT prep to the next level. Visit www.learnerator.com
182

183

endorsement of their opinion -- if that were the case, both A and C would be correct.
But the last paragraph of the text provides evidence for D -- the author calls into
question the continued existence of these cultural studies program, claiming that
they leave students with no applicable job skills and subtly denigrating the skills
they do impart (the ability to name all the episodes of Buffy the Vampire Slayer in a
single breath). Though this is supposedly a neutral text (the author sets up multiple
claims), the inclusion of this last sentence is a not-so-subtle hint that the author also
has some feelings about this issue. When evaluating a claim and/or a bias, consider
any reason the author may have for including information. If it doesnt serve the
obvious purpose of bolstering the central claim (as the quotes from parents and
students do), then there is a very good chance that it is being included for the sole
purpose of demonstrating the authors opinion.

Claims and counterclaims are an important part of many forms of writing, whether
narrative, expository, or persuasive. If you can spot a claim when it is being made
and analyze its purpose, you are already poised for success on the SAT as well as in
any later academic context.

Take your S AT prep to the next level. Visit www.learnerator.com


183

184

Assessing Reasoning
As demonstrated in previous articles, the new SAT is very concerned with testing
your ability to understand arguments -- in structure, tone, style, and reasoning. But
it is not enough to simply identify the claims and counterclaims that an author
makes over the course of an argument. It is crucial that you are able to assess those
claims and arguments, and evaluate whether or not they are sound.
When assessing a claim, you must consider both its soundness and its validity. A
valid argument is one that follows logically (even if it isnt true). For example:

Premise:
All people can fly.
Beyonce is a person.
Therefore, Beyonce can fly.

This is a perfectly valid argument. It happens to be complete nonsense, but it follows
logically. When analyzing an authors argument, you must consider whether the
argument follows logically -- whether each premise builds logically on the next. This
is one very important part of assessing reasoning.

The SAT isnt only asking you to assess whether a text is valid, however; it also asks
you to assess whether an authors reasoning is sound -- i.e., whether the argument is
not only logical but true. You may not have deep familiarity with every topic that the
SAT Reading section will cover; often, you will be presented with persuasive or
expository texts that contain information that you will not be able to assess because
you dont have the content knowledge to do so. For readings like that, assume that
the content is actually true -- the SAT will not trip you up or try to trick you by
presenting you with false information. Therefore, as you read, assume the argument
is true (unless presented with evidence otherwise, i.e. if the piece is intended to be

Take your S AT prep to the next level. Visit www.learnerator.com


184

185

absurdist or satiric) and assess instead its logic and the flow or structure of its
premises.

Assess the following excerpts validity and soundness; as you read, it may be helpful
to annotate, underline, or number key portions of the text that make up the
argument. See if you can follow the arguments trajectory. This speech, The Perils of
Indifference, was given by Elie Weisel in 1999.

In a way, to be indifferent to... suffering is what makes the human being
inhuman. Indifference, after all, is more dangerous than anger and hatred.
Anger can at times be creative. One writes a great poem, a great symphony.
One does something special for the sake of humanity because one is angry at
the injustice that one witnesses. But indifference is never creative. Even hatred
at times may elicit a response. You fight it. You denounce it. You disarm it.

Indifference elicits no response. Indifference is not a response. Indifference is
not a beginning; it is an end. And, therefore, indifference is always the friend of
the enemy, for it benefits the aggressor -- never his victim, whose pain is
magnified when he or she feels forgotten. The political prisoner in his cell, the
hungry children, the homeless refugees -- not to respond to their plight, not to
relieve their solitude by offering them a spark of hope is to exile them from
human memory. And in denying their humanity, we betray our own.
Indifference, then, is not only a sin, it is a punishment. And this is one of the
most important lessons of this outgoing century's wide-ranging experiments in
good and evil.

What is the reasoning behind Elie Weisels claim that indifference is the
greatest evil?
A. Indifference is a sin and a punishment
B. Indifference is more dangerous than anger and hatred

Take your S AT prep to the next level. Visit www.learnerator.com


185

186

C. Indifference benefits the aggressor
D. Indifference denies the humanity of others

In order to answer this question, you must consider how Weisel structures his
argument. His central claim is that indifference is the greatest evil, but how does
he justify this claim? Read answer A closely -- while he does say that indifference is a
sin and a punishment, this does not reveal the reasoning behind his argument. In
fact, this answer option simply repeats or extends his claim without actually
providing the reasoning behind it. The same is true for answer B and partially for
answer C -- none actually explain Weisels reasoning, or the way he configures the
argument behind the central premise. Only answer D logically
explains why indifference is the greatest evil. When you think about the way
reasoning works, ask yourself: does this explain WHY the author has made this
central claim? If you cant answer that question, then the answer option is probably
incorrect.

One way to tackle this problem is to map out the argument itself and see which
pieces of it lead logically to others.

1. In a way, to be indifferent to... suffering is what makes the human being inhuman.

This is the central claim. The rest of the argument should prove this, and answer the
question why is this claim true?

2. Indifference, after all, is more dangerous than anger and hatred.

This does not answer the question of why is this claim true; it simply extends the
premise that indifference to suffering is evil.

3. Anger can at times be creative. One writes a great poem, a great symphony. One
does something special for the sake of humanity because one is angry at the injustice
Take your S AT prep to the next level. Visit www.learnerator.com
186

187

that one witnesses. But indifference is never creative. Even hatred at times may elicit a
response. You fight it. You denounce it. You disarm it.

This begins to answer the question of why: indifference is the greatest evil
because, unlike anger or hatred, it is not creative.

4. Indifference elicits no response. Indifference is not a response. Indifference is not a
beginning; it is an end. And, therefore, indifference is always the friend of the enemy,
for it benefits the aggressor -- never his victim, whose pain is magnified when he or she
feels forgotten. The political prisoner in his cell, the hungry children, the homeless
refugees -- not to respond to their plight, not to relieve their solitude by offering them
a spark of hope is to exile them from human memory. And in denying their humanity,
we betray our own.

Again, this reinforces the premise by answering the question of why: why is
indifference the greatest evil? Because it isnt creative and has no response, and
ignores and forgets the people who need the most help. In so doing it betrays the
humanity we all share.

5. Indifference, then, is not only a sin, it is a punishment. And this is one of the most
important lessons of this outgoing century's wide-ranging experiments in good and
evil.

This extends the argument -- BECAUSE indifference is the greatest evil (the then
indicates the idea of because), it is both a sin and a punishment. This subtly moves
on from the central premise and extends it so that it is even more damning. When
youre stuck, use this numbering / why strategy, and see if it helps you better assess
and evaluate the reasoning behind an argument.

Take your S AT prep to the next level. Visit www.learnerator.com


187

188

Try this strategy one more time with the following text from the Atlantic, entitled
North Korea Is Not Funny. As you read, ask yourself:

What is the authors claim?
What is the reasoning behind it?
Is the reasoning behind it logical, valid, and sound?
If you're not sure how to do this, number the claims and make sure that
all of them answer the question of why.

This film (The Interview, about the assassination of Kim Jong Un) is not an act
of courage. It is not a stand against totalitarianism, concentration camps, mass
starvation, or state-sponsored terror. It is, based on what we know of the movie
so far, simply a comedy, made by a group of talented actors, writers, and
directors, and intended, like most comedies, to make money and earn laughs.
The movie would perhaps have been better off with a fictitious dictator and
regime; instead, it appears to serve up the latest in a long line of cheap and
sometimes racism-tinged jokes, stretching from Team America: World Police to
ongoing sketches on Saturday Night Live.

Humor can be a powerful tool for surviving in a closed society, and lampooning
dictators can lend latent popular movements the confidence they need to
challenge their oppressors. In Libya, dissidents heaped mockery on the Qaddafi
family in the early stages of their Arab Spring revolution. In the Soviet Union,
activists like Natan Sharansky employed dark humor to weather persecution
and labor camps. In a confrontation with evil, Sharansky once observed, it is
important to take yourself and everything thats happening very seriously, to
understand that you are part of a very important historical process, and thats
why everything [that] youll say and do has tremendous importance for the
future. Nevertheless, he added, its very important not to take anything
seriously, to be able to laugh at everything, at the absurdity of this regime, at
this KGB prison, and even at yourself.
Take your S AT prep to the next level. Visit www.learnerator.com
188

189


Yes, North Korea has long been ruled by an eccentric dynasty of portly dictators
with bad haircuts. Yes, the propaganda the regime regularly trumpets to shore
up its cult of personality is largely ridiculous. And yes, we on the outside know
better, and can take comfort in pointing fingers and chuckling at the regimes
foibles.

But it takes no valor and costs precious little to joke about these things safely
oceans away from North Koreas reach. When a North Korean inmate in a
political prison camp or a closely monitored Pyongyang apparatchik pokes fun
at Kim Jong Un and the system he representsthat is an act of audacity. It very
literally can cost the persons life, and those of his or her family members. To
pretend that punch lines from afar, even in the face of hollow North Korean
threats, are righteous acts is nonsense.

Whats more, crowding the North Korea story with anecdotes of nutty
behavior and amusing delusions may ironically benefit those in charge in
Pyongyang. It serves to buffer and obscure the sheer evil of a regime that
enslaves children and sentences entire families to death for crimes of thought,
while building ski resorts, dolphinariums, and other luxury escapes for elites
with funds that could feed its malnourished people for several years. How many
people would have watched The Interview and concluded that they should do
something to help change this odious regime and bring about human rights for
North Koreans?

In Charlie Chaplins 1964 autobiography, the star discussed the backlash that
he faced from Hollywood and the German and British governments when plans
for The Great Dictators release were announced. He moved forward with the
project despite these concerns, but years later suggested that he regretted that
decision: Had I known of the actual horrors of the German concentration

Take your S AT prep to the next level. Visit www.learnerator.com


189

190

camps, I could not have made The Great Dictator; I could not have made fun of
the homicidal insanity of the Nazis.

Kim Jong Un and his barons are also representative of a singularly horrific
system, one in which the scale and scope of suffering among 25 million North
Koreans does not, as a recent United Nations inquiry noted, have any parallel
in the contemporary world. North Korea is not funny. It is hard to imagine a
comparable comedy emerging about quirky Islamic State slavers or amusing
and complicated genocidaires in the Central African Republic. The suffering
in question is happening now, as I write. The day will soon come when North
Koreans are finally free, and liberated concentration camp survivors will have
to learn that the world was more interested in the oddities of the oppressors
than the torment of the oppressed.

What is the central claim of this text?
A. That films can never adequately capture the horror of mans evils
B. That, like Charlie Chaplin, the makers of this film will regret their decision
to parody an evil regime
C. That comedy in the face of evil is a bold and audacious response
D. That The Interview is a poor vehicle for adequately expressing the evils of
the North Korean regime

Consider the title of this text: North Korea Isnt Funny. From there it should be fairly
simple to isolate the central claim. Answer A is too much of a stretch; in fact, the
writer says The movie would perhaps have been better off with a fictitious dictator
and regime, suggesting that there is a certain value -- even if it is just entertainment -
- to depicting evil on film. Nor is there evidence for B. C is certainly an assertion, as
evidenced by this quote: Humor can be a powerful tool for surviving in a closed
society, and lampooning dictators can lend latent popular movements the confidence
they need to challenge their oppressors. But if you follow the structure of the
argument, that is a counterclaim, not a central claim, and it does not apply to the
Take your S AT prep to the next level. Visit www.learnerator.com
190

191

main premise, which is D -- that The Interview is a poor vehicle for this kind of
commentary, and that North Korea is not funny, certainly not when those of us
poking fun at it live thousands of miles away from this regime. As the writer says:
But it takes no valor and costs precious little to joke about these things safely oceans
away from North Koreas reach. When a North Korean inmate in a political prison
camp or a closely monitored Pyongyang apparatchik pokes fun at Kim Jong Un and the
system he representsthat is an act of audacity. It very literally can cost the persons
life, and those of his or her family members. To pretend that punch lines from afar,
even in the face of hollow North Korean threats, are righteous acts is nonsense.

What is the writers reasoning behind the claim that The Interview is a
poor vehicle through which to express outrage at North Koreas various
abuses of human rights?
A. The film is slightly racist, and thus inappropriate
B. The film is not funny, and thus it is inappropriate
C. The film is a comedy and not a meaningful stand
D. The film will be overly sympathetic to the North Korean regime

This question asks you to trace the writers reasoning. If the central claim is that the
film is inappropriate, you must ask yourself as you read each answer choice Does
this answer the question of WHY the film is an inappropriate vehicle? A is partially
true -- the writer does say that the portrayal is cheap and sometimes racism-
tinged. Keep it for now. B is not supported by the text; the writer has not seen the
film and makes no mention of its quality, just its subject matter. C is supported
repeatedly throughout the text, as the writer continually makes clear that making a
comedy about North Korea is deeply insensitive (The day will soon come when
North Koreans are finally free, and liberated concentration camp survivors will have to
learn that the world was more interested in the oddities of the oppressors than the
torment of the oppressed) as well as lacking in any actual meaning: (But it takes no
valor and costs precious little to joke about these things safely oceans away from
North Koreas reach. Whats more, crowding the North Korea story with anecdotes
Take your S AT prep to the next level. Visit www.learnerator.com
191

192

of nutty behavior and amusing delusions may ironically benefit those in charge in
Pyongyang. It serves to buffer and obscure the sheer evil of [the] regime. How many
people would have watched The Interview and concluded that they should do
something to help change this odious regime and bring about human rights for North
Koreans?) All of these arguments demonstrate the writers line of reasoning when
s/he claims that the film is inappropriate. So while A may be partially true, it is in no
way the best or most widely-supported answer.

As you can see from this article, in order to assess reasoning, you must also be able
to cite evidence accurately to justify your assessment; the easiest and best way to do
this is by asking yourself why? or "Does this support the central claim?" You will
learn more about this skill in the next article.

Take your S AT prep to the next level. Visit www.learnerator.com


192

193

Analyzing Evidence

Analyzing Evidence
As discussed in the last article, part of being able to assess an authors argument is
the ability to analyze the evidence that s/he brings to justify the central claims of the
argument s/he makes. You must be able point to the claim, prove how the author
uses reasoning to support it, and analyze whether or not this reasoning (i.e. the
evidence presented) is convincing.

As always, you must be able to ask yourself why? Why does the author make this
claim? How does the author justify this claim? If pieces of evidence thoroughly and
convincingly answer the central question of why, then they work; if not, then they
dont.

Try with this scathing review of Love Actually from the Atlantic. As you read, assess
what the central claim is, and how the writer supports the claim. Are his reasoning
and his evidence convincing? (Can you think of a counter to any of them?)

...So take the film on its own titular terms. What does Love Actually tell us about
love, actually? Well, I think it tells us a number of things, most of them wrong
and a few of them appalling. Now, anyone who goes to the cineplex with any
regularity knows that the last decade has seen more than its share of bad
romantic comedies. But Love Actually is exceptional in that it is not merely, like
so many other entries in the genre, unromantic. Rather, it is emphatically,
almost shockingly, anti-romantic.

...The bulk of the film offers up at least three disturbing lessons about love.
First, that love is overwhelmingly a product of physical attraction and requires
virtually no verbal communication or intellectual/emotional affinity of any
kind. Second, that the principal barrier to consummating a relationship is
Take your S AT prep to the next level. Visit www.learnerator.com
193

194

mustering the nerve to say I love youpreferably with some grand gesture
and that once you manage that, youre basically on the fast track to nuptial
bliss. And third, that any actual obstacle to romantic fulfillment, however
surmountable, is not worth the effort it would require to overcome.

Begin with the elevation of physical attraction over any of the other factors
typically associated with romantic compatibility: similar likes and dislikes,
overlapping senses of humor, shared values, what have you. [One character]
falls in love with [another] the first time he speaks with herGet a grip, he
chides himself moments afterwardwhen essentially the only thing he knows
about her is that she accidentally uses profanity a lot. (Charming? Sure.
Evidence of a soul mate? Unlikely.) [Two characters] meanwhile, fall in love
despite not sharing a word of language in common. Moreover, the movie
telegraphs very clearly that the moment when Firth really falls for Moniz is
when he watches her strip down to her underwear.

...The pattern is repeated throughout the film. [One character] is in love with a
beautiful, popular girl at school with whom hes never spoken. [One character]
recognizes that a ray of sunshine may enter his entombed love life the instant
he meets a mom who looks exactly like (i.e., is played by) Claudia Schiffer. We
can assume, I suppose, that [two characters] have had some conversations
they do work in the same office, after allbut the film doesnt bother to show
them having any. All we know about him is that she thinks hes too good for
her and, later, that he has washboard abs. The storyline regarding [one
character]s quest for American babes is played as a gag, of course: dorky
British guy is convinced that his accent will prove irresistible to super-hotties in
Wisconsinand, lo and behold, hes right! But the plotlines comically
exaggerated infatuation with physical attraction is actually not very far out of
keeping with the rest of the film.

Take your S AT prep to the next level. Visit www.learnerator.com


194

195

Creepiest of all is the storyline involving [two characters]. Why is he so
desperately in love with his best friends bride? Well, its not the result of any
conversation theyve had or experience theyve shared, because the movie is at
pains to note that hes barely spoken to her and he goes out of his way to avoid
her company. Indeed, the video tribute to her bridal radiance that he records at
her wedding makes pretty clear what it is about her that so captivates him.
(Hint: not her mind.) And he, too, like [another character] ultimately suggests
that the only way he will ever get over this love of his life is by hooking up with
a supermodel. Im barely scratching the surface of whats wrong with this
subplotthe movies worstwhich somehow manages to present the idea that
its romantic to go behind a friends back to ostentatiously declare your
everlasting love for his wife. But lets not get off track.

This is the point at which defenders of the film will reply, reasonably enough: So
what? In movies beautiful people always fall in love with other beautiful
people! Whats wrong with love at first sight, anyway? Which are both fair
responses, as far as they go. But Love Actually is a considerable outlier among
romantic comedies in its rigorous conviction not only that people fall in love
without really knowing one another, but that they dont even need to learn
anything about each other to confirm their initial attraction.

This is not some abstruse or esoteric component of high-end cinema. The core
of most romantic comediesthe core, for that matter, of most romantic
comedies written and/or directed by Richard Curtisis one form or another of
mutual exploration between potential lovers. Some movies do it well and some
do it poorly, but almost all at least make an effort to do it. The protagonists
bicker their way into love (27 Dresses, Sweet Home Alabama, Something's
Gotta Give...). The guy gradually persuades the gal that hes worthy, or vice
versa (Groundhog Day, Knocked Up, Working Girl ...). One helps the other
overcome a foolish obsession with a Mr. (or Mrs.) Wrong (The Wedding Singer,
Forgetting Sarah Marshall, While You Were Sleeping ...). The free spirit teaches
Take your S AT prep to the next level. Visit www.learnerator.com
195

196

the control freak to let go and embrace life (Along Came Polly, Pretty Woman,
The Ugly Truth ...). Opposites discover that they are attracted (Two Weeks
Notice, Notting Hill, Maid in Manhattan ...). Etc., etc. My point is in no way to
suggest that these are all good movies. (Theyre emphatically not.) Rather it is
to point out just how far outside the ordinary it is that none of Love Actuallys
fated couples spends any meaningful time getting to know one another at all.

What is the authors main source of support for his central claim that
the film Love Actually is anti-romantic?
A. Unlike other films, it doesnt accurately portray relationships
B. The film offers up three disturbing lessons about love
C. The film is about love at first sight, which is an unrealistic concept
D. The film emphasizes physical attraction over everything else

The author is clearly building a case against Love Actually. While he does compare it
to other films, he does not suggest that these other films are accurate, which leaves
A in the dust. B is true, and a direct quote from the text, but it is not support for the
idea that Love Actually is anti-romantic; its more an extension of the argument as it
does not adequately answer the question of why. It just makes a claim but doesnt
back it up. C is also true-ish, but it isnt the main source from where the author
draws his argument. Instead, the author spends nearly the entirety of this article
proving that D, the film emphasizes physical attraction over everything else, and this
is the real reason that the film is anti-romantic.

How does the author use evidence to support his argument?
A. The author gives multiple examples from the film that all function as
support for his central claim
B. The author compares Love Actually to other films to support his central
claim
C. The author effectively addresses a counterclaim in order to support his

Take your S AT prep to the next level. Visit www.learnerator.com


196

197

argument
D. All of the above

This question doesnt ask you what the authors evidence IS, but rather asks you to
evaluate his strategy. A is clearly true; the majority of the article is given over to
discussing the various characters and their relationships. The author does compare
the film to others in its genre, all to demonstrate that its conception of relationships
is far outside the ordinary. The author does address the counterclaim (beautiful
people fall in love all the time) by saying that Love Actuallys idea of love at first
sight does not fit any sort of accepted love at first sight model because the
characters do not even speak to each other once they have fallen in love. Clearly, D is
the best answer -- the author uses multiple tactics in order to demonstrate his main
point.

Try once again to analyze the evidence that an author uses to structure and support
the following argument. As you read, make sure you are paying attention to the
central claim as well as to HOW the author uses evidence to prove his point, as well
as the actual content of the evidence, and whether or not it effectively supports the
central claim. This article is also from The Atlantic.

One of the most celebrated pieces of advice to writers is Write what you know.
Unfortunately, it shows.

The demographics of published writers in the West are largely homogeneous,
and as a result, our literature is also largely homogeneous. Growing up, for
example, my heroes were Atreju, Frodo, and Paul Atreides. All I ever really
wanted to do was go on adventures like them. I readily identified with them,
and their trials became my scripture: the loss of Artax, the recovery at
Lothlrien, the knife fight with Feyd-Rautha. Despite a liberal upbringing and
an education at a womens college, it didnt occur to me that my identification
with male heroes had damaged me in any waythat is, until I became a writer,
Take your S AT prep to the next level. Visit www.learnerator.com
197

198

and found myself weirdly reluctant to write a woman hero. This wasnt an
accident.

As Vanessa Veselka wrote in The American Reader, there is a profound relative
lack of female road narratives in the Western literary tradition. This absence
hurt her in much more concrete ways. When recounting her years as a teenage
hitchhiker, Veselka writes, my survival depended on other peoples ability to
envision a possible future for me[but] there was no cultural narrative for [us]
beyond rape and death. Male hitchhikers had Jack Kerouac, Walt Whitman,
and dozens of others. Veselka had bodies in dumpsters on the six oclock news.

Meanwhile, in Joseph Campbells The Hero With a Thousand Faces, a work that
compares mythologies from cultures around the globe, the hero pretty much
just has one face: that of a white (or white-washed) man. Women are usually
guiding spirits or goddesses encountered along the way, not the heroes
themselves. This has troubling implications when we view writing stories as an
act of creation: not just of a narrative, but of the society in which we live, and
the possibilities prescribed for the people who live in it. Literature is our
collective creation myth.

...Western writers still make up the majority of published English language
authors, and English is one of the global lingua franca. Western literature
already has extraordinary women heroes created by extraordinary writers:
Toni Morrisons Sethe, Ursula Le Guins Tenar, Chimamanda Ngozi Adichies
Ifemelu. But theyre a tiny proportion of the whole. We need more. Writing
characters different from usfor all creators, in all directionsis integral to
creating a literature in which all phenotypes are heroic, and therefore, all are
humanized.

I wont go so far as to say that creators have a responsibility to do so. But I do
submit that our increasingly global society offers an unprecedented
Take your S AT prep to the next level. Visit www.learnerator.com
198

199

opportunity for all creators to write what we dont know. The defining heroic
journey of the 20th century was to conquer evil: the Nothing, Sauron, the
Harkonnens. But the defining heroic journey of the 21st century will be to
reconcile the Other with the Self.

What is the writers central premise in this text?
A. That the lack of diversity in Western literature has concrete negative
effects on our society
B. That writers have a responsibility to the reader to create more diverse
characters
C. That the advice write what you know is mistaken and should be
rethought
D. That identification with male heroes is damaging

The writer does spend the bulk of the text lamenting the lack of diversity in Western
literature, first citing her own experiences reading exclusively male protagonists,
then citing other cases of women who have found themselves stymied and
frustrated by the lack of protagonists and narratives that accurately represent the
vast spectrum of humanity. Keep A for now. B is clearly wrong, as the writer says in
the last paragraph I wont go so far as to say that creators have a responsibility [to
write characters different from themselves]. She does start off with C -- that write
what you know is popular advice -- but she doesnt advocate letting go of this
advice entirely. And while she does claim that her identification with male heroes
was personally damaging, this is not the central claim of her article -- rather, it is a
piece of evidence that she uses to bolster her claim. (It answers the question
why?)

Which is the most effective piece of evidence that the writer uses, and
why?
A. Her assertion that her own identification with male heroes was damaging,
because it adds a personal touch to this persuasive text that helps the reader
Take your S AT prep to the next level. Visit www.learnerator.com
199

200

connect better with its content
B. Her example of Vanessa Veselkas lack of female role models in the road
narrative, because it graphically emphasizes the gulf between what is viewed
as possible for men and what is viewed as possible for women
C. Her example of the different evils our literature has had to conquer,
because they give a real face to the problem that our literature and our
society face
D. Her assertion that literature is our collective creation myth, because this
illustrates just how deep this problem runs.

Remember, the writer uses evidence to support her central claim that the lack of
diversity in modern literature is deeply problematic. To be effective, these pieces of
evidence must answer the question of WHY this assertion is true. Read A; while this
is certainly evidence that she used, and while it does add a personal touch, consider
whether it indeed helps the reader connect better with the content. Does this
accurately demonstrate the extent or seriousness of the damage done to our
society? Keep A for now, as it does reflect a form of damage, but consider whether
its the best answer. B is certainly true; it is extremely damaging for a young woman
to see only two (very dark) options for herself as she embarks on a road trip. The
fact that women are missing entirely from the road narrative certainly constitutes a
compelling piece of evidence from the assertion that the lack of diversity in
literature is indeed damaging to our society. C is a clever answer that in fact has
nothing to do with the argument; she gives an example of these evils (the Nothing,
Sauron, etc) only to contrast these villains with the villains that we find within
ourselves (the Self and the Other). And D is also a clever answer, but it doesnt
prove her point since there is no tangible evidence for it; it is just another piece or
extension of her central argument. Between A and B, B is the better answer; while
both provide examples of the kind of damage caused by the lack of diversity in
literature, Bs example is far more graphic, pervasive, and violent, and therefore
more convincing as an example of "damage."

Take your S AT prep to the next level. Visit www.learnerator.com
200

201

As you read and analyze evidence and claims, make sure that you understand the
distinction between evidence / support and the actual argument itself. The new SAT
will attempt to distract you by offering you answer choices about pieces of
evidence that are simply extensions of the authors argument. Watch out for this,
and make sure you are always asking yourself does this answer the question of
why?

Take your S AT prep to the next level. Visit www.learnerator.com


201

202







Part III: Synthesis (Reading)

Take your S AT prep to the next level. Visit www.learnerator.com


202

203

Take your S AT prep to the next level. Visit www.learnerator.com


203

204

Of the various skills required for success on the SAT (as well as other academic
pursuits), one of the most important (and sometimes the most difficult for students)
is synthesis, or the ability to understand and integrate the information found in
multiple texts. A truly high-level student can extract relevant information from
multiple texts and weave together a coherent argument that takes into account
these various sources of information.

Synthesis is just a combination of the skills you have reinforced over the course of
the previous articles, but in the context of multiple texts. This skill requires you to
read carefully, think critically, make inferences, asses reasoning, analyze evidence,
and form justifiable opinions -- on basis of more than one passage. Sometimes the
texts will present two sides of the same argument -- in some ways making your job
of analysis easier -- and sometimes they will not. Try your hand at analyzing the
following two texts; as you read, pay careful attention to similarities and differences
between the two. It may be helpful to make a chart of the various ways in which
these texts discuss the same main ideas:

Text One, from The University of Florida Institute of Food and Agricultural
Sciences

A species is endangered when it is threatened with extinction. Since time began,
countless species have gone extinct from natural processes. The extinction of
dinosaurs is the best known example.

Why Save Endangered Species?
If extinction is a natural process, why should we make an effort to save
endangered species? Because we can no longer attribute the accelerating
extinction of plants and animals to natural causes. Today most species of plants
and animals become extinct because of habitat destruction (loss of living space
to development or pollution), introduction of non-native organisms, and direct
killing (over-harvesting, poisoning)....
Take your S AT prep to the next level. Visit www.learnerator.com
204

205


Our understanding of the value of endangered species to humans has increased
together with the recognition that human activities cause extinction. In
general, benefits of species can be classified as ecological, economic, and social.
Different combinations of benefits occur for any particular species, and some
species are obviously more "valuable" than others. More important than
knowing why a particular species is valuable is understanding why so many
kinds of plants and animals are valuable.

Biological Diversity
The assemblage of populations of plants and animals in an area is termed its
"biological diversity." The term biological diversity is often used
interchangeably (sometimes confusingly) with two other terms, "genetic
diversity" and "ecological diversity." Genetic diversity (amount of genetic
variability among individuals of the same species) and ecological diversity
(number and relative abundance of species) are both components of biological
diversity.

Genetic diversity is directly related to a species' ability to survive environmental
change. For example, plants and animals can be characterized by their ability
to exist under different climatic (moisture and temperature) conditions.

However, within different species there is a certain amount of variability in the
tolerance of individuals to climatic conditions. The ability of different species to
cope with environmental--in this example climatic-- change depends on this
variability. When genetic variability is reduced, as with the Florida panther, the
risk of extinction increases.

The loss of a single species can set off a chain reaction affecting many other
species. The total impact of extinction is not always apparent, and is difficult to

Take your S AT prep to the next level. Visit www.learnerator.com


205

206

predict, but it is clear that conserving biological diversity is essential for
maintaining intact ecosystems.

Benefits of Biological Diversity
How does maintaining biological diversity benefit humanity? It only takes a
moment to realize that throughout history plants and animals have provided
humans with food, clothing, energy, medicines, and structural materials. Today,
solutions to problems in agricultural production in tropical countries, reliance
on petrochemicals, and the cures for cancers may lie in organisms not yet
discovered. It would be a shame to lose these benefits without even knowing we
had them.

Text Two, Public Library Of Science Blogs Network - Plants with Personality

If you had to be an endangered animal, youd be better off as a tiger than a
toad. If you were a tiger, filmmakers might cast you in wildlife documentaries
and journalists might write heart-rending stories about the disappearance of
your kind. Your furry mug might appear on magazine covers and postage
stamps. And conservation organizations just might make you their flagship
species, a stand-in for all the critters whose survival is threatened. In other
words, if you were a tiger, you might have a fighting chance of at least making
humans care about your predicament.

Thats a taller order if youre a toad, an animal that wins over few human
hearts. Instead, we prefer the so-called charismatic mega fauna, funneling
our emotional and conservational energies into species like tigers, lions,
elephants, dolphins, pandas, and the like.
A number of psychologists and biologists have begun to uncover why some
species appeal to us more than others, identifying a number of factors that
make certain kinds of critters especially attractive. For instance, we have a soft
spot for our fellow mammals, and we prefer big beasts to smaller ones. Were
Take your S AT prep to the next level. Visit www.learnerator.com
206

207

also strongly attracted to neotenic, or juvenile-looking, features. The
youngsters of many species have large heads, large eyes, big foreheads, and
snub noses. Human infants have these characteristics, as do puppies, kittens,
and all sorts of other critters that we find cute. In some species, adult animals
retain features associated with youthsuch as oversized eyesand were
naturally drawn to these neotenic faces.

...Our preferences for certain species over others have serious implications for
conservation. Studies have shown that charismatic mega fauna attract more
than their fair share of conservation attention and funding. As Stokes put it in
his 2006 paper: Much of the worlds biodiversity will survive only if humans
choose to protect it. Given that people are likely to protect what is important to
them, human preferences will be important determinants of many species
prospects for survival

As you read, take note of the fact that these two articles clearly discuss a similar
theme -- but that they take very different angles on this theme. Make sure that you
pay attention to similarities and differences in context, tone, style, and structure --
and, of course, to the content of the passages themselves.

What is the best way to characterize the difference between Text One
and Text Two?
A. Text One is persuasive, while Text Two is purely informative
B. Text One contains much scientific jargon, whereas Text Two is highly
descriptive
C. Text One provides general information about a scientific context while
Text Two details a phenomenon specific to that context
D. Text One is a call to action whereas Text Two is clearly written increase
awareness

Take your S AT prep to the next level. Visit www.learnerator.com


207

208

These two texts, while somewhat related in their subject matter, are in fact very
different -- partially because of their structure, partially because of their tone, and
partially because of their content. However, A claims that Text One is persuasive,
which -- if you read it closely -- it isnt. It doesnt contain much of a central claim,
other than the fact that extinction is a bad thing; nor does it spend too much time
trying to convince you that extinction is a bad thing. Instead, it simply and clearly
lays out the facts about extinction and genetic, ecological, and biological diversity.
For this reason alone answer A is wrong. B is incorrect as well -- while its true that
Text One contains many scientific definitions, Text Two contains at least two
unfamiliar scientific words as well (charismatic megafauna and neotenic.)
Additionally, Text Two hardly contains multiple adjectives or long descriptions; it is
somewhat informal, rather than highly descriptive. Answer C is true; Text One
DOES provide general information about why endangerment and extinction are
highly negative phenomena, and Text Two DOES detail a specific phenomenon
within the context of endangerment and extinction, as it describes the animals that
are most likely to be helped or saved by concerned humans. D is incorrect; Text One,
objective as it is, is hardly a call to action. There are no strong words and there is no
direct address of the reader; simply detailing why extinction and endangerment are
negative things surely does not constitute a call to action. And while Text Two DOES
inform the reader of a specific phenomenon, its hard to get away with saying that
its primary aim is to raise awareness, as that has a political connotation that is
absent here. All in all, C is the best answer.

In what way do the structures of these two texts help to shape the way
they convey information?
A. Text One provides a sequence of events that allows readers to better
understand the process of extinction; Text Two offers no such sequence
B. Both texts provide in-depth, objective explanations of scientific
phenomena, allowing the reader to better understand the material
C. Both texts lay out a problem and suggest possible solutions, offering the
reader an insight into some scientists approaches to mass extinctions
Take your S AT prep to the next level. Visit www.learnerator.com
208

209

D. Text One defines important terms while Text Two outlines cause and
effect, so that readers are better able to understand the immediacy and
importance of the phenomena described

There is a definite text structure to Text One; the multiple subheadings should alert
you to that immediately. However, this text structure is not at all a sequence of
events; youll be hard pressed to find even one chronologically significant signal
word (e.g. first, second, third, etc.) For this alone you can get rid of answer A. B
is certainly true of Text One, but its hard to make that same case for Text Two, as
the majority of the content is taken up with a catchy introduction and a less-than-
objective-sounding tone. Keep B for now but consider better options. The first part
of C is true -- both texts do lay out a problem -- but neither proposes a possible
solution. D is true -- Text One is full of important terms that help the reader better
understand the phenomena described, while Text Two does provide a cause
(humans are predisposed to like neotenic animals) and effect (non-neotenic
endangered animals are in greater danger than neotenic ones). Of all the answers, D
is the most true.

Both texts can be said to have the aim of
A. Galvanizing the reader towards change
B. Frightening the reader with grim tidings
C. Educating the reader with accessible language
D. Propelling the reader towards an unavoidable conclusion

This question asks you to make inferences regarding why both of these texts were
written. Assess the tone of these texts -- one is neutral and objective, laying out facts,
while the other is informal and engages more with the reader. But both contain
important scientific information that sheds light on certain aspects of the same
phenomenon. A may be correct -- its possible that the writers of both these texts
wished to spur the reader towards some kind of action -- but its also quite doubtful
considering that neither of these can really be considered a call to action, as neither
Take your S AT prep to the next level. Visit www.learnerator.com
209

210

uses fiery language or impels the reader towards any real kind of action.
Additionally, B is also possible -- but it is unlikely considering the notable absence of
any strong language or sobering statistics. C is certainly true -- though these articles
do contain some jargon, they both take care to either define these terms or to give
the reader enough context to understand them, and both pieces are written in a
style that is easy to follow; additionally, the textbook-like tone of the first passage is
a strong indicator that the author wished to educate and inform the reader. D is
interesting, but not very compelling -- which conclusion, exactly, does the second
article try to propel the reader towards? What words or sentences can be said to be
propelling the reader? Though you may indeed reach an unavoidable conclusion
after reading these pieces, the neutral, subdued tone of the first piece and the lack of
any conclusion in the second make it very hard to find evidence for D being the right
answer. C is the best answer.

Both Text One and Text Two contain unfamiliar words and phrases.
What is the approach these texts take to defining these phrases?
A. Both texts consistently define every unfamiliar phrase they use
B. Text One consistently defines phrases whereas Text Two defines some
phrases and lets the reader infer the meaning of others
C. Text One does not define relevant phrases whereas Text Two does
D. Neither text adequately explains the scientific phrases it uses

As explained above, these texts have similar aims in that they both seek to make
scientific phenomena clear and easy for the layperson to understand. Therefore
their approach to phrasing and terminology is very important; if you look closely at
Text One, it certainly defines all the terms it uses, from endangered to genetic
diversity. Yet Text Two does not ever define the phrase charismatic mega fauna,
leaving it to the reader to figure it out. This invalidates Answer A and makes Answer
B the only correct choice.

Take your S AT prep to the next level. Visit www.learnerator.com


210

211

Based on Text Two, charismatic mega fauna most closely means
A. Those with great personalities
B. Humans invested in saving animals
C. Animals that humans feel compelled to save
D. Plants with personality

You may be tempted to choose A based on your understanding of the word
charisma (magnetism), but remember, this is a scientific article about animals, so
its unlikely that the writer would devote much time to considering animals great
personalities. B is clearly wrong; the article uses the phrase in the following
context:

Thats a taller order if youre a toad, an animal that wins over few human hearts.
Instead, we prefer the so-called charismatic megafauna, funneling our emotional
and conservational energies into species like tigers, lions, elephants, dolphins, pandas,
and the like.

Replace the word charismatic megafauna with humans invested in saving
animals and it quickly becomes clear how ridiculous this answer is. C is a much
better choice. D references the title of the piece, but as there is no mention of plants
yet, it makes little sense in this context. Lastly, if you know the meaning of flora
(plants) and fauna (animals), the meaning of this should be clear, as only one of
the choices is animals.

How do the varying tones of these works influence the relationship
between the reader and the texts?
A. The objective tone of Text One introduces a problem to the reader,
whereas the informal tone of Text Two allows the reader to feel satisfied that
the problem is being addressed
B. The second-person tone of Text Two contains a subtle indictment of the
reader, whereas the detached tone of Text One distances the reader from the
Take your S AT prep to the next level. Visit www.learnerator.com
211

212

immediate problems of endangered species
C. The jargon in the first text gives the reader an unbiased, in-depth
perspective, while the informal tone of the second text clearly reveals the
authors perspective
D. The textbook-like tone of the first text, contrasted with the informal tone
of the second, make it clear that these articles would be read in very different
contexts.

This question is in essence asking you about the tone of the text. Dont be fooled by
this jargon-y idea of the relationship between the reader and the texts; simply
focus on the fact that the question is asking you about the tone. Assess the answer
choices and see which makes the most sense -- even if you dont understand the
question, you should be able to eliminate answer choices because some of them will
contain blatantly inaccurate information such as answer A. While the first part is
true, the informal tone of Text Two does nothing to address the problem. In fact, all
the text does is present a problem, without even a mention of a solution. This
answer is wrong. B is also wrong; there is absolutely no subtle indictment of the
reader here. Or rather, if there is a subtle indictment (criticism) of the reader, it is so
subtle as to be nonexistent. There are simply no words in this text that contain any
sort of criticism of the reader, or even imply it. This alone makes the answer wrong.
C is also wrong; we have no idea what the authors perspective is for the second
article. Just because an article is informal does not mean it is biased; this is
important to remember. The second article still clearly lays out all of the facts, and
quotes experts in the field -- while at the same time not even hinting at the authors
perspective. The only answer left is D, and it is clearly true -- these clearly WERE
written for very different contexts, such as a textbook or a magazine. In fact, you
dont need much beyond the little blurb at the top of each text to tell you this -- Text
One comes from a college website while Text Two comes from the Public Library of
Sciences Blogs network. Even if youre not totally sure what a question is asking
you, simply paying attention to the context can sometimes be enough to propel you

Take your S AT prep to the next level. Visit www.learnerator.com


212

213

towards the right answer.

These two passages are best read together because
A. Text Two defines and explains Text One
B. Text Two gives a reason for a phenomenon found in Text One
C. They give two oppositional sides of one story
D. Text One contains a general picture of a phenomenon while Text Two
details a specific feature of this phenomenon
This question asks you to consider why the evil geniuses behind the SAT paired
these texts. Clearly, all paired texts will have some sort of relationship. Clearly, A is
incorrect; considering its definition-heavy tone,Text One needs no additional
definition or explanation. B is tempting, but please consider that Text Two does not
actually give a REASON that certain species are endangered -- it simply gives a
reason for why certain endangered species are more cared about or impacted by
conservation efforts. Nor is C correct; these two articles do not give opposing sides
or differing accounts; they both simply describe certain features of endangerment. D
is clearly the best answer -- while Text One is about a general phenomenon
(endangerment and extinction), Text Two addresses another, smaller phenomenon
that is only possible in context of the phenomenon described in Text One. Clearly, D
is the best answer.

As you can see, synthesis will mostly center on asking you to compare and contrast
the various tones and approaches that two texts take. Remember, the SAT pairs
texts for a reason, and it wont always be as simple as one is pro, one is con. For
example, the texts above are clearly related yet not at all the same; as soon as youve
assessed the similarities (thematic or otherwise) between two texts, it will become
much easier to assess how they diverge and how they complement one another.
This will make it easier to understand how to answer Synthesis questions.

Take your S AT prep to the next level. Visit www.learnerator.com


213

214

Analyzing Quantitative Information


The new SAT will have a strong emphasis on the role that quantitative information
plays within a larger text. In a world that is increasingly governed by data, the
ability to identify, understand, summarize, and effectively use data to come to
conclusions is paramount. Data will make an appearance on all sections of the SAT --
not just math, but Reading and Writing, too. Luckily, data analysis is just another
form of reading comprehension, so all of the work you have been doing in the past
few articles will help you form data-based conclusions.
Perhaps the most important part of analyzing quantitative information is simply to
pay attention. Look at what information the data is presenting. Read all parts of the
graph or chart, including the labeling of the horizontal and vertical axes, and make
sure that you pay attention also to the way information and numbers are
categorized. Remember also that charts and data are not there to confuse you;
instead, they are there to help you better understand the text of the article, so think
of them as an added benefit to your reading.
Lets start with the following article, linked here. As you can see, the map is clearly
here to illustrate an essential theme of the article. The article asserts that North
Korea is very poor and gives a map as evidence, then explains how the map
functions.

What does the inclusion of this map show about poverty in North
Korea?
A. That it is restricted to only a few small sections of the country
B. That economic sanctions against North Korea have increased in recent
years
C. That the poverty in North Korea is a heinous crime
D. That, compared to neighboring countries, North Korea is very poor

It is important to understand what the question is really asking -- it is asking how
the map changes or deepens your understanding of poverty in North Korea
Take your S AT prep to the next level. Visit www.learnerator.com
214

215

specifically. But read carefully -- while A looks like it may be true (after all, the lights
are restricted to one part of the country) if you read closely youll see that the
clustering of lights is in fact an indication that resources, not poverty, are restricted
to just one area. The emptiness and lack of light in the surrounding areas denote
poverty. Clearly, considering how little light there actually is, the poverty isnt
restricted -- the resources are. A is wrong. B is true -- economic sanctions against
North Korea HAVE increased in recent years -- but is completely unrelated to the
data you see in front of you. (While the article does say that electricity use increases
in cities but dims in the countryside after new sanctions are imposed, wed have to
see a much more detailed breakdown of the light situation after a few sanctions in
order for this claim to be relevant to the data.) C is also true, and also supported by
the article, but has little to do with the data. Maps and charts reveal facts and
statistics; they do not make judgments. D is the best answer because remember that
the map compares two different but neighboring countries -- North and South
Korea. Without the comparison to South Korea, this map of North Koreas light
situation (or lack thereof) would be completely meaningless, as wed have nothing
against which to measure it, no real rubric for understanding this map. As this
question proves, you must read carefully and make sure you understand and are
answering the question, or else your choice will be inaccurate.

This article, too, contains data that will help you to better understand what the
article itself is trying to say?

Based on the chart, what inference can you make about the number of
MFAs that will be earned in 2012-2013?
A. The number will most likely continue to go up
B. The number will most likely stay the same
C. The number will most likely decrease
D. There is no prediction that can reasonably be made about this chart

Take your S AT prep to the next level. Visit www.learnerator.com


215

216

If you look at the chart -- and read the article -- its pretty clear that the number of
people earning MFAs has gone up significantly in the past few years, and that this
trend shows no sign of stopping despite the high costs and low pay of being an
artist. Based on the trends depicted in the graph -- as well as the fact that artists
consider their profession to be an honor, and tend to have wealthier parents than
the rest of the population -- it is pretty easy to predict that A is the only correct
answer.
This article contains a more challenging graph. Read the article, and look at the
graphs, then answer the questions. Some will ask you merely to point to data,
whereas others will ask you to analyze it:

Based on the first graph, what percentage of five-year-olds believe in
Santa?
A. Nearly 70%
B. Nearly 83%
C. Nearly 62%
D. Nearly 65%

This is the kind of question designed to see if you are paying attention. A refers to
the percentage of three year olds who believe in Santa, B is the correct answer, C
refers to number of three-year-olds who believe in the Tooth Fairy, and D refers to
the number of five-year-olds who believe in the Tooth Fairy. When answering
questions like this, all you need to do is pay attention to what the graph actually
says.

Based on both the article and the first graph, what conclusion can you
draw about the number of eight-year-olds who believe in Santa?
A. Twice as many eight-year-olds than nine-year-olds believe in Santa
B. Eight-year-olds on the whole no longer believe in Santa
C. Eight-year-olds who no longer believe in Santa most likely also no longer

Take your S AT prep to the next level. Visit www.learnerator.com


216

217

believe in the Tooth Fairy
D. Eight-year-olds are the most skeptical age group
The question asks about both the article and the first graph, so make sure that
youre using information from the article to answer this question. The article does
mention that a 1978 study found that only a quarter of eight-year-olds believe in
Santa -- though the graph would seem to contradict this finding because more than
25% of nine-year-olds believe in Santa, and it doesnt make sense for the belief in
Santa to go up and then down again once children reach a certain age. (Though its
true that belief in Santa goes up between the ages of three and five, this can be
explained by the fact that three-year-olds just might not be mature enough to grasp
the concept of Santa fully). Based on the trend in the chart (a decreasing belief in
Santa correlated with age) and the anecdotes in the article, it doesnt stand to reason
that twice as many eight year olds believe in Santa; not even twice as many seven-
year-olds believe in Santa than nine-year-olds, and based on the graph we can safely
assume that a belief in Santa will only decrease, not increase, with age. There is also
no evidence for B, considering that nearly a third of nine-year-olds believe in Santa,
according to the chart, so it makes little sense for no eight year olds to believe in
Santa. C is supported by the text as well as the chart; belief in Santa and the Tooth
Fairy appear to be related, as each time a belief in Santa grows or shrinks, belief in
the Tooth Fairy does the same. We can assume that eight-year-olds skeptical of
Santa will also be skeptical of the Tooth Fairy, but we cannot assume that eight-
year-olds are the most skeptical age group, especially given the data and the
evidence about Wooleys daughter (though the article does not say how old she
was).

Based on the second graph, adults of which religious group are most
likely to still believe in Santa?
A. No adults believe in Santa
B. Catholic adults are most likely to believe in Santa
C. Adults of another religion are most likely to believe in Santa
D. Mainline Protestant adults are most likely to believe in Santa
Take your S AT prep to the next level. Visit www.learnerator.com
217

218


Again, this question is just trying to see whether or not you are paying attention. All
you need to do is pay attention to the key -- the graph is color coded. The green bar
will tell you what percentage of which group of adults still believes in Santa; looking
at the green bar, it is clear that adults of Other Religion are most likely to still
believe in Santa. All this question is asking you to do is to look at information that is
already in the text.

What is the relationship between the two graphs?
A. Both graphs depict the same trend: that belief in Santa peaks between the
ages of 5-8
B. Both graphs depict the same trend: that children under five are more
likely to believe in Santa
C. Both graphs depict the same trend: that belief in Santa is correlated with
belief in the Tooth Fairy
D. Both graphs depict the same trend: that belief in Santa is correlated with
religion

Remember, this question is asking you about the relationship of the two graphs to
one another -- not about one graph in isolation. Consider the options carefully; the
first graph does indeed show that five-year-olds are the most likely of any age group
to believe in Santa, while the second graph is a little bit more generous and says that
five to eight year olds are the most likely to believe in Santa. In essence both graphs
say the same thing, though the parameters given by the second graph are slightly
larger than the parameters given in the first. Neither graph depicts the trend
described in option B, and only one graph depicts the trend found in C. Only the last
graph depicts the trend found in D. Clearly, there is only one right answer, and it is
A.

What is one surprising question that the data in the second graph
presents but does not explain?
Take your S AT prep to the next level. Visit www.learnerator.com
218

219

A. Why five-to-eight-year-old children of other religions are least likely to
believe in Santa while nine-to-twelve year old children and adults of other
religions are most likely to believe in Santa
B. Why 5-8 year-old children of no religion are most likely to believe in
Santa
C. Why adults believe in Santa
D. Why more kids dont believe in Santa

This is a bit of a tricky question, because the only role of this data is to present
information. Youll have to look at the data critically and in isolation -- meaning that
you cant use too much outsider information, like the idea that Christmas is
technically a Christian holiday, to drive your question. Instead, look only at the data.
The first option, A, is based on data and only on data -- it has nothing to do with
other religions. It simply depicts the most interesting and unexpected trend. Where
the rest of the religions are all more or less the same (belief peaks between 5-8 and
steadily declines), the Other Religion bar shows belief peaking at 9-12, declining at
older than twelve, and then peaking again during adulthood. Why would this
happen? The data does not behave like the data for the other religions, which is why
A is the best answer. B is also an interesting question, but it isnt surprising -- after
all, the findings there are consistent with the findings for all five-to-eight-year-olds;
and the percent of 5-8-year-olds of No Religion who believe in Santa is not
significantly higher than the percentage of most 5-8-year-olds who believe in Santa.
C has nothing to do with the data; personal beliefs are not the concern of data and
charts. Our only interest here is why the data is fluctuating -- hence why D is also
incorrect.

As demonstrated by the questions above, data analysis and the use of quantitative
information can enrich your reading experience tremendously. Pay attention to
detail and remember to think objectively about data, and you will see much success
on this portion of the SAT.

Take your S AT prep to the next level. Visit www.learnerator.com
219

220







Part IV: Expression of Ideas (Writing)

Take your S AT prep to the next level. Visit www.learnerator.com


220

221

Development
Text Development
Text development refers to the rhetorical structure and purpose of a text. That is, it is
a way to describe the components of a text that make that text successful in
communicating ideas.

Analyzing text development entails looking at four components:

Proposition the main claim or controlling idea of the text

Support the details, facts, and statistics used to back up the proposition

Focus the relevance of all of the details to the main claim

Quantitative Information information in graphs, charts, or tables that can


be related to information in the text.

Review the following text and chart before continuing the explanation that follows

Sample Text - Linking Poverty and Obesity
By now it has become common knowledge that America is suffering from an obesity
epidemic. The United States is home to the greatest number of overweight or obese
people in the world. In fact, 66% of all Americans are overweight. The statistical
frequency of obesity in the U.S. is a complicated number. Although America is one of
the wealthiest nations in the world, obesity is most often associated with its poorest
citizens. Researchers have identified a direct correlation between BMI, or body mass
index, a measurement of obesity, and income. However, the connection between
obesity and poverty is not a result of uneducated choices. Rather, those living in
poverty are more likely to become obese because of the unique challenges in
accessing nutritious foods one faces when living in a high poverty area.

In low-income neighborhoods, residents are less likely to find large grocery stores,
farmers markets, or other places where they can purchase healthy foods including
fruits, vegetables, whole grains, or low-fat dairy products. As their food shopping
Take your S AT prep to the next level. Visit www.learnerator.com
221

222

choices are often limited to convenience stores or corner stores, the poor often find
that their food choices are consequently limited to convenience foods, such as
frozen meals, canned foods, and other choices that are high in fat and salt, and often
contain highly refined grains and added sugar.

When high quality nutritious foods are available to those living in poverty, these
food items are often much more expensive than less nutritious, but more filling
foods, such as boxed pasta mixes. With limited funds to spend on food, poor
shoppers seek to maximize the calories per dollar ratio. For example, while a
healthy lunch choice, such as a chicken Caesar salad, provides about 60 calories for
each dollar spent, a frozen beef burrito provides 112 calories for the same dollar,
making it the more economical choice. Unfortunately, these inexpensive, high
calorie foods are not nutrient rich. Combined with the uniquely ample portion sizes,
most Americans have become accustomed to, these cheap, filling foods lead to
overconsumption and weight gain.

In addition, those living in low-income neighborhoods often have easy access to fast
food restaurants. In fact, in many neighborhoods, fast food restaurants outnumber
stores selling nutritious foods nearly four to one. Fast food restaurants offer high
calorie meals for relatively low prices. A fast food taco, for example, provides 100
calories per dollar. Unfortunately, the fast food taco also offers high fat and sugar
content, and very little nutritious value. Consuming fast food on a regular basis can
lead to rapid weight gain and obesity. Some people believe fast food restaurants
should be held responsible for obesity rates nationwide. However, these businesses
often counter than they provide affordable food options to Americans at all income
levels and that healthy choices are available on their menus.

Finally, while a simplistic analysis of obesity rates among the poor could lead to the
conclusion that lack of education, depression, or other factors such as substance
abuse could cause the higher rates of obesity among that population, a close analysis
demonstrates that the opportunity to make different food choices may not exist.
Take your S AT prep to the next level. Visit www.learnerator.com
222

223

Often the poor live in what have been called food deserts, areas where affordable,
nutritious food is difficult or impossible to obtain.


Proposition
The proposition is the main claim a text sets out to prove. One can usually find the
proposition near the end of the first paragraph. In an argumentative essay, the
proposition is also called the thesis statement.

In the article above, which sentence is most likely the proposition?
1. Rather, those living in poverty are more likely to become obese because of the
unique challenges in accessing nutritious foods one faces when living in a high
poverty area.
2. Although America is one of the wealthiest nations in the world, obesity is most
often associated with its poorest citizens.
3. Researchers have identified a direct correlation between BMI, or body mass
index, a measurement of obesity, and income.

The answer is A.

While all three sentences make a connection between poverty and obesity, only A
includes the argument the author is about to make, that the connection between
obesity and poverty is caused by a lack of access to nutritious foods.

Take your S AT prep to the next level. Visit www.learnerator.com


223

224

In the sample text, the proposition is introduced specifically in the introduction, and
confirmed in the conclusion. Note that both sentences of the conclusion make
reference to the availability of nutritious foods:

a close analysis demonstrates that the opportunity to make different food


choices may not exist.

Often the poor live in what have been called food deserts, areas where
affordable, nutritious food is difficult or impossible to obtain.

When analyzing a text to determine the proposition, it is crucial that the reader
consider the entire text. Finding the proposition is not as simple as choosing the last
sentence in the introduction.

Sample Question: How does the author introduce the proposition?
A. By lamenting America's obesity epidemic
B. By railing against the many factors that have led to the obesity epidemic
C. By leveling blame on the poor
D. By dispassionately outlining facts

Your clues for answering these questions can be found in the verbs that make up the
answers. There is simply no textual evidence to support Option A, which claims that
the author "laments" the obesity academic. This strong language suggests that the
author has a strong, regretful opinion of the epidemic -- but there is no evidence of
this kind of passion in the article. The same is true of B and C -- there is no "railing
against" or "blame." In fact, if you look at how the author introduces the proposition,
all you will see are facts outlined without real comment. In order to answer this
question, you must be familiar with both the terms -- i.e., to know what a
proposition actually is -- as well as the vocabulary used in the answer options. You
must also understand the difference between an objective tone (which is used here,
in this expository text) and a subjective one (which makes use of many adjectives,
descriptions, and clearly illustrates the author's opinion.)

Take your S AT prep to the next level. Visit www.learnerator.com
224

225

Support
The support consists of specific facts, examples, and details used to provide
evidence for the claim. Support can be in the form of statistic data, but may also
include elaboration or explanation that strengthens the proposition.
In the sample text, which information is not support for the proposition?
1. 68% of all Americans are overweight
2. chicken Caesar salad provides about 60 calories for each dollar spent
3. in many neighborhoods, fast food restaurants outnumber stores selling
nutritious foods nearly four to one.

The answer is A.

The percentage of Americans who are overweight is certainly a statistic, which
might lead one to conclude that it supports the proposition. However, here that is
not the case. The use of this statistic provides introductory context, and lets the
reader know why this topic is important, but the percentage of all Americans
suffering from obesity does not support the propositions connection between
obesity and poverty.

Choice B, however, provides information that explains why high calorie foods may
be chosen more frequently than healthier, lower calorie foods.

Choice C elaborates on the imbalance of food options available in high-poverty
neighborhoods, supporting the authors proposition that obesity is a result of a lack
of access to nutritious foods.

When analyzing a text to determine what information supports the proposition, it is
important that the reader does not assume all statistics or numbers are relevant
support. Understanding the text holistically is necessary in order to identify relevant
support.

Take your S AT prep to the next level. Visit www.learnerator.com
225

226

Sample question: What is the best support from the article for the claim
that the poor have unique challenges when it comes to making healthy
choices?
A. The poor have many issues with substance abuse in their communities
that make it hard to eat well.
B. Fast food restaurants do not offer healthy food options at all.
C. Lower-income neighborhoods are "food deserts" that do not offer
affordable high-quality foods.
D. High-calorie foods are often not nutrient-rich.

This question asks you to consider information that supports a claim. Often, answer
choices for questions like this will include information that was actually featured in
the article, but be careful to pay attention and make sure that this information
actually supports the claim. For example, while Option A isn't incorrect -- the poor
can have issues with substance abuse, which are mentioned in the article -- you
should not use outside knowledge, only what is explicitly stated in the text, to
demonstrate your understanding of the way the text is structured. In fact, the text
explicitly states that this explanation is "simplistic" -- so clearly it's not the best
support for this claim. B is incorrect as well, as the text states that "fast-food
restaurants provide affordable food options to Americans at all income levels and
that healthy choices are available on their menu." C is the best answer, as the writer
mentions in the last paragraph of the text that this is a challenge that the poor face.
D is also true, but has nothing to do with the claim statement in question.

Focus
When evaluating the focus of a text the reader should consider how and if each
paragraph clearly relates to the proposition. In the sample text, the proposition
makes a connection between poverty and obesity. Therefore, all of the information
in the body paragraphs should provide support for that claim.

Which of the following details is not relevant to the proposition?
Take your S AT prep to the next level. Visit www.learnerator.com
226

227

1. Some people believe fast food restaurants should be held responsible for
obesity rates nationwide.
2. Combined with the uniquely ample portion sizes most Americans have
become accustomed to, these cheap, filling foods lead to overconsumption
and weight gain.
3. poor shoppers seek to maximize the calories per dollar ratio

The answer is A.

Remember, the purpose of this essay is to support the proposition. The two
sentences at the end of the fourth paragraph that refer to the debate about whether
or not fast food restaurants are responsible for obesity are not related to this
proposition.
The writer argues that lack of access to affordable, high quality foods leads to
obesity. There is no claim that there is a connection directly to fast food restaurants.
And, the author also mentions convenience and corner stores as other sources of
low quality, low price food.

When analyzing the focus of a text it is important to keep the specific proposition in
mind. Information that seems to be related may only refer to the same topic, but not
the same controlling idea, as in this example. Again, it is impossible to analyze focus
without understanding the entire text.

Where is the most logical place to insert the following statement? "In
fact, many have claimed that it is this factor that differentiates
American consumers from consumers in other countries; no one knows
how these over-large servings developed, but experts argue that
without proper portion control there can be no answer to this
epidemic."
A. Right before this statement: "A fast food taco, for example, provides 100
calories per dollar. Unfortunately, the fast food taco also offers high fat and
Take your S AT prep to the next level. Visit www.learnerator.com
227

228

sugar content, and very little nutritious value."
B. Right after this sentence: "However, the connection between obesity and
poverty is not a result of uneducated choices."
C. Right after this sentence: "Combined with the uniquely ample portion
sizes, most Americans have become accustomed to, these cheap, filling foods
lead to overconsumption and weight gain."
D. Right after this sentence: "Finally, while a simplistic analysis of obesity
rates among the poor could lead to the conclusion that lack of education,
depression, or other factors such as substance abuse could cause the higher
rates of obesity among that population, a close analysis demonstrates that
the opportunity to make different food choices may not exist."

This question tests your understanding of how to best organize a piece of writing so
that it retains its focus. Pay attention to the sentence itself "it is this factor that
differentiates American consumers..." So you know that this sentence must have to
go after another sentence that introduces some new factor. Look at A. The sentence
before "A fast food taco, for example" reads "Fast food restaurants offer high calorie
meals for relatively low prices." Perhaps this phenomenon could be a differentiating
factor -- but look at the rest of the sentence, which is clearly about overlarge
servings. The preceding sentence makes no mention of overlarge servings, so this
cannot be the right place for a sentence about portion control.

If you look at the rest of the choices, the only one that mentions portion control is C -
- so C must be the best answer, as it is the only one that introduces the "unique
differentiating factor" of portion control, and is thus the best place for a sentence
about portion size.

Quantitative Information
Often a graph or chart will accompany a text and provide additional information
related to the text. When analyzing quantitative information, it is necessary to
consider the information in that graphic as it relates to the proposition in the text.
Take your S AT prep to the next level. Visit www.learnerator.com
228

229

Many conclusions can likely be drawn from a graphic representation, but only those
related to the proposition are relevant.

What related conclusion can the reader reasonably draw based on the information
in the graph Obesity Rate by Income?

1. Those earning more than $50,000 a year are more likely to have access to
farmers markets than those earning $30,000 a year.
2. Those earning $35,000 a year are less likely to be obese than those earning
$20,000 a year.
3. To avoid becoming obese Americans should try to earn at least $50,000 a
year.

Answer: A
According to the text one reason those living in poverty are more likely to be obese
is that they dont have access to large supermarkets or farmers markets where they
can purchase nutritious foods. Based on the claim in support of the texts
proposition, the reader can conclude that it is likely that wealthier Americans have
greater access to these sources of food.

Choice B is accurate, and it is obvious that more income is correlated to lower
obesity rates. However, there is no direct connection between this observation and
the proposition of the text. That is, this statement doesnt make the connection to
the availability of nutritious foods.

Choice C is not the correct choice because the proposition of the text is not to
advocate for higher wages or to suggest that those living in poverty simply try to
earn more money in order to improve their health. This observation is not related to
the text.

Take your S AT prep to the next level. Visit www.learnerator.com


229

230

Conclusion
Before responding to questions about the development of a text it is crucial that the
reader consider the entire text holistically and be particularly focused on the
proposition or main claim the text makes. Begin by underlining or highlighting the
proposition, and consider other components, including support, focus, and
quantitative information as they apply to that proposition.

Take your S AT prep to the next level. Visit www.learnerator.com


230

231

Organization
Text Organization
The organization of a text refers to all of the elements that make it logical and
cohesive. All levels of a text, from sentences to paragraphs, should be organized and
ordered logically and cohesively.
To analyze the organization of a text, it is necessary to consider four different
elements:

Sequence the order in which information is presented

Introductions how a text begins, and what information is provided to


establish context and purpose of the text.

Conclusions how a text ends, and the synthesis of ideas to leave the reader
with an understanding of the purpose and content

Transitions words and sentences used to move from one concept to


another within a text

Sequence
The sequence of a text is the order in which details are presented in support of the
main idea. Often, the thesis or proposition will establish the order of information.
There are several ways to organize details in a text:

Chronologically in time order. This strategy is particularly effective for


narrative fiction, biographies, or texts that recount events in history.

Description focusing on sensory details. This order divides details into


what can be seen, heard, smelled, tasted, and felt.

Process describes a sequence of steps necessary to complete a task

Division and classification sorts details into categories

Compare and contrast identifies similarities and differences between two


entities

Take your S AT prep to the next level. Visit www.learnerator.com


231

232

Causal (cause and effect) examines either a single case/effect relationship


or a causal chain, or series of components leading to a phenomenon, event, or
condition.

Examples of Organizational Strategies:


Strategy

Example

Chronologically

George Washington was born on February 22, 1732 in


Westmoreland, Virginia. When he was just 11 years old, in 1743,
Washingtons father passed way, and little Georges half-brother
took him into his home and raised him with his own children. In
1748, when Washington was 16, he went on his first surveying
job, mapping the western sections of Virginia.

Description

The sand beneath my feet is damp, cool and soft. A gentle breeze
wafts in from the water, bringing with it the smell of seaweed
and cotton candy from a carnival just a half a mile away. As I walk
closer to the carnival the sounds of people screaming in joyful
fear from the top of the roller coaster reaches my ears. I can still
taste the salt on my lips from my swim in the water earlier today.

Process Analysis Before mixing the ingredients, preheat the oven to 350 degrees.
Slowly mix the butter, flower, and cream together, being careful
to remove all lumps from the batter. Then, slowly add cocoa,
vanilla, and chopped nuts.
Division and

Developing a strong heart and healthy muscles requires a

classification

regimen of both aerobic and anaerobic exercises. Aerobic


exercises include swimming, walking, running and biking.
Anaerobic exercises include weight lifting and yoga.

Compare and

Both cats and dogs are considered valued pets and even

contrast

members of American families. There are differences between


these two most common pets that, if analyzed, can help a
potential pet owner understand which pet might be a better

Take your S AT prep to the next level. Visit www.learnerator.com


232

233

choice. For example, cats are often considered more difficult to
train than dogs, while dogs tend to require more maintenance,
including frequent exercising and grooming.
Causal

Poor lifestyle choices in ones youth can lead to serious health


issues that dont emerge until middle age. For example, people
who smoke between 10-20 cigarettes per day for five years or
more are 70% more likely to develop emphysema by age sixty
than those who smoke fewer cigarettes or dont smoke at all.


Use the text below to answer the questions about these concepts.
1. The Hindi-language film industry, based out of Mumbai, India, is one of the
most prolific producers of feature films in the world, producing up to 1000 films
per year. Nicknamed Bollywood, a combination of the name of the American
film-hub, Hollywood, and the colonial name of Mumbai, Bombay, the network of
studios produces hundreds of films per year, many of which reach worldwide
audiences. Unlike Hollywood, which is a geographical location, Bollywood is a
concept that includes production companies located throughout India and in
other parts of the world where Hindi-language films are produced.

2. Bollywood films have enjoyed a long and profitable history, beginning with
the first film, a silent picture produced in 1913, around the time when the
American movie industry was relocating from New York to California, and
producing the first Hollywood films. The first Bollywood film with sound
appeared in 1931, just four years after Hollywoods breakthrough talking
picture The Jazz Singer By the 1940s Bollywood films were garnering
worldwide attention, and in 1946 Neecha Nagar, a Bollywood film, won the
best picture award, called the Palme dOr, or Golden Palm, at the Cannes Film
Festival in France.

3. While the development of Bollywood films may have paralleled the growth of

Take your S AT prep to the next level. Visit www.learnerator.com


233

234

the American film industry in California, Hindi-language films are unique in
several ways, and hardly an imitation of American film genres. For example,
Bollywood films tend to be considerably longer than American films, stretching
up to three hours, as compared to two hours or less for American films. In
addition, almost every Bollywood film, regardless of genre, includes at least one
elaborate singing and dancing segment, which is often cast as dream within the
plot. These musical numbers are also a response to strict censorship codes that
were once imposed on films produced in India. Because characters could not
kiss, hold each other, or engage in more intimate contact, the music and
dancing came to represent romance within a love story. Eventually, the musical
numbers, and in fact, the films themselves developed unique aesthetic
signatures, including the bumping of shoulders between characters who were
romantically involved, exaggerated expressions of love or passion, and
elaborate dances that led to the romantic characters being almost, but not
quite, cheek to cheek.

4. The Bollywood film industry has experienced three distinct eras in its history.
Like the American film industry, Bollywood found its roots in a studio-based
model, or the studio era. During this time, from the dawn of Bollywood films to
about 1950, studios and the executives who ran them maintained almost all
control of film production, and garnered the most profits from those films.
Actors, directors, musicians, and all of the other creative people engaged in film
production were merely employees of the studios, often paid flat salaries, and
required to work on the films to which they were assigned for the duration of
their employment contract.

5. During World War II, it became difficult for Bollywood studios to procure
raw film stock. In fact, the only film stock available was on the black market,
and filmmaking became a moderately criminal activity. Producers engaged in a
wide range of crimes saw film production as a means to launder money, or turn

Take your S AT prep to the next level. Visit www.learnerator.com


234

235

illegally obtained profits into legal funds that could be banked or used for
purchases in general market. In the midst of this cycle, actors began to
command and receive very high salaries for their work, as the black market
studios saw actor salaries as a means of converting illegal profit to legal
business expenses.

6. Following World War II, and the demise of the black market studios, a few
film directors emerged as the most sought-after. These directors partnered
with the most successful and popular music directors and a few styles of
filmmaking and musical production dominated the industry from the 1950s
until the 1990s. As the celebrity of specific directors styles continued to
increase, so too did their salaries. Like the exorbitant salaries paid actors
during the war years, these expenditures were not sustainable. With the onset
of digital music sources and music pirating, the profits studios once saw from
selling the soundtracks to films all but evaporated, and it became increasingly
difficult to meet the salary demands of the music directors. In the 1990s several
studios were bankrupted and forced to close. This period in Bollywood history is
referred to as the Directors Era.

7. Some critics refer to the period from about 1998 to the present as a time of
transition, or the Transition Era. There are still some film and music directors
who command higher salaries than others, based on their past success and
unique approaches to film. However, they now accept package salaries, from
which they are required to pay many other employees, including musicians,
camera operators, etc. These directors are, in fact, contractors within the
industry. The new payment scheme encourages cost savings so that, for
example, instead of hiring a full orchestra for days at a time, the music director
might record all of the music in a short period of time and use digital music for
some sequences.

Take your S AT prep to the next level. Visit www.learnerator.com


235

236

8. Some actors, similarly, are paid more than others, and there are a few,
mostly male, actors who command salaries much higher than any other
performer. These actors, however, receive payment through a combination of
acting salaries, musical royalties, and payments for personal appearances and
endorsements.

9. Despite these hurdles and recent changes in the Bollywood film industry, it is
a vibrant and growing business. Today, Bollywood films are screened in ninety
different countries, and in India alone, about fourteen million people see a
Bollywood or Hindi-language movie every day.
The essay above uses compare and contrast to provide context for the details about
Bollywood films. Aspects of Bollywood films are compared to aspects of American films
in order to clarify points for an American reader.

What other organizational strategy is applied in the passage above?
A. Chronological
B. Process
C. Causal

Answer: A
The essay above is organized chronologically, which is a strategy well suited to
explain the history of a phenomenon. The essay starts with the beginning of
Bollywood films, in 1913, and ends with a description of current trends.

Introductions
A successful introduction must accomplish two goals:

Provide context or background for the information in the essay.

Reveal the controlling idea or proposition of the essay


Take your S AT prep to the next level. Visit www.learnerator.com


236

237

There are several ways to introduce a text, each of which is particularly applicable
to specific types of essays. This list is not exhaustive, but does provide some of the
most common strategies:

Describe a scene or tell an anecdote with a description or anecdote the


reader is brought into the world of the subject, either through empathy with
the subject, or with an understanding of the setting necessary to comprehend
the details.

Provide relevant background information this is historical or technical


information necessary to understand the purpose and relevance of the text.

Use a startling statistic or remark from a cited source this strategy is


particularly effective when writing about a topic that readers may not
consider a serious or important issue. A startling statistic or quote will
highlight the relevance of the subject.

Define an important term providing a definition can reveal rarely


considered context of a subject. Usually this is not a dictionary definition, but
one that relies on scientific research or discovery or a philosophical point of
view.

Take your S AT prep to the next level. Visit www.learnerator.com


237

238

Examples of Introductions
Type of Introduction

Example

Description/anecdote

The first time I saw a Bollywood film I was amused,


confused, and utterly thrilled. I can still remember how
surprised I was when a seemingly predictable romantic
comedy suddenly morphed into an elaborate musical
dance number. When I left the theater that afternoon, I was
hooked, and wanted to know more about the roots and
history of Bollywood.

Relevant background

There are more than 1,000 Bollywood films produced each

information

year, and these films are shown on almost every continent


every day. The first Bollywood film was produced in 1913.

Startling statistic or

I would love to work in a Bollywood film, as there is so

remark

much drama and color, in the films there (Pitt, 2013).


Brad Pitt, the seminal American film star is not alone in his
admiration for Bollywood cinema. These unique films
continue to read a broader, global audience of people who
are wowed by the unique spectacle only Bollywood can
deliver.

Define an important

The term Bollywood does not refer to a specific place.

term

Rather Bollywood is a term coined to refer to Hindi-


language films produced in Mumbai, India.


Review the text above. Which is most likely the introduction strategy
this author applied?
A. Provide relevant background information
B. Define an important term
C. Use a startling statistic

Take your S AT prep to the next level. Visit www.learnerator.com


238

239

Answer: A

In this essay the author provided relevant background information about the Hindi-
language film industry and the origin of the term Bollywood. This information
provides relevant context for the reader who is not familiar with these types of
films.

If the writer was to revise this introduction and focus on defining an
important term, which would be the most relevant term that would
benefit from a clearly articulated definition?
A. Bollywood
B. Hindi
C. Film concept

Answer: A

Bollywood may be an unfamiliar term to most readers, and it is a sort of
nickname applied to Hindi-language films. Since it is not a commonly understood
term, a thorough definition would benefit the reader by providing information
necessary to understand the rest of the essay.

Transitions
Transitions are words or phrases used to connect ideas between sentences or
between paragraphs. Effective transitions build coherence in an essay and ensure
the reader is able to follow a logical line of reasoning from the beginning to the end
of a text.
Transitional words and phrases fall into many categories. Review the categories and
examples below.

To show agreement or

In addition

Likewise

Take your S AT prep to the next level. Visit www.learnerator.com


239

Similarly

240

similarity:
To show contrast or

Although

contradiction:

On the other

Nevertheless

hand

To provide examples or

In other

Notably

In fact

emphasize a point:

words

To show cause/effect:

As a result

Because

Therefore

Sequence/time:

In the first

Prior to

Next

place
To summarize or conclude:

In conclusion Summing up

On the
whole


Review the text above. Which type of transition would most
successfully improve the logical development from paragraph (4) to
paragraph (5)?
A. Cause/effect
B. Sequence/time
C. Summarize/conclude

Answer: B

Paragraph (4) discusses the challenges faced by the film industry during WWII.

Paragraph (5) begins to describe a new era in Bollywood film, following WWII. This
is clearly a change in time and a sequence or time transition, such as following,
after, or later would be appropriate.

Review the text above. Which type of transition would most successfully
improve the logical development from paragraph (8) to (9)?
A. Cause/effect
Take your S AT prep to the next level. Visit www.learnerator.com
240

241

B. Sequence/time
C. Summarize/conclude

Answer: C

Paragraph (9) is the concluding paragraph of the essay. To signal the end of the text
and to alert the reader that summary or implication will follow, a summarizing or
concluding transition is appropriate.

Conclusion:
The conclusion of an essay pulls together all of the information presented and
comes to some final concept that expresses the relevance of the text. There are
several strategies that can be applied to the conclusion, including:

Brief summary of the essays main points the conclusion pulls the most
important information from the essay and synthesizes it into an objective
summary.

Provocative question challenges the reader to reconsider previously held


opinions on the topic

Call for action challenges the reader to do something about the topic.

Explain the significance or implications of the findings in the text how does
this information change the body of knowledge on the topic, and change how
the topic will be considered in the future?

Which of the strategies listed is most likely the one used by the author of
the text above?
A. Brief summary of main points
B. Provocative question
C. Explain significance or implications

Take your S AT prep to the next level. Visit www.learnerator.com


241

242

Answer: A

The conclusion of this essay explains the significance and implications, by explaining
that the Bollywood film industry will likely continue to prosper, despite challenges
faced over its long history. The inclusion of the statistic about the number of people
who view a Bollywood movie daily suggests a large audience for these films, further
solidifying the point.

Which other strategy could be successfully applied to this essay?
A. Brief summary of main points
B. Provocative question
C. Call to action

Answer: A

A brief summary of the essays main points would offer another effective conclusion
to this essay. Since the essay is expository, or informative, rather than
argumentative or persuasive, a call to action, such as See a Bollywood movie soon,
would not effectively synthesize the authors main points. The essay does not offer a
contentious issue or debate, so a provocative question would not be an effective way
to end the essay.

Take your S AT prep to the next level. Visit www.learnerator.com


242

243

Precision and Concision


Effective Language Use

Ideas are only communicated effectively if a text uses language effectively. There
are many considerations that impact how effective a text is in achieving the
writers rhetorical purpose. Among these are:

Precision The exactness of word choice

Concision The economy of word choice

Precision

Precise language uses the most accurate, descriptive term. Nouns are specific,
rather than general, and modifying words call on imagery to draw connections a
reader can relate to in order to develop a mental image of a situation that closely
matches the intent of the writer. Some characteristics of precise writing include:

Use of the active voice in which the noun or subject performs the action of
the verb.
o Active: Marta moved the couch to the center of the room.
o Passive: The couch was moved to the center of the room.

Use of precise words, rather than their definitions.


o

Precise: Marta rode a commuter train to work yesterday.

Imprecise: Yesterday, Marta utilized a mode of rail transportation


frequented by those who work some distance from their homes.

Convert nouns to verbs


o

Precise: Marta worked with other teachers to develop a new


curriculum.

Imprecise: Marta worked with other teachers in the development of a


new curriculum.

Take your S AT prep to the next level. Visit www.learnerator.com


243

244

Replace vague words with specific words


o

Precise: The band featured a talented singer with a soulful voice who
reminded me of Aretha Franklin.

Imprecise: The band was good.

Read the text below and continue on to the content that follows:
Californias San Joaquin Valley is home to more than ten percent of
Californias agricultural production, which means one of every ten farms in
the state is in the Valley.1 The United States relies on this narrow strip of
farmland, located in central California, for food and cotton2. In recent years,
however, an extended drought, which has gone on for a long time3, has
threatened crop production. To combat this drought, farmers have turned to
deep wells, which pump ground water from far below the earths
surface. What seems like a reasonable solution to a bad4 agricultural problem
may have significant unforeseen consequences. Recent research suggests that
the removal of ground water in the Valley has elevated mountains, caused
several small shifts of the earths crust, resulting in vibrations5, and has the
potential to trigger more frequent earthquakes. The earthquakes could also
become more intense.6
Review the excerpt (2) in the passage above. What would be the best way to make this
sentence more precise?

A. for growing food and cotton


B. for artichokes, oranges, lettuce, and cotton
C. for many kinds of food and cotton
D. for many plants used for food and manufacturing
Answer: B
This sentence already includes a specific crop, cotton. To make the entire
Take your S AT prep to the next level. Visit www.learnerator.com
244

245

sentence more precise, the best strategy is to list specific foods that are grown in
the region.

Review the excerpt (4) in the passage above. What would be the best way to make this
sentence more precise?

A. reasonable solution to a serious


B. reasonable solution to a worsening
C. reasonable solution to a growing
D. more reasonable solution to an increasing problem
Answer: B
The main idea of this passage is that the problems caused by deep well pumping
are becoming more serious. While growing suggests some increase, it is not as
precise as worsening, which clearly indicates the problem will only become
more severe as time goes on.

Concision

Concise writing is direct and to the point. It avoids restating ideas, and is clear
and specific. There are several strategies to evaluate the concision of a text,
including:

Identifying unnecessary words or phrases

Identifying passive sentences

Identifying negative sentences, which use more words than positive


sentences to express the same idea.

Using short, related sentences, rather than combining ideas into fewer,
concise sentences.

Read excerpt (5) in the passage above. What problem interferes with the concision of this
sentence?

A. Unnecessary words or phrases


Take your S AT prep to the next level. Visit www.learnerator.com
245

246

B. Passive sentence
C. Negative sentence
D. Improper word use
Answer: A
This sentence includes unnecessary words and phrases. Since the sentence
already uses the word earthquake, the author attempted to vary the sentences
by using the definition of an earthquake in this section. The result, however, is a
sentence that is overly wordy and potentially confusing. The sentence should be
revised.
Which is the most effective revision of excerpt 5 in the passage above?

A. caused small earthquakes


B. caused vibrations from shifts in the earths crust
C. caused several small shifts of the earths crust, called earthquakes
D. led to vibrations or earthquakes
Answer: A
Small vibrations caused by shifts in the earths crust are called earthquakes.
Avoid defining a term rather than naming it, especially when the term is
commonly understood by the average reader. If the term were highly technical or
specific to a certain discipline, it might be necessary to elaborate, but that is not
the case in this excerpt.

Review excerpt (6) in the passage above. What problem interferes with the concision of the
text?

A. Passive sentence
B. Negative sentence
C. Lack of concision in sentences

Take your S AT prep to the next level. Visit www.learnerator.com


246

247

D. Lack of precision in word choice
Answer: C
The last sentence in the text provides additional information about the
predictions for future earthquakes. The result is a passage that is overly wordy,
and the suggestion that new, more detailed information will be presented. The
passage should be revised.

What is the best revision of excerpt (6) in the passage above?

A. and has the potential to trigger more frequent and intense


earthquakes.
B. and has the potential to trigger more frequent earthquakes that could
be more intense.
C. and has the potential to trigger earthquakes. The earthquakes could
be more frequent and more intense.
D. and has the potential to trigger more intense earthquakes that
happen more frequently.
Answer: A
Option A combines the important details from both sentences into a single,
concise sentence that successfully communicates that future earthquakes may be
both more frequent and intense.

Take your S AT prep to the next level. Visit www.learnerator.com


247

248

Style and Tone


Effective Language Use

Ideas are only communicated effectively if a text uses language effectively. There
are many considerations that impact how effective a text is in achieving the
writers rhetorical purpose. Among these are:

Style most often refers to the diction of the writing, which can be formal,
informal, or very informal. The style is the created through the authors
word choice and organization. An authors style is also influenced by his
or her intended audience. For example, someone writing for an audience
of children will maintain a different style than an author writing for
graduate students.

Tone - the writers attitude toward the subject, which must be appropriate
to the purpose.

Style

Style refers to how something is written to achieve a specific purpose. The style
of the writing affects the readers impression of what he or she is reading,
including assessments of the credibility and persuasiveness of an argument. The
style of a text must remain consistent in order to achieve its purpose. Review the
passage below and assess the writer's style, paying careful attention to the words
and sentence structure that s/he uses.

Take your S AT prep to the next level. Visit www.learnerator.com


248

249

Venice, Italy is a city composed of 117 small islands, connected by bridges and
canals. There is no wheeled traffic in the streets of Venice. In other words, there
are no cars, no bicycles, no scooters, and no horse carts in the entire city. There
is, however, a public transportation system, consisting of vaporetti, or water
buses, which provides service to the citys main canals, the lagoon surrounding
the city, and the train station. Visitors travel to Venice either by waterbus, from
nearby Marco Polo Airport, or by train from many cities in Europe. The train
station, Venezia Santa Lucia, is located on the banks of the Grand Canal,
Venices main thoroughfare.
Think about how this passage is written. Is it informative? Does it give
directions? Is it poetic, with a lyrical style? Does it directly address the reader or
give the writer's opinion? Once you assess the passage and consider these
factors, take a stab at the following question:

Which sentence most effectively matches the style of this passage and could be added to the
end of the paragraph?

A. On my last trip to Venice, the lack of wheeled traffic drove me crazy!


B. Those arriving by train can transfer to local waterbuses, which offer
stops near most hotels and tourist attractions, or hire private water-taxis
to take them directly to the their destination.
C. Though many of the ancient canals are choked by trash and grime, the
water retains a smoky, mystical quality when lit by the soft glow of
streetlights that wink
D. If arriving by train, you can either take one of the waterbuses, which
stop at most of the hotels, or get a private water taxi to get right to the
doorstep of your own hotel.
Answer: B

Take your S AT prep to the next level. Visit www.learnerator.com


249

250

This sentence matches the style of the passage for several reasons. First, the
sentence is informative -- rather than narrative or persuasive -- matching the high
level of detail and elaboration in the rest of the passage. Second, it maintains
focus on visitors to Venice, rather than residents, and, third, it does not express
an opinion or address the reader directly (you). While this is a passage intended
for visitors, it is more informative than directive.
Taking these ideas into consideration, review the passage below:
Motivated by concern for the environment, as well as a desire to avoid large financial
commitments, fans of tiny houses, or microhomes, have rejected the typical 2600 square
foot American house in favor of homes measuring just 100 to 400 square feet. Tiny
homes offer many benefits. Residents pay less to heat and cool their homes, and property
taxes are much lower than those on typical houses. Tiny homes consume less resources
in construction and use less energy overall. There are, however, some drawbacks to
micro-living. Residents often find it difficult to entertain, because there is very little
space for guests, or even for additional seating. Finally, living in a microhome requires
discipline. Since a small space can quickly become cluttered, residents must discard
anything they dont use regularly.
Which of the following sentences is aligned with the style of this passage and could be added
as the first, or topic sentence of the paragraph?

A. In the last decade a growing number of Americans have embraced the


tiny house movement.
B. Micro is a Latin prefix that means very small.
C. Tiny homes have pros and cons, but mostly they are more affordable
than traditional homes and much better for the environment.
D. Microhomes are also called tiny homes and they are a new fad that is
really growing in popularity.

Take your S AT prep to the next level. Visit www.learnerator.com


250

251

Answer: A
Begin by analyzing the overall style of the passage. Notice the word choices the
author makes, large financial commitment, rather than big expense, and
motivated by concern for the environment, rather than because they care
about the environment. This analysis tells the reader that the writer anticipates
a specific audience adults familiar with home ownership and familiar with
trends in environmental activism. From this analysis, we can assume the diction,
or word choice, will be elevated, and the author will not define terms (as in
Option B) or use informal language (as in D -- "really growing in popularity" is
too colloquial, or conversational, to fit in with this style). Nor will the author
express an opinion (as in C) -- instead, s/he simply lays out the facts.

Tone

The style of a piece is often closely linked to the tone. The tone of a text expresses
the writers attitude toward the subject. The most effective way to analyze tone is
to establish if the text is objective or subjective.

Objective tone

Impartial, neutral, unbiased

Does not show an opinion or preference for a topic

Uses words with neutral/objective connotations ("economical" rather


than "cheap," for example).

Avoids first and second person pronouns

Generally more formal diction or style

Subjective tone

Personal point of view

May use first or second person pronouns

Often relies on emotions and personal experiences or impressions

Take your S AT prep to the next level. Visit www.learnerator.com


251

252

Uses evaluative words (clever, thoughtful, lazy, argumentative)

Biased or clearly opinionated

Generally informal diction or style

In both objective and subjective texts, the reader can further analyze the tone by
evaluating the connotations, or implications, of the words the author chooses. The
authors attitude toward the subject is revealed in the evaluative or connotative
words he or she uses.
Review the following excerpt from Wuthering Heights:
This time, I remembered I was lying in the oak closet, and I heard distinctly the gusty
wind, and the driving of the snow; I heard, also, the fir bough repeat its teasing sound,
and ascribed it to the right cause: but it annoyed me so much, that I resolved to silence it,
if possible; and, I thought, I rose and endeavoured to unhasp the casement. The hook was
soldered into the staple: a circumstance observed by me when awake, but forgotten. I
must stop it, nevertheless! I muttered, knocking my knuckles through the glass, and
stretching an arm out to seize the importunate branch; instead of which, my fingers
closed on the fingers of a little, ice-cold hand! The intense horror of nightmare came over
me: I tried to draw back my arm, but the hand clung to it, and a most melancholy voice
sobbed, Let me inlet me in! Who are you? I asked, struggling, meanwhile, to
disengage myself. Catherine Linton, it replied, shiveringly (why did I think of Linton? I
had read Earnshaw twenty times for Linton)Im come home: Id lost my way on the
moor! As it spoke, I discerned, obscurely, a childs face looking through the
window. Terror made me cruel; and, finding it useless to attempt shaking the creature
off, I pulled its wrist on to the broken pane, and rubbed it to and fro till the blood ran
down and soaked the bedclothes: still it wailed, Let me in! and maintained its tenacious
gripe, almost maddening me with fear. How can I! I said at length. Let me go, if you
want me to let you in! The fingers relaxed, I snatched mine through the hole, hurriedly
piled the books up in a pyramid against it, and stopped my ears to exclude the lamentable
prayer. I seemed to keep them closed above a quarter of an hour; yet, the instant I listened

Take your S AT prep to the next level. Visit www.learnerator.com


252

253

again, there was the doleful cry moaning on! Begone! I shouted. Ill never let you in,
not if you beg for twenty years. It is twenty years, mourned the voice: twenty
years. Ive been a waif for twenty years! Thereat began a feeble scratching outside, and
the pile of books moved as if thrust forward. I tried to jump up; but could not stir a limb;
and so yelled aloud, in a frenzy of fright. To my confusion, I discovered the yell was not
ideal: hasty footsteps approached my chamber door; somebody pushed it open, with a
vigorous hand, and a light glimmered through the squares at the top of the bed. I sat
shuddering yet, and wiping the perspiration from my forehead: the intruder appeared to
hesitate, and muttered to himself. At last, he said, in a half-whisper, plainly not
expecting an answer, Is any one here? I considered it best to confess my presence; for I
knew Heathcliffs accents, and feared he might search further, if I kept quiet. With this
intention, I turned and opened the panels. I shall not soon forget the effect my action
produced.
Which words best described the authors tone in the passage?

A. Frightened and embarrassed


B. Apprehensive and angry
C. Panicked and spiteful
D. Alarmed and callous
Answer: D
The narrator in this passage is alarmed by the noise outside the window, and
again by the clasp of the ghostly hands on his own. He states, Terror made me
cruel, and goes on to explain how he tricked the ghost into releasing him, only
to taunt her by telling her he will never let her in.
Review the following sentence from the passage above:
I heard, also, the fir bough repeat its teasing sound, and ascribed it to the right
cause: but it annoyed me so much, that I resolved to silence it, if possible; and, I

Take your S AT prep to the next level. Visit www.learnerator.com


253

254

thought, I rose and endeavored to unhasp the casement.
What tone does the author employ in this sentence to best express the narrators emotional
state?

A. Fear
B. Anxiety
C. Frustration
D. Apprehension
Answer: C
In this sentence the author chose words such as teasing and endeavored to
communicate a frustrated tone, as the narrator complains about an annoying
sound outside his window and struggles to open the window to resolve the
problem. (This question has a very obvious answer if you read the text: "It
annoyed me so much...")
Sometimes a writer will let his/her feelings creep into an otherwise neutral piece.
Use context clues, such as words that demonstrate emotion, to ascertain which
parts of a text display some sort of bias.
Review the excerpt below before going on to the questions that follow:
The Dark Romantic authors are a favorite among middle school and high school
readers. The works of Poe, and to a lesser extent Hawthorne and Melville, touch
on the depraved, the grotesque, and the horrific, all topics that seem elicit, yet
appealing to the adolescent mind.1 Many Americans, in fact, report that the most
memorable reading experiences of their secondary academic careers involved
reading the works of one or more of these authors. In contrast, the
Transcendentalist works of Emerson and Thoreau, when they are taught, tend to
leave student-readers feeling confused and disconnected from the text. Although
the Dark Romantics emerged directly as a response to Transcendentalism, the

Take your S AT prep to the next level. Visit www.learnerator.com


254

255

connection between the two genres is seldom explored in secondary classrooms,
thus leaving students with an incomplete understanding of the works they so
enjoy. 2Unlike the Transcendentalists, who celebrated the inherent perfection of
the human condition and the notion that the spirit of each individual, if it could
be reached, would connect all people and nature, the Dark Romantics asserted
that mankind was inherently flawed, fallible, and prone to sin and self
destruction.3 That is, while the heroes of Transcendental literature sought their
more perfect selves, the protagonists of Dark Romantic literature struggled to
avoid their natural states of depravity, and while the Transcendentalists saw the
supreme divinity personified in the natural world, accessible to anyone who could
transcend the

selfishness of the individual, the Dark Romantics perceived the world

around them to be populated with anthropomorphic representations of the devil


and his demon-minions.4 Almost every character in a Dark Romantic text has the
potential to be Satan himself.
Edgar Allen Poe, perhaps the most well-known of the Dark Romantics,
rejected Transcendentalism to the point of publicly mocking the
movement for what he considered an overtly literary approach that lacked
meaning. He decried the use of metaphor for metaphors sake, and
thought many of the self-proclaimed Transcendentalists were pretenders
to the movement who produced falsely mystical stories which included
impossibly spiritual outcomes.5 Poes own works were populated by
criminals and madmen. As the inventor of American detective fiction, Poe
created a genre dedicated to identifying and flushing out evil in society.
His focus on motive, on what made criminals commit crime, frequently
led his detectives to identify deep-seeded madness within a criminal, and
to suggest that all humans are capable of such depravity, but that some
are more successful than others at suppressing it.6

Take your S AT prep to the next level. Visit www.learnerator.com


255

256

While Poes reaction to Transcendentalism was primary based on what
hed read and those he encountered who were engaged in the movement,
Nathaniel Hawthorne found himself drawn away from
Transcendentalism to Dark Romanticism because of his own experience
and family history.
Which sentence in the passage above most demonstrates a biased or subjective point of
view?

A. Sentence (1)
B. Sentence (2)
C. Sentence (4)
D. Sentence (5)
Answer: B
In the following sentence, the author demonstrates a subjective opinion of the
teaching of Transcendentalism and Dark Romanticism in secondary classrooms.
The author notes, Although the Dark Romantics emerged directly as a response
to Transcendentalism, the connection between the two genres is seldom explored
in secondary classrooms, thus leaving students incomplete understanding of the
works they so enjoy. This sentence suggests that teachers ignore the connection
between the two schools of literature and thus neglect making connections
necessary for students to best understand literature they already enjoy.

What revision of sentence (2) best results in an objective tone?

A. While 90% of secondary teachers report teaching the works of the Dark
Romantics, only 40% report that they consistently teach the works of the
Transcendentalists, whose works inspired the Dark Romantics.
B. Most teachers cover the Dark Romantics in great detail but dont bother
with the Transcendentalists because their work is too challenging for high

Take your S AT prep to the next level. Visit www.learnerator.com


256

257

school students.
C. Teachers should cover both the Dark Romantics and the
Transcendentalists, since it is difficult to understand the works of authors
like Poe if one hasnt read authors like Emerson.
D. Dark Romantic literature is very popular with students, so it is taught
more often than Transcendentalism, which students dont like much at all.
Answer: A
Using statistics is one of the best ways to avoid a subjective or biased tone. Note
that this revision does not suggest that teachers are providing an incomplete
education, or that either genre is neglected. The revision includes only facts that
can be supported and confirmed with evidence.

Review sentence (3) from the passage. Which revision most effectively improves the style of
the writing?

A. Unlike the Transcendentalists who celebrated the inherent perfection of


the human condition and the notion that each individual could reach his
own spirit and connect it to those of all other people and nature, the Dark
Romantics asserted that mankind was flawed, fallible, and prone to sin
and self destruction.
B. The Transcendentalists thought people were most good, while the Dark
Romantics thought people were generally evil.
C. The Transcendentalists celebrated mans spirit and how it could be
connected to the spirits of nature, while the Dark Romantics insisted that
mankind was flawed, fallible and prone to sin and self destruction.
D. Unlike the Dark Romantics, who saw that self-destruction was
inevitable for mankind, the Transcendentalists celebrated how perfect the
human condition was and how all living things are connected.

Take your S AT prep to the next level. Visit www.learnerator.com


257

258

Answer: A
Revision A corrects the passive form of the sentence by suggesting that the
individual can connect his spirit, rather than the passive construction, the
spirit, if it could be reached, would connect
Review the sentence below from the passage:
The works of Poe, and to a lesser extent1 Hawthorne and Melville,
touch on the depraved2, the grotesque3, and the horrific, all topics
that seem elicit4, yet appealing to the adolescent mind.
Which section includes an error in diction?

A. 1
B. 2
C. 3
D. 4
Answer: D
In option D (4), the writer has made a word-choice error. Elicit means to evoke
or drawn a reaction or result. It is likely this author intended to use the word
illicit, which means forbidden.
Review the sentence below from the passage:
His focus on motive1, on what made criminals commit crime,
frequently led his detectives to identify deep-seeded madness2
within a criminal, and to suggest that all humans are capable of
such depravity3, but that some are more successful than others at
suppressing it4.

Take your S AT prep to the next level. Visit www.learnerator.com


258

259

Which section includes an error in diction?

A. 1
B. 2
C. 3
D. 4
Answer: B
This is an error in the use of a phrasal adjective. The correct phrase is deepseated, meaning that something is seated or firmly in place within a context.
In this case, the madness noted is firmly in place within the criminal mind.
You can determine the style and tone of a piece by paying attention to the words
used -- particularly if they elicit a specific emotion, communicate a bias, or argue
a point. As you read, pay attention to the diction of a piece, and consider its
intended audience. Remember too that all written works have some sort of style,
whether it's formal or informal, objective or subjective, or somewhere in
between. This will help you accurately answer questions that the new SAT may
pose about style, tone, and grammar.

Take your S AT prep to the next level. Visit www.learnerator.com


259

260

Syntax
Syntax
Syntax questions require readers to consider a text at the sentence level. That is, the
reader should be able to identify and apply necessary changes to sentences in order
to improve the clarity, precision, or logic of the writing. Questions on this topic
might require students to:

Combine short or related sentences

Identify sentence revisions that improve logical connections between ideas.

Identify sentence revisions that correct grammar errors.

Identify sentence revisions that improve clarity.


Read the passage below, and consider the relationships between the sentences as
you do so:
Puritan literature is rooted in the belief that God is always at work in the daily
lives of humans. Most texts describe the discovery of evidence of Gods
intervention or participation in human lives.1Texts often include allusions to
classical literature, as Puritans valued education.2Because they eschewed
flowery prose, the allusions are straightforward and direct. 3Puritan literature
is the seed of the American identity.4It stresses self-reliance and overcoming
hardship to achieve a goal.5These values are often associated with pursuit of
the American Dream.6

Which transition added to the beginning of sentence (2) would best
indicate the relationship to the preceding sentence?
A. Although
B. As a result
C. To begin with

Take your S AT prep to the next level. Visit www.learnerator.com


260

261

D. Moreover

Answer: B

The first two sentences of the passage indicate a cause/effect relationship. The first
sentence describes the root of Puritan literature. The second sentence describes
the focus of the literature resulting from that root-belief.

Which is the best revision of sentence (3)?
A. Because the Puritans eschewed flowery prose, the allusions are
straightforward and direct.
B. To avoid flowery prose, the allusions are straightforward and direct.
C. They eschewed flowery prose; the allusions they preferred are
straightforward and direct.
D. In lieu of flowery prose, the allusions are straightforward and direct.

Answer: A

To answer this type of question the reader should begin by confirming what the
sentence intends to say. In this case, there are two main points: the Puritans
rejected (eschewed) flower prose, and, as a result of that rejection, the allusions are
straightforward and direct. This sentence has a dangling modifier error, caused by
the lack of an antecedent for the pronoun they in the first clause. That is, this
sentence seems to say that they refer to the allusions in the second clause. In fact,
they refers to the Puritans. So, replacing they with Puritans, corrects the
sentence and makes it clear.

Which sentence most effectively combines sentence (5) and sentence
(6)?
A. It stresses self-reliance and overcoming hardship to achieve a goal, values
often associated with pursuit of the American Dream.
Take your S AT prep to the next level. Visit www.learnerator.com
261

262

B. It stresses self-reliance, overcoming hardship to achieve a goal, and values
often associated with the American Dream.
C. It stresses self-reliance and overcoming hardship to achieve a goal; values
often associated with pursuit of the American Dream.
D. It stresses values associated with the pursuit of the American Dream like
self-reliance and overcoming hardship to achieve a goal.

Answer: A

To choose the most effective sentence the reader must understand the context of the
passage. The main point of the last three sentences is that Puritan literature was a
seed from which the American identity grew meaning it came before the
American identity, which makes sense, since there was no America when it was
written.

Understanding the context enables the reader to dismiss choice D as incorrect,
because it suggests that the Puritans were in pursuit of the American Dream, which
was not the case. Choice B indicates that the values often associated with. are
different from self-reliance and overcoming hardship, which is not the case. Choice C
is not correct because one should use a semicolon (;) only between two independent
clauses. The second clause in this sentence could not stand-alone; it is not an
independent clause. Choice A is correct.

Review the passage below:
Most people are familiar with catnips effects on cats, but many are not
aware that catnip has a long history of use as a medicinal and culinary
herb.1A member of the mint family, catnip can also be used in tea, and
as a savory accent to soups and stews.2While American cooks prefer
more subtle seasonings, French cooking uses the sharp, minty taste of
catnip in many well-known dishes.3One popular French dish that uses

Take your S AT prep to the next level. Visit www.learnerator.com


262

263

catnip is bouillabaisse, a fish stew.4

Take your S AT prep to the next level. Visit www.learnerator.com


263

264

Review the sentence below:
Catnip is used in traditional medicines to treat insomnia, stomach upset, and
headaches.

Where would this sentence best fit within the given passage?
A. After sentence (1)
B. Before sentence (1)
C. After sentence (2)
D. Before sentence (4)

Answer: A

Sentence (1) is the topic sentence, and it establishes two focus areas: medicinal and
culinary uses for catnip. Since medicinal is mentioned first, it would logically be
elaborated on first. Note, as well, that sentence (2) reads, catnip can also be used
This transitional word indicates a comparison. Adding the given sentence before
sentence (2) provides the point of comparison.

Which is the most effective revision of sentence (3)?
A. While American cooks prefer more subtle seasonings, French chefs use the
sharp, minty taste of catnip in many well-known dishes.
B. While American cooks prefer more subtle seasonings, the French use the
sharp, minty taste of catnip in many well-known dishes.
C. Although American cooks prefer more subtle seasonings, French cooking
uses the sharp, minty taste of catnip in many well-known dishes.
D. While American cooks prefer more subtle seasonings, but French cooking
uses the sharp, minty taste of catnip in many well-known dishes.

Answer: A

Sentence (3) includes an error in comparison. The sentence begins with American
Take your S AT prep to the next level. Visit www.learnerator.com
264

265

cooks, so it refers to the people who do the cooking. In the existing sentence, cooks
are compared to cooking, which is flawed. To correct the sentence, the comparison
must be between cooks or between types of cooking. Only A revises the sentence so
that cooks are compared to chefs (cooks).

Understanding syntax is an important part of honing your verbal skills. As you read,
consider the relationships that each sentence sets up. Are the sentences all
grammatical? Do they introduce or reinforce direct and logical relationships to one
another, or to the content of the text? Paying close attention and thinking critically
about these ideas will help you to score high on the SAT.

Take your S AT prep to the next level. Visit www.learnerator.com


265

266







Part V: Standard English Conventions
(Writing)

Take your S AT prep to the next level. Visit www.learnerator.com


266

267

Sentence Formation
Basic Parts of a Sentence
A simple sentence consists of a subject and a predicate. The subject of the sentence
is a person, place, thing, or idea. The predicate of the sentence contains a verb that
states something about the subject.

He walks.

John writes.

They talk.

The first words in the examples above are subjects, and the final words in these
examples are predicates. This is the basic structure of all simple sentences.

The Subject
The subject of a sentence can be a noun, a pronoun, or a noun phrase. It usually
appears at or near the beginning of a sentence. The subjects in the following
examples have been underlined.

These sentences have nouns as their subjects. Nouns can be common, proper, or
collective. Always remember that the noun that functions as a subject will NEVER be
part of a prepositional phrase.

The dude lied. (common noun)

Lilith dances beautifully. (proper noun)

The pack of wolves howled at the moon. (collective noun)


These sentences have pronouns as their subjects. Always use subjective pronouns as
subjects, and never use an objective pronoun.

Take your S AT prep to the next level. Visit www.learnerator.com


267

268

He giggles. (singular pronoun)

Yesterday, they fell down the stairs. (plural pronoun)

Me and Joe are very good friends. (incorrect usage of an objective pronoun)

These sentences have noun phrases as their subjects. Although a noun phrase is
made up of multiple words, they collectively form a single component that functions
as a normal noun in the sentence. Noun phrases may be gerund phrases or infinitive
phrases.

Lying about yourself is bad. (gerund phrase)

To learn is a good thing. (infinitive phrase)


Verbs
All predicates must contain at least one verb. The three primary types of verbs are
action verbs, linking verbs, and auxiliary verbs. The verbs in the following examples
have been underlined.

Action verbs are expressive of an activity being performed. An action verb can be
either transitive or intransitive. A transitive verb is followed by a direct object. An
intransitive verb is not followed by a direct object. A direct object cannot be found in
a prepositional phrase.

The dog runs. (intransitive verb)

A fireman climbs the ladder. (transitive verb--the word ladder is the direct
object)

The dolphins swim in the water. (intransitive verb--the word water is not the
direct object because it is the object of a prepositional phrase)

A linking verb attaches a subject compliment to the subject. The subject


compliments in these sentences are in italics. A subject compliment can be a noun

Take your S AT prep to the next level. Visit www.learnerator.com


268

269

that renames the subject or an adjective that describes the subject.

The food on the table smells delicious.

The old man looks very happy.

The fireman is Joe.


Auxiliary verbs are smaller verbs placed in front of the main verb. Up to three
auxiliary verbs may be used in a sentence along the main verb. Auxiliary verbs
express tense, mode, and voice.

Jane is baking. (expresses tense - present participle)

Jane could have baked the cake. (expresses mode - possibility)

The cake might have been baked by Jane. (expresses voice - passive)

Direct Objects
The predicate of the sentence may also contain a direct object, an indirect object, or
a subject compliment. The direct object receives the action of the verb, and the
indirect object receives what is being given or done. The subject compliment is an
adjective or a noun and either describes or renames the subject.
The italicized words are examples of direct objects. Direct objects can be nouns,
pronouns, noun phrases, or subordinate clauses.

Robert reads books.(noun)

Jane hit him. (pronoun)

She loves running in the morning. (noun phrase)

Bill hates when she doesn't listen. (subordinate clause)


Indirect Objects
The words in italics are examples of indirect objects. Indirect objects can be either
nouns or pronouns. Indirect objects can also be placed within prepositional phrases.

Phil brings Jane candy. (noun)

Take your S AT prep to the next level. Visit www.learnerator.com


269

270

He finds you money. (pronoun)

Bill gives food to the needy. (inside a prepositional phrase)


Subject Compliments
The words in italics are subject compliments. A subject compliment can be a noun or
a pronoun that renames the subject or an adjective that describes the subject (use
subject form of pronoun as a subject compliment--it sounds strange, but it is
correct). A subject compliment is always followed by a linking verb. A subject can
also be a participial phrase or an infinitive phrase).

Jim is the principal. (noun)

The principle is he. (pronoun - subject form)

Mountains are beautiful. (adjective)

She seems to be distracted. (infinitive phrase)

She is floating on a cloud. (participial phrase)


Clauses
A clause is formed with a subject and a predicate like a sentence; however, it is used
as a building block for a larger sentence that is more complex. A clause may be
either independent or subordinate.

Independent clauses are capable of standing alone as complete sentences. Every
sentence must contain at least one clause. An independent clause expresses a
complete thought.

He sat on his porch while his cat played on his lawn.

Since they were so thirsty, they bought some bottled water to drink.

Bob was riding on his green tractor because he needed to plow his field.

Subordinate clauses begin with a subordinating conjunction and are not capable of
standing alone as complete sentences. The subordinating conjunctions in the
Take your S AT prep to the next level. Visit www.learnerator.com
270

271

following examples have been italicized. Subordinate clauses begin with either a
subordinate conjunction or a relative pronoun. In addition, subordinate clauses
make the reader want more information

He shot his rifle because he felt like it. (subordinate conjunction)

He was on the team, which had won every game. (relative pronoun)


Four Types of Sentences
There are four types of sentences: simple, compound, complex, and compound-
complex.
Simple sentences are formed from a single independent clause that stands on its
own. No further information is needed in order to completely understand the idea
that is being asserted by the independent clause.

Jay walks.

The gorilla weighed 800 pounds.

The preacher spoke for several hours.


Compound sentences are formed from two independent clauses that are
connected by a coordinating conjunction. There are only seven coordinating
conjunctions, and they are for, and, nor, but, or, yet, and so. They can be
remembered by using the acronym FANBOYS.
Compound sentences MUST use a comma right before the coordinating conjunction
unless the clauses are very short. This is the most common punctuation mistake in
English. The coordinating conjunctions in the following examples are underlined.

He went the grocery store yesterday, and he went to the zoo last week.

Bob ran all the way home, but he forget his key at work.

The big lion wandered out of its cage, so the zookeeper shot it with a
tranquilizer dart.


Take your S AT prep to the next level. Visit www.learnerator.com
271

272

Complex sentences are formed from an independent clause and a dependent
clause. When the dependent clause comes first, a comma is used to separate the two
clauses. When the independent clause comes first, nothing is used to separate the
two clauses.

Because he likes the taste of ice cream, he eats his desserts very quickly.

He eats his desserts very quickly because he likes the taste of ice cream.


Compound-complex sentences are formed from two independent clauses and a
dependent clause. There are many possibilities and combinations that are possible
with these types of sentences.

He went to the stadium because he wanted to see the game, but he fell asleep
in his chair.

Since he was a model citizen, he picked up the smelly garbage, and he threw
it away in a nearby trash can.

Shifts in Construction
Certain components of a sentence need to remain consistent and parallel. However,
there are inappropriate shifts in tense, voice, mood, person, number, and discourse
that make it hard for the reader to grasp the intended meaning of a sentence. The
most common inappropriate shifts in sentence construction will now be examined.

Tense
Verb tense expresses the time that the action of the verb takes place. If the tense
within the sentence is changed, then the meaning of the sentence will often become
illogical. However, unnecessary shifts in tense are easy to notice and repair.

Incorrect Shift of Tense

Take your S AT prep to the next level. Visit www.learnerator.com


272

273

He caught the fish and then throws in back into the water. (past to present
tense)

She will buy the chicken and cooked it in her oven. (future to past tense)

Correct Repair

He caught the fish and then threw it back into the water. (both past tense)

She bought the chicken and cooked it in her oven. (both past tense)


Voice
A verb can be used to express either active or passive voice. Active voice is when the
subject of the sentence performs the action of the sentence. Passive voice is when
the subject of the sentence does not perform the action of the sentence. When a
sentence uses two verbs they usually need to use the same voice.

Incorrect Shift of Tense

Some people want candy, but sometimes bananas are received. (active to
passive)

When the lady turned up the radio, her favorite song was heard. (active to
passive)

Correct Repair

Some people want candy but sometimes receive bananas. (both active)

When the lady turned up the radio, she heard her favorite song. (both active)


Mood
A verb may also be used to express mood. There are three types of mood: indicative,
imperative, and subjunctive. The errors in mood shift occur when either indicative is
shifted to imperative within a sentence or when imperative is shifted to indicative.

Incorrect Shift of Mood

Take your S AT prep to the next level. Visit www.learnerator.com


273

274

Stand up in the room, and you speak to the audience. (imperative to


indicative)

Take your medicine, and you call your doctor. (imperative to indicative)

Correct Repair

Stand up in the room, and speak to the audience. (both imperative)

Take your medicine, and call your doctor. (both imperative)


Person
There are three perspectives (or points of view) in English. They are first person,
which is the person speaking. The next is second person, which is the person spoken
to. The last one is third person, which is the person spoken about. Most unnecessary
shifts in person usually occur when the third person changes to the second person.

Incorrect shift in person

Individuals like being busy, so when a job is slow, you are sad. (third to
second)

Dogs want dog food, so while they eat, you are satisfied. (third to second)

Correct Repair

You like being busy, so when a job is slow, you are sad. (both second)

Dogs want dog food, so while they eat, they are satisfied. (both third)


Number
An antecedent is a word or a phrase that is represented by a pronoun. One should
use singular pronouns to refer to singular antecedents and use plural pronouns to
refer to plural antecedents. Errors that involve unnecessary shift of number are
usually very easy to detect and repair.

Incorrect shift in number

Take your S AT prep to the next level. Visit www.learnerator.com


274

275

If Janet goes out in the sun, then they need to wear sunscreen. (singular to
plural)

While he rode in the car together, the boys cleaned their shoes. (singular to
plural)

Correct Repair

If Janet goes out in the sun, then she needs to wear sunscreen. (both singular)

While they rode in the car together, the boys cleaned their shoes. (both
plural)


Discourse
Indirect discourse is when dialogue is reported and not quoted. Direct discourse
uses quotation marks and actually quotes what is said. Most errors involving
discourse occur when direct discourse is not enclosed in quotation marks and
appears as indirect discourse.

Incorrect expression of discourse

He stated he was really happy and let's to go to the park. (both indirect)

Correct Repair

He stated he was really happy and said, "Let's go to the park." (indirect and
direct)

He said, "I am really happy. Let's go to the park."(both direct)


Take your S AT prep to the next level. Visit www.learnerator.com


275

276

Pronouns
Pronouns
A pronoun is a word that is substituted for a noun or noun phrase. The word for
which it is used as a substitute is named or understood within the context of the
prose.
There are eight different types of pronouns:

personal - related to a singular entity or group

indefinite - refers to a non-specific item(s), being(s), or location(s)

relative - begins a relative clause that functions as either a noun or an


adjective

interrogative - asks a question

reflexive - refers back to the subject of the sentence

possessive - indicates ownership

demonstrative - distinguishes things that are spoken about


PERSONAL PRONOUNS
This type of pronoun specifies either a person, animal, thing, or idea. A personal
pronoun will use different forms depending on case, number, gender, and
grammatical person.

Number
Singular and plural pronouns are used in order to denote the number of people or
items that are referenced.

I will go fishing with him. (Singular pronouns)

They have seen us going to the river with them. (Plural pronouns)

Gender
Singular pronouns referring to a person will take on different forms in order to
Take your S AT prep to the next level. Visit www.learnerator.com
276

277

denote gender. The pronoun it is gender neutral. Only third person pronouns specify
gender.

She walks with her. (Feminine gender)

He plays with him. (Masculine gender)


Case
Personal pronouns may be either subjective or objective, depending on their use in a
sentence. The subjective form is used as either as a subject of a sentence or as a
subject compliment. The objective form is used either as a direct object (or indirect
object) of a sentence or as the object of a prepositional phrase.

We will go to the aquarium. (Subjective - used as the subject of a sentence)

Bob has left Martha for her. (Objective - object of the preposition for)


Person
Personal pronouns have three different forms to denote the concept of the
grammatical "person":

First person: Person or persons speaking

Second person: Person or persons spoken to

Third person: Person or persons spoken about


Here are some examples of this concept:


We like football. (first person plural - subjective)

You watch football on television. (second person singular or plural -


subjective)

Football annoys her. (third person singular - objective)


Here is a list of the personal pronouns used in English:


Take your S AT prep to the next level. Visit www.learnerator.com
277

278


Subjective Case

First person - I (singular), we (plural)

Second person - you (all forms)

Third person - he (singular masculine), she (singular feminine), it (singular


neutral), they (plural neutral)

Objective Case

First person - me (singular), us (plural)

Second person - you (all forms)

Third person - him (singular masculine), her (singular feminine), it (singular


neutral), them (plural neutral)


INDEFINITE PRONOUNS
This type of pronoun refers to a non-specific item(s), being(s), or location(s). This is
the largest group of pronouns in the English language. Indefinite pronouns can be
easily classified by the types of verbs they use (singular and/or plural).

Nobody knows everything. (singular)

Everybody is at risk. (singular)

Others play with that team. (plural)

Many were selected. (plural)


There are also possessive and compound forms of indefinite pronouns. Possessive
forms end with an apostrophe and the letter s. Compound forms fuse two indefinite
pronouns together so that they may act like a singular entity.

Possessive forms of indefinite pronouns

It could be anybody's fault

It was everyone's responsibility

Compound forms of indefinite pronouns


Take your S AT prep to the next level. Visit www.learnerator.com
278

279

They despised one another.

They loved each other.


Here is a list of the indefinite pronouns used in English:



Singular

no one, nobody, everyone, everybody, someone, somebody, anyone, anybody,


nothing, everything, something, anything, each, another, other, whatever,
whichever, whoever, whomever, enough, little, less, much, more, most, plenty,
one, either, neither

Plural

both, others, none, all, some, any, several, few, fewer, many, more, most


RELATIVE
Relative pronouns begin relative clauses that function as either nouns or adjectives.

He saw the man who was running away from the robber. (adjective clause)

She saw the bag that fell from the robber's hands. (adjective clause)

She despised whomever stood in her way. (noun clause)


In first two examples above, the relative clauses that begin with relative pronouns
serve as adjectives that describe the direct objects of the sentences. In the final
example, the relative pronoun whomever forms a noun clause that functions as the
direct object of the sentence.
There are nine relative pronouns in English:
Adjective clauses: "who", "whom", "which", "what", "that"
Noun clauses: "whoever", "whomever", "whichever", "whatever"

INTERROGATIVE

Take your S AT prep to the next level. Visit www.learnerator.com


279

280

Interrogative pronouns ask a question. There are five interrogative pronouns, and
they are "which", "what", "who", "whom", and "whose". The word who is the
subjective form and the word whom is used as the objective form.

What is she going to do?

Who did that? (subjective form used as the subject)

To whom does that belong? (objective form used as the object of the
prepositional phrase that begins with the word to)

The biggest error with interrogative pronouns is the incorrect usage of the word
who as the objective case. Most individuals are not sure how to use the words who
and whom.

Incorrect usage

The letter was addressed to who? (incorrect usage of subjective form)

To who was the package delivered? (incorrect usage of subjective form)


Correct usage

The letter was addressed to whom? (correct usage of objective form)

To whom was the package delivered? (correct usage of objective form)


REFLEXIVE
These types of pronouns refer back to the subject of the sentence. Every subjective
personal pronoun has a reflexive form. Thus, the reflexive pronouns in English are
"myself", "yourself", "himself", "herself", "itself", "ourselves", "yourselves", and
"themselves". In addition, the pronoun one also has a reflexive form (oneself).

I needed it myself. (first person)

You bought the purse yourself. (second person)

He brought all of that on himself. (third person masculine)


Take your S AT prep to the next level. Visit www.learnerator.com


280

281

Most of the grammatical errors that occur with reflexive pronouns happen when
they are used as the subject instead of referring back to the subject.

Incorrect Usage

Myself is to blame. (incorrect usage of the singular reflexive form)

Ourselves have caused it. (incorrect usage of the plural reflexive form)

Correct Usage

I am to blame. (correct usage of first person singular)

We have caused it. (correct usage of first person plural)


POSSESSIVE
Possessive pronouns indicate ownership. Every personal pronoun has an objective
form. The personal pronouns he and him both use the pronoun his for their
possessive form.
Therefore, the possessive pronouns in English are: "its", "my", "mine", "your", "her",
"hers", "his", "our", "ours", "their", "theirs", and "yours".

We rode in their car. (third person plural - objective)

Your car is black. (second person singular - subjective)


By far, the biggest and most common mistake is the misuse of the possessive
pronoun "its". Quite often, it is confused with the contraction it's, which is a
combination of the word it and is.

Incorrect usage

The green car lost it's wheel. (incorrect usage of contraction)

Its all in a days work. (incorrect usage of pronoun)


Correct usage

The green car lost its wheel. (correct usage of possessive pronoun)

Take your S AT prep to the next level. Visit www.learnerator.com


281

282

It's all in a day's work. (correct usage of contraction)


DEMONSTRATIVE
There are four demonstrative pronouns: "this", "that", "these", and "those". They
stand on their own and distinguish the entity or entities that the speaker is speaking
about.

This is what I mean. (singular)

Who needs those? (plural)


One of the most common mistakes when using demonstrative pronouns is the
incorrect use of the pronouns this and these. The pronoun this is singular, and the
pronoun these is plural. Here are some examples of correct and incorrect usage of
these terms.

Incorrect Usage

These is not what I mean. (incorrect usage of the plural form)

This are not the proper kinds of napkins. (incorrect usage of the singular
form)

Correct Usage

This is not what I mean. (correct usage of the singular form)

These are not the proper kinds of napkins. (correct usage of the singular
form)


Sample Test Questions
Select the answer that correctly identifies the pronoun (or pronouns) in the
sentence.

1) The grumpy old man fell down the stairs and hurt his knee.

Take your S AT prep to the next level. Visit www.learnerator.com


282

283

A. grumpy
B. old
C. fell
D. and
E. his

2) The cats smelled the fish and ran out the door
A. cats
B. and
C. ran
D. out
E. There is no pronoun in the sentence.

3) This is good advice because she really likes to run.
A. This, because
B. This, she
C. because, she
D. she, to
E. she, likes

4) Those are all of the items in the blue basket that they brought to the picnic.
A. Those, all, that
B. Those, in, they
C. all, that, to
D. Those, all, they
E. all, in, they

ANSWERS: 1) E 2) E 3) B 4) D

Take your S AT prep to the next level. Visit www.learnerator.com


283

284

Possessive Determiners
Also referred to as possessive adjectives, possessive determiners assign possession
to a grammatical element that functions as a noun in a sentence.
There are two types of possessive determiners in the English language: weak
possessive pronouns and nouns that end with an apostrophe and the letter "s".

Pronouns As Possessive Determiners
The group of seven weak possessive pronouns form the first group of possessive
determiners. They are called weak because they cannot stand alone like other
pronouns and must have some type of noun to modify.
The possessive pronouns "mine, hers, theirs, ours", and "yours" are considered to be
strong possessive pronouns and stand on their own; they do not modify anything.
Thus, they are NOT possessive determiners.
There are seven weak possessive pronouns that function as possessive determiners
in the English language:

"my" - first person singular

"your" - second person singular and plural

"his" - third person singular (masculine)*

"her" - third person singular (feminine)

"its" - third person singular (neutral)

"our" - first person plural

"their" - third person plural


*The pronoun "his" may function as either a strong or weak possessive pronoun.
There is also another pronoun that functions as a possessive determiner; it is the
pronoun "whose", and it is interrogative in nature.

What Weak and Strong Possessive Pronouns Imply
The weak possessive pronouns always imply the definite article "the" because they
Take your S AT prep to the next level. Visit www.learnerator.com
284

285

refer to specific things and do not imply a sense of generality. The weak possessive
pronouns are found in both the subject and the predicate of the sentence.
In contrast, the strong possessive pronouns can be used to imply either the
indefinite article "a" or the definite article "the". They are primarily found in the
predicate of the sentence and are usually used as subject compliments but may also
be used as subjects; however, this second type of usage is not very common.
These two types of possessive pronouns may be modified by the adverbs "more, less,
as, much, as much", and "mostly".
Here are some examples of strong and weak pronouns:

1. Randy was her friend. (weak possessive pronoun in the predicate)
2. Her friend was Randy. (weak possessive pronoun in the subject)
3. Randy was a friend of hers. (strong possessive pronoun in the predicate)
4. Hers was a gift that belonged to Randy. (strong possessive pronoun in the
subject)

In the first two examples, the pronoun "her" implies the definite article "the" and
makes Randy her specific friend. In the third example, the pronoun "hers" implies
the indefinite article "a" and makes Randy one of many friends.
While the usage of a strong possessive pronoun as the subject in the last sentence is
grammatically correct, it is a type of sentence construction that is not used very
often in English.
Here are some examples of weak possessive pronouns used as possessive
determiners:

1. My car is a very good car.
2. Their car is more her car than it is his.
3. Whose car is it?

In the first example, the word "my" is used as a possessive determiner to modify the
word "car". In the second example, the word "their" is used as a possessive
Take your S AT prep to the next level. Visit www.learnerator.com
285

286

determiner to modify the word car, and the word "her" is used to modify the second
usage of the word "car"; the adverb "more" is then used to modify the pronoun "her".
The pronoun "his" is used as a strong possessive pronoun and stands alone;
therefore, it is NOT a possessive determiner. In the final example, the word "whose"
is used as an interrogative possessive determiner that modifies the word "car".

Common Grammatical Errors
Many common errors in the English language occur because of the misuse of the
words "their" and "there". The word "their" is a possessive determiner, and the word
"there" is an adverb; these two words are constantly confused with each other.
These two words are also confused with the contraction "they're", which is a
reduction of the phrase "they are".
Here are several examples of common errors with these words:

Incorrect Usage

It's not there fault. (adverb used as a determiner)

She cannot tell when their lying. (determiner used as a contraction)

The two boys did not want to do there chores. (adverb used as a determiner)

Jack wants to go and see what there doing. (adverb used as a contraction)

Correct Usage

It's not their fault. (correct use of determiner)

She cannot tell when they're lying. (correct use of contraction)

The two boys did not want to do their chores. (correct use of determiner)

Jack wants to go and see what they're doing. (correct use of contraction)


Nouns as Possessive Determiners
Nouns can be used as possessive determiners when they are spelled with an
apostrophe and the letter "s". However; if a noun already ends with the letter "s"
because it is plural, then only an apostrophe is used to denote possession.
Here are some examples:
Take your S AT prep to the next level. Visit www.learnerator.com
286

287

That is John's house.

The swans like to bathe in the pond's fragrant waters.

The babies' beds are all in a row.

The lions' food source is almost gone, and soon they will grow hungry.

These types of possessive determiners are very similar to the weak possessive
pronouns that are used as possessive determiners. They always imply the definite
article "the" because they always refer to specific things. In addition, they may also
be modified with adverbs in the same way that the weak possessive pronouns are.

Common Errors
Many mistakes happen when the apostrophe is not placed correctly. Quite often, the
word is misspelled and/or the meaning of the word is unintentionally changed.

Incorrect Usage

The mariachi band loved Johns' generous tip. (incorrect plural)

They destroyed the walruses's habitat. (incorrect plural)

The old mans' dog ate the kitten's food before they could finish it. (incorrect
singular and plural)

Take your S AT prep to the next level. Visit www.learnerator.com


287

288

Correct Usage

The mariachi band loved John's generous tip. (correct singular)

They destroyed the walruses' habitat. (correct plural)

The old man's dog ate the kittens' food before they could finish it. (correct
plural and singular)


Sample Test Questions
Select the answer that correctly identifies the possessive determiner (or
determiners) in the sentence.

1) That was her idea of a great artistic presentation.
A. that
B. was
C. her
D. of
E. great

2) Nobody thought it was John's idea.
A. nobody
B. thought
C. was
D. John's
E. There is no possessive determiner in the sentence.

Take your S AT prep to the next level. Visit www.learnerator.com


288

289

3) His idea was that she should try Jane's recipe and not hers.
A. his, she
B. was, she
C. she, hers
D. that, hers
E. his, Jane's

4) Its main flaw was that her idea was not as good as his.
A. its, his
B. was, his
C. her, his
D. its, her
E. that, not

5) Their solution was a combination of Jane's recipe and John's idea.
A. their, was, Jane's
B. was, Jane's, John's
C. their, Jane's, John's
D. their, was, and
E. was, and, idea

Solutions
1) C - The word "her" is a weak possessive pronoun that functions as a possessive
determiner and modifies the word "idea".
2) D - The word "John's" is a possessive determiner formed from the proper noun
"John"; this possessive determiner modifies the word "idea".
3) E - The word "his" is a weak possessive pronoun that functions as a possessive
determiner and modifies the word "idea". The word "Jane's" is a possessive
determiner formed from the proper noun "Jane"; this possessive determiner
modifies the word "recipe". The word "hers" at the end of the sentence is a strong

Take your S AT prep to the next level. Visit www.learnerator.com


289

290

possessive pronoun in the predicate and stands alone; thus, it is NOT a possessive
determiner.
4) D - The word "its" is a weak possessive pronoun that functions as a possessive
determiner and modifies the word "flaw". The word "her" is a weak possessive
pronoun that functions as a possessive determiner and modifies the word "idea".
The word "his" at the end of the sentence is a strong possessive pronoun in the
predicate and stands alone; thus, it is NOT a possessive determiner.
5) C - The word "their" is a weak possessive pronoun that functions as a possessive
determiner that modifies the word "idea". The word "Jane's" is a possessive
determiner formed from the proper noun "Jane"; this possessive determiner
modifies the word "recipe". The word "John's" is a possessive determiner formed
from the proper noun "John"; this possessive determiner modifies the word "idea".

Take your S AT prep to the next level. Visit www.learnerator.com


290

291

Grammatical Agreement
In English, there are three major types of agreement that are required in order to be
grammatically correct:

Agreement of pronoun and its antecedent

Agreement of subject and verb

Agreement of nouns


Agreement of Pronoun and Antecedent
The antecedent of a pronoun is the person, thing, location, or idea in a sentence to
which a pronoun refers. The antecedent may be either a noun or another pronoun.
Therefore, a pronoun must agree both in number and gender with its antecedent in
order for a sentence to be grammatically correct.

Agreement of number
A singular pronoun is used with a singular antecedent, and a plural pronoun is used
with a plural antecedent. Here are several examples:

1. The dog thought that its bone was in the backyard.
2. The kittens ate all of their food in the kitchen.
3. The dog and the cat were glad that they were friends.

In the sentences above, the number of each pronoun agrees with its antecedent. In
the first example, the possessive pronoun its is singular because its antecedent is the
singular noun dog. In the second example, the possessive pronoun there is plural
because its antecedent is the plural noun kittens.
In the final example, The plural antecedent of the plural pronoun they is a compound
noun formed from the nouns dog and cat. In this case, the compound noun takes a
plural pronoun. This brings us to our next topic, which is compound nouns.

Take your S AT prep to the next level. Visit www.learnerator.com
291

292

Compound nouns
Compound nouns are created from two nouns, two pronouns, or a noun and a
pronoun; the two elements of a compound noun are connected by the conjunctions
and, or, or nor. When the two words are connected by the conjunction and, a plural
pronoun is used.

When the two words are connected by the conjunction or or nor, the element of the
compound noun that is closest to the pronoun determines whether a singular or a
plural pronoun is used.

1. Either the dogs or I will eat my food.
2. Either the duck or the pigs will eat their food.
3. The turkey and the chicken will drink their water.

The compound nouns in the examples are in agreement with the pronouns that are
using them as antecedents. In the first example, the compound noun consists of a
plural noun and a singular pronoun connected by the conjunction or; therefore,
since the singular pronoun I is closer to the pronoun that is using the compound
noun as an antecedent, the singular form is used.
In the second example, a singular noun and a plural noun are connected by the
conjunction or; thus, the pronoun using the compound noun as an antecedent is
plural since the plural noun is closer to the antecedent.
In the last example, two singular nouns are connected by the conjunction and;
therefore, the pronoun using the compound noun as an antecedent will be plural.

Collective Nouns
Collective nouns are singular nouns that are used to identify various kinds of
groups. When a collective noun is used as an antecedent by a pronoun, the singular
form of the pronoun is used.

1. The team won its game.
Take your S AT prep to the next level. Visit www.learnerator.com
292

293

2. The herd began its stampede.
3. A colony of ants was about to meet its doom.

In all three examples, collective nouns are used as antecedents by singular
pronouns.

Common Errors
Most of the errors with compound and collective nouns occur when an incorrect
pronoun is chosen to be the antecedent.

Incorrect Usage

The cats eat its food. (plural antecedent with singular pronoun)

That pair of animals eats their food. (singular collective noun with plural
pronoun)

The mice are eating its food. (plural noun with singular pronoun)

Correct Usage

The cats eat their food. (plural antecedent with plural pronoun)

That pair of animals eats its food. (singular collective noun with singular
pronoun)

The mice are eating their food. (plural noun with plural pronoun)


Agreement of Gender
A feminine pronoun is used with a feminine antecedent, and a masculine pronoun is
used with a masculine antecedent. The same rules for compound nouns apply,
depending on whether or not the two words are connected by the conjunction and
or by the conjunctions nor and or.

1. William likes to think that he is always right.
2. Mary knows in her heart that William is always wrong.
3. William and Mary do not think that they have a good relationship.
Take your S AT prep to the next level. Visit www.learnerator.com
293

294

4. Does either Mary or William think that he has a good relationship.

In the above sentences, the gender of each pronoun agrees with its antecedent. In
the first example, the pronoun he is masculine because its antecedent is William, and
he is a male.
In the second example, the possessive pronoun her is feminine because its
antecedent is Mary, and she is a female. In the third example, the plural pronoun
they is gender neutral.
In the last example, the compound noun because it has a compound noun connected
by the conjunction or, so the pronoun using the compound noun as its antecedent
uses the proper name William (the element of the compound noun closer to the
pronoun) to determine its number and gender.

Indefinite Pronouns
Indefinite pronouns refer to things in a non-specific manner. Indefinite pronouns
may be singular or plural (some indefinite pronouns may be used either way) when
used as antecedents within a sentence.

The following indefinite pronouns use singular antecedents:

no one, nobody, everyone, everybody, someone, somebody, anyone, anybody,


nothing, everything, something, anything, each, another, other, whatever,
whichever, whoever, whomever, enough, little, less, much, more, most, plenty,
one, either, neither


These pronouns use plural antecedents:

both, few, many, others, several


These pronouns may use either singular or plural antecedents:

all, any, more, most, none, some


Take your S AT prep to the next level. Visit www.learnerator.com
294

295

Here are a few examples of indefinite pronouns being used as antecedents:

1. All of the monkeys were let out of their cage.
2. Everything finds its place.
3. Both of them know their purpose.

In the first example, the indefinite pronoun all is used in its plural form as the
antecedent of the indefinite pronoun. In the second example, the indefinite pronoun
everything is used as the singular antecedent of the pronoun its. In the final
example, the plural indefinite pronoun both is used as the antecedent of the
pronoun their.

Controversial usage of plural third person to insure gender neutrality
In the late 20th century, various writers began to change the practice of using the
masculine singular third person pronoun as the antecedent for singular indefinite
pronouns and substituted the neutral plural third person instead.
By doing this, there would be no gender discrimination. In other words, the pronoun
his was paired with the singular indefinite pronouns, and certain individuals began
to substitute it with the pronoun their. Others would use the words his or her.

Everybody knew his place. (older style - instantly defaults to masculine


pronoun)

Everybody knew their place. (modern style - uses plural pronoun instead)

Everybody knew his or her place. (modern style - uses masculine and
feminine)


All three of these forms are considered correct by different entities. However, the
first example is being shunned by more and more people as time progresses.

Take your S AT prep to the next level. Visit www.learnerator.com


295

296

Subject and Verb Agreement
In English, the subject and verb must agree in number. Thus, a singular subject
requires a singular verb and a plural subject requires a plural verb.

1. Jack throws the knife at the wall.
2. Jack's friends throw him out of the pub.
3. Everyone is throwing a fit.

In the first example, a singular subject (Jack) uses a singular verb. In the second
example, a plural subject (friends) is using a plural verb. In the final example, a
singular subject (the singular indefinite pronoun everyone) is using a singular verb.
In addition, the rules for compound subjects used as antecedents for pronouns are
identical to those used for subject and verb agreement. Therefore when the two
elements of a compound subject are connected by the conjunction and, they require
a plural verb.

Likewise, when the two elements of a compound subject are connected by the
conjunction or (or the conjunction nor), the element that is closer to the verb
determines the number to be used.

1. The pony and the dog no longer run through the fields
2. The pony nor the dogs run through the fields.
3. The ponies nor the dog runs through the fields.

In the first example, the compound subject is connected with the conjunction
and uses a plural verb. In the second example, the elements of the compound
sentence are connected with the conjunction nor; therefore, the word closer
to the verb is used to determine number. The same is true for the third
example.

Take your S AT prep to the next level. Visit www.learnerator.com


296

297

Common Errors
The most popular errors that occur are when there is a noun or pronoun of a
different number that is placed between the subject and the verb. This sort of
occurrence is most common with prepositional phrases beginning with the
preposition of. The verb is erroneously matched with the other word that is closer.

Incorrect Usage

One of the fryers are broken. (singular subject with plural verb)

John's friends standing by the policeman is yelling. (plural subject with


singular verb)

The candy that she put in the jars are gone. (singular subject with a plural
verb)

Correct Usage

One of the fryers is broken. (subject and verb are singular)

John's friends standing by the policeman are yelling. (subject and verb are
plural)

The candy that she put in the jars is gone. (subject and verb are plural)


Agreement of Nouns
When a sentence uses a form of the verb be as a linking verb, the subject and the
subject compliment must agree in number and gender.

1. James is a big man.
2. John was a very small individual.
3. These men were regular sized people.
4. This lady is a highly intelligent professor.
In the first example, a singular masculine subject is paired with a singular masculine
subject compliment. In the second example, a singular masculine subject is paired
with a singular neutral subject compliment. In the third example, a plural masculine

Take your S AT prep to the next level. Visit www.learnerator.com


297

298

subject is paired with a plural neutral subject compliment. In the last example, a
singular feminine subject is paired with a singular neutral subject compliment.

Common Errors
Most errors occur when the subject compliment is a noun that is used or spelled
incorrectly. This sort of error can easily be spotted and corrected without much
difficulty.

Incorrect Usage
1. Greg is a lady. (feminine subject compliment with masculine subject)
2. Frank is a cowards. (plural subject compliment with singular subject)
3. Samoans are big peoples. (incorrect spelling of plural noun)

Correct Usage
1. Greg is a man. (gender agreement between nouns)
2. Frank is a coward. (number agreement between nouns)
3. Samoans are big people. (correct spelling of noun)

Sample Test Questions
In the following questions, select the answer that contains an INCORRECT usage of
agreement:

1)
A. They love their team.
B. He loves his team.
C. We loves his team.
D. I love our team.

2)
A. Greg drinks from his cup.
B. Greg and I drink from our cups.
Take your S AT prep to the next level. Visit www.learnerator.com
298

299

C. I drink from my cup.
D. You drinks from your cup.

3)
A. The men and the boy are playing golf.
B. The boys or the men are playing golf.
C. The men or the boy is playing golf.
D. The boy or the men is playing golf.

4)
A. One of the ducks is walking.
B. One of the ducks are walking.
C. The ducks are walking.
D. The duck is walking.

Answers
1. C) This sentence is incorrect because it has a plural pronoun as its subject
and uses a singular verb.
2. D) This sentence is incorrect because it has a second person pronoun and
uses the incorrect verb tense.
3. D) This sentence is incorrect because it has a compound subject with a
singular noun and a plural noun connected by the conjunction or. Since the
plural noun men is closer to the verb, a plural verb must be used instead.
4. B) This sentence is incorrect because it has the singular indefinite pronoun
one for its subject. This singular subject requires a singular verb. The
confusion in this example is the result of the prepositional phrase that
contains the plural noun ducks. Since this plural noun directly precedes the
verb are, it tricks the writer into thinking that a plural verb should be used.

Take your S AT prep to the next level. Visit www.learnerator.com


299

300

Frequently Confused Words


1) Affect - Effect
Affect - to influence (verb); an emotional response (noun)

She did not want to affect his decision. (most common usage)

He displayed a happy affect. (used as a noun primarily in psychology--very


rare)

Effect - a result (noun); to cause (verb)

It did not have much of an effect. (most common usage)

He wanted to effect change within the system. (used as a verb--not as


common)


2) Advice - Advise
Advice - a recommendation (noun)

He gave her some good advice.

She did not like his advice.


Advise - to recommend (verb)

They did not advise him about the situation.

John would advise him about the problem.


3) Complement - Compliment
Complement - something that completes (noun); to complete something else (verb)

The purse was a perfect complement to her outfit. (noun)

The table and the chair complement each other well. (verb)

Compliment - praise, flattery (noun); to say nice things about (verb)

He appreciated her compliment. (noun)

She complimented his performance (verb--past tense)


Take your S AT prep to the next level. Visit www.learnerator.com
300

301

4) Council - Counsel
Council - governing body (noun)

The council met every month.

She was a member of the council.


Counsel - advice (noun); a lawyer or lawyers (noun); to give advice (verb)

He sought the counsel of his teacher (noun--advice)

Jane is serving as counsel for the defendant. (noun--lawyer)

All counsel must obey the rules of the court system. (noun--lawyers)

He tried to counsel her. (verb)


5) Discreet - Discrete
Discreet - prudent (adjective); modest (adjective); unobtrusive (adjective)

He was very discreet and only said what was required. (adjective--prudent)

She was impressed by the discreet elegance of the home. (adjective--modest)

He followed at a discreet distance. (adjective--unobtrusive)


Discrete - separate, distinct (adjective)

The book has several discrete sections.

That thing has many discrete parts.


6) Eminent - Imminent - Immanent
Eminent - prominent (adjective)

She was an eminent artist.

There are many eminent doctors in the city.


Imminent - about to happen (adjective)

The situation was imminent.

He was in imminent danger.

Immanent - inherent (adjective)

Beauty is not something acquired; it is immanent.

Take your S AT prep to the next level. Visit www.learnerator.com


301

302

Kindness is immanent in all people.


7) Farther - Further
Farther - at a greater distance (adjective)

She threw the ball farther than he did.

He ran farther down the trail.


Further - in greater depth (adjective)

This requires further analysis.

They needed further detail.


8) Its - It's
Its - possessive form of the word it (pronoun)

The whale lost its way.

The monkey found its cage.


It's - contraction for the clause it is (pronoun and verb)

It's clear to see that he didn't have a clue.

It's time for tea.


9) Principal - Principle
Principal - foremost (adjective); administrator of a school (noun)

The tomato was the principal ingredient in the stew. (adjective)

She was the principal of the local elementary school. (noun)


Principle - moral rule (noun); basic truth (noun)

She was a woman of principle. (noun--moral rule)

It was a principle of human behavior. (noun--basic truth)


10) Than - Then
Than - in comparison to (conjunction, preposition)
Take your S AT prep to the next level. Visit www.learnerator.com
302

303

He is older than you are. (conjunction)

He is older than you. (preposition)


Then - next (adverb); that time (noun)

He then traveled all over the world. (adverb)

Since then, she's been more prudent. (noun


11) Their - There - They're
Their - possessive form of the pronoun they (pronoun)

It was their decision.

Their goal was to succeed.


There - in that place (adverb); introduces a sentence (pronoun); that place (noun);
relied on for support (adjective)

Stand over there. (adverb)

There was a time. (pronoun)

She took it from there. (noun)

She was always there for him. (adjective)


They're - contraction for the clause they are. (contraction)

They're bad.

They're good.


12) To - Too
To - toward; begins an infinite phrase (preposition)

He went to the movies. (preposition--toward)

Bill went to see her house. (preposition--infinitive)


Too - also; very (adverb)

They wanted to see her, too. (adverb--also)

She was too smart for them. (adverb--very)

Take your S AT prep to the next level. Visit www.learnerator.com


303

304


13) Whose - Who's
Whose - interrogative possessive determiner denoting ownership (pronoun)

Whose wallet is that?

Randy did not know whose purse that was.


Who's - contraction for the clause who is (contraction)

Who's at the door?

She wants to see who's who.


14) Your - You're
Your - second person possessive (pronoun)

Your car is green

I do not like your green car.


You're - contraction for the clause you are (contraction)

You're dumb.

You're falling.


15) Capital - Capitol
Capital - assets (noun); related to assets (adjective); upper case (adjective); major
city (noun)

The business needed more capital. (noun--assets)

He paid taxes on his capital gains. (adjective--assets)

Start the sentence with a capital letter. (adjective--upper case)

The capital of Japan is Tokyo. (noun--major city)


Capitol - government building (noun or adjective)

She went to the capitol building. (adjective)

He works at the Texas State Capitol. (noun)


Take your S AT prep to the next level. Visit www.learnerator.com
304

305

16) Die - Dye
Die - to lose life (verb); a cube that functions as a random number generator (noun)

Tyrone did not want to die. (verb)

Rudy rolled the die. (noun)


Dye - to change or add color by applying liquid dye (verb)

Tyrone did not want to dye his shirt blue.

Rudy was out of dye.


17) Gorilla - Guerilla
Gorilla - animal in ape family (noun)

The gorilla was small.

The gorilla was hungry.


Guerrilla - a type of soldier (noun); indicative of a guerilla (adjective)

The guerilla was small. (noun)

The guerilla soldier was hungry. (adjective)


18) Knew - New
Knew - to have been familiar with (verb)

The girl knew the dog.

The dog knew the girl.


New - fresh, not yet old (adjective)

Randy wore a new shirt.

His new girlfriend bought it for him.


19) Maybe - May be
Maybe - perhaps (adverb)

Maybe, she would be interested.

John might be interested, maybe.

Take your S AT prep to the next level. Visit www.learnerator.com


305

306


May be - might be (verb)

She may be interested.

John may be interested.


20) Personal - Personnel
Personal - intimate (adjective); owned by a person (adjective)

It was a very personal question. (adjective--intimate)

Those were his personal belongings. (adjective--owned by a person)


Personnel - employees (collective noun)

He fired all of his personnel.

The store's personnel were all given Christmas bonuses.


21) Plain - Plane
Plain - simple (adjective)

It was plain to see

That was plain enough.


Plane - to shave wood (verb); aircraft (noun)

It was time for the carpenter to plane the board.

It was a plane up in the sky.


22) Presence - Presents
Presence - attendance (noun); being at hand (noun)

His presence was requested. (noun--attendance)

Her presence was felt by all. (noun--being at hand)


Presents - gifts (noun)

George received lots of presents.

Bill gave George many presents.

Take your S AT prep to the next level. Visit www.learnerator.com


306

307


23) Stationary - Stationery
Stationary - standing still (adjective)

Tom was in a stationary position.

Tina was stationary.


Stationery - writing paper (noun)

The stationery was pink with red trim.

She ran out of stationery, so she didn't write all of her letters.


24) Through - Threw
Through - finished (adjective); into and out of (adverb)

He was through. (adjective)

He went through the field. (adverb)


Threw - toss (verb)

He threw the red ball.

She threw it back at him and knocked him over.


25) Waist - Waste
Waist - midsection of the body (noun)

He had a slim waist.

Her waist was much slimmer.


Waste - discarded material (noun); to squander (verb)

The garbage truck was full of waste. (noun)

They did not want to waste their money. (verb)


Sample Questions
In the questions below, identify the sentence that is correct:
1)
Take your S AT prep to the next level. Visit www.learnerator.com
307

308

A) He threw the stationery into the trash.
B) He threw the stationary into the trash.
C) He through the stationary into the trash.
D) He through the stationery into the trash.
2)
A) The gorilla was flying in the knew plane that was taking it to the zoo.
B) The guerilla was flying in the new plain that was taking it to the zoo.
C) The guerilla was flying in the knew plane that was taking it to the zoo.
D) The gorilla was flying in the new plane that was taking it to the zoo.
3)
A) There problem is that their not multiplying they're numbers correctly.
B) They're problem is that their not multiplying there numbers correctly.
C) There problem is that they're not multiplying their numbers correctly.
D) Their problem is that they're not multiplying their numbers correctly.
E) Their problem is that they're not multiplying there numbers correctly.
4)
A) Its not plain too see the affect of your presents.
B) Its not plane to see the effect of you're presence.
C) Its not plain to see the affect of your presence.
D) It's not plane too see the effect of your presents.
E) It's not plain to see the effect of your presence.
F) It's not plain to see the affect of your presence.

Solutions
1. A) The word threw must be a verb, and the word stationary must be a noun.
2. D) The animal flying to the zoo is a gorilla and not a guerilla soldier. Also, the
word new must be an adjective, and the word plane must be a noun.
3. D) The word at the beginning of the sentence must be a pronoun, and the
word before the word not must be a contraction of the clause they are. In
addition, the word before the word numbers must also be a pronoun.

Take your S AT prep to the next level. Visit www.learnerator.com


308

309

4. E) The word at the beginning of the sentence must be a contraction of the
clause it is, and the third word of the sentence must be an adjective. Also, the
word before the word of must be a noun that means "a result." In addition,
the last word of the sentence must be a noun that means "being at hand."

Take your S AT prep to the next level. Visit www.learnerator.com


309

310

Logical Comparisons
Logical comparisons that occur between two or more items must have these three
characteristics in order to be correct:

completeness

consistency

clarity


Completeness
When making any type of logical comparison, there must be two things that are
compared. The most popular error with this aspect of logical comparison occurs
when a claim of comparison is made and only one element of the comparison is
mentioned in the sentence.

Incorrect Usage
1. The diet product contains 33% less fat.
2. The rock is ten pounds lighter.

Correct Usage
1. The diet product contains 33% less fat than the regular product.
2. The rock is ten pounds lighter than the brick.

In the above examples, only one item is mentioned in a statement that is attempting
to make a logical comparison. Therefore, another item must be included so that a
true logical comparison is correctly made.

Incorrect Usage
1. Mary walked so slow.
2. John swam so fast.

Take your S AT prep to the next level. Visit www.learnerator.com


310

311


Correct Usage
1. Mary walked so slow that she was late for work.
2. John swam so fast that he won the race.

In the above examples, the word so needs to be qualified in order to have logical
comparison. Both sentences can be corrected by comparing speeds of movement
with specific events.

Consistency

Similarity
The items being compared need to have some type of similarity that allows for a
logical comparison to be made so that consistency is maintained.

Incorrect Usage
1. The chips at the corner store are cheaper than the supermarket.

Correct Usages
1. The chips at the corner store are cheaper than the supermarket's chips.
2. The chips at the corner store are cheaper than the chips at the supermarket.
3. The chips at the corner store are cheaper than those at the supermarket.

In the incorrect example above, chips are being compared to a supermarket.
Therefore, the sentence may be corrected if the chips are compared to other chips.
The three correct usages present different ways that this correction can be made.

Specificity
The sentence must be specific with the exact elements that are being compared.
Quite often, the first element mentioned is also included in the second element.

Take your S AT prep to the next level. Visit www.learnerator.com


311

312

Therefore, the writer needs to be specific about the various elements that are being
compared in order for the logical comparison to be consistent.

Incorrect Usage
1. The boy runs faster than anyone.

Correct Usage
1. The boy runs faster than anyone else.
2. The boy runs faster than any one of the others.
3. The boy runs faster than any one of the other boys.

In the above example, the boy is being compared to himself because he is part of the
universal pronoun anyone. This illogical comparison occurs because the terms in the
sentence are not providing an adequate amount of specificity. Thus, all correct
usages need to be more specific in order to remedy this situation. This type of repair
is usually made by including the words else or other(s).

Clarity
There are also many errors with logical comparison that occur because of clarity
issues. These types of problems are the result of sentences that lack precision.
Oftentimes, an ambiguous meaning is not meant but is expressed because the
correct words are not employed.

Incorrect Usage
1. She gave her best friend more candy than her mother.

Correct Usages
1. She gave her best friend more candy than her own mother did.
2. She gave her best friend more candy than she gave her own mother.
3. She gave her best friend more candy than her friend's mother did.

Take your S AT prep to the next level. Visit www.learnerator.com
312

313

The incorrect usage in the example above demonstrates how the lack of
precision causes ambiguity and affects the clarity of the sentence. Thus, the
logical comparison is uncertain because is unclear which things are being
compared. The correct usages use specific terminology in order to precisely
clarify the meaning of the sentence.

Sample Test Questions
In the questions below, please select the answer that contains an INCORRECT usage
of logical comparison.

1)
A) The diet candy contains half the amount of calories than the regular candy
contains.
B) The apple pie is better than all of the others.
C) He ran faster than anyone.
D) She baked her cookies more quickly than everyone else.
2)
A) She cried so hard that her tears dripped onto her shirt.
B) They ran so fast that they fell.
C) The very large boy in the blue shirt ran so fast.
D) The tiny dog walked so fast that it quickly lost sight of its owner.
3)
A) The grapes at the farmers' market were more expensive than the store.
B) The grapes at the farmers' market were more expensive than the store's
grapes.
C) The grapes at the farmers' market were more expensive than the grapes at
the store.
D) The grapes at the farmers' market were expensive.
4)
A) Linda typed faster than her mother.
B) Linda's mother typed faster than anyone.
Take your S AT prep to the next level. Visit www.learnerator.com
313

314

C) Linda's mother typed faster than anyone else her age.
D) They both typed faster than all of the others in the room.
E) All of the sentences are correct.

Solutions
1. C) The subject of the sentence ran faster than anyone. However, the subject is
included in the pronoun anyone. Since the subject cannot run faster than
himself, it is logically incorrect.
2. C) The subject of this sentence ran so fast that something happened.
However, the sentence does not explain what this something is; therefore, it
is logically incorrect.
3. A) The grapes are being compared to the store. This unintended comparison
is illogical; thus, the sentence contains an incorrect usage of comparison.
4. B) Linda's mother is included in the word anyone; therefore, an incorrect
usage of logical comparison has occurred.

Take your S AT prep to the next level. Visit www.learnerator.com


314

315

Conventional Expressions
There are many conventional expressions that are inconsistent with standard
written English. While we are not going to cover them all here, several of these
incorrect expressions are given are given below along with their proper corrections.

1) For all intensive purposes For all intents and purposes
This phrase is commonly misused when a writer wishes to express an idea meaning
that in every practical sense, something has occurred. The misuse of this phrase
means that the purposes with which something has occurred were extremely
intense.

Incorrect Usage
For all intensive purposes, the marriage was over.

Correct Usage
For all intents and purposes, the marriage was over.

In the incorrect example above, the writer is expressing that the purposes
with which the marriage had ended were very intense. In the correct
example, the writer is expressing the intended notion that for all practical
purposes, the marriage was over.

2) I could care less I couldnt care less
This phrase is misused when a writer tries to express the idea of apathy, but instead
expresses the idea of caring for something and then being capable of reducing the
amount of this concern.

Incorrect Usage
I could care less if she loves him.

Take your S AT prep to the next level. Visit www.learnerator.com
315

316

Correct Usage
I couldnt care less if she loves him.

In the incorrect example above, the writer actually does care that she loves him;
however, the writer is able to reduce the amount of this care to a lesser degree. In
the correct example, the writer has absolutely no concern about whether or not she
loves him.

3) Its a doggy-dog world Its a dog-eat-dog world.
This phrase is misused when the writer wishes to express the fact that the world is
enormously competitive but instead expresses the unusual notion that the world is
like that of a tiny little dog.

Incorrect Usage
My father always told me that its a doggy-dog world out there.

Correct Usage
My father always told me that its a dog-eat-dog world out there.
In the incorrect usage above, the father of the writer has informed his child that the
world is similar to a cute, little puppy. In the correct example, the father of the
writer has informed his child that the world is a very competitive place because
friends and colleagues will often betray or destroy each other in order to get ahead

4) Nip it in the butt Nip it in the bud
This phrase is misused when a writer wishes to express that it is time to put
something to an end but instead states that it is time to literally bite the
hindquarters of something.

Incorrect Usage
She saw the way that she came on to him and decided it was time to nip it in
the butt before he fell prey to her amorous advances.
Take your S AT prep to the next level. Visit www.learnerator.com
316

317


Correct Usage
She saw the way that she came on to him and decided it was time to nip it in
the bud before he fell prey to her amorous advances.

In the incorrect example, she decided to bite another ladys flirtation in its figurative
hindquarters before he fell for this ladys advances. In the correct example, she
decided to put an end to another ladys flirting before he fell for this ladys advances.

5) Statue of Limitations Statute of Limitations
This phrase is misused when a writer wishes to express that there is a deadline for
filing a law suit but instead states that there is a stone carving that is somehow
shaped like limitations (whatever that shape may be). The writer could also be
referring to a statue created by an artist named Limitations.
Incorrect Usage
The statue of limitations has expired.
Correct Usage
The statute of limitations has expired.
In the incorrect example, the writer is expressing an illogical notion that somehow a
stone carving has expired. In the correct example, the writer is stating that a
criminal charge can no longer be legally prosecuted because the deadline to file a
lawsuit has passed.

Take your S AT prep to the next level. Visit www.learnerator.com


317

318

6) One in the same One and the same
This phrase is most often misused when the writer wishes to express the concept
that two things are alike but instead expresses the illogical notion that something is
contained within an object referred to as the same.

Incorrect Usage
They are one in the same.

Correct Usage
They are one and the same.

In the incorrect example above, multiple entities are united in some form or fashion
while they are inside of an object known as the same. In the correct example, two
people, two animals, or two things are actually the same.

7) Got off scott free Got off scot free
This phrase is most often misused when the writer wishes to express the concept
that someone has escaped punishment for a wrong doing or misdeed but instead
claims that someone charged nothing to get off of an individual named Scott.

Incorrect Usage
He got off scott free.

Correct Usage
He got off scot free.

In the incorrect example above, the writer states that a man removed himself from
an individual named Scott for no charge. In the correct example, the writer states
that a man has escaped punishment for a crime.

8) Windshield Wind chill
Take your S AT prep to the next level. Visit www.learnerator.com
318

319

A windshield is a large piece of glass placed near the front of an automobile. The
wind chill is the perceived decrease in the temperature felt by the human body.

Incorrect Usage
The windshield factor was 30 degrees.

Correct Usage
The wind chill factor was 30 degrees.

In the incorrect example above, an illogical concept is expressed. In the correct
example above, an expression of the perceived outdoor temperature is asserted.

9) Irregardless Regardless
The word "irregardless" is not accepted by most teachers of English; however, it is
used a lot in common speech of an informal nature. It is a type of double negative
that is intended to mean without regard; however, the term "regardless" means
without regard, so the prefix ir- is unnecessary.

Incorrect Usage
One should use words correctly, irregardless of how other people may use
them.

Correct Usage
One should use words correctly, regardless of how other people may use
them.

In the incorrect example above, the word irregardlessis used and could be viewed as
a type of double negative. In the correct example above, the word regardless is used
and the sentence is grammatically and logically perfect.

10) Literally - Figuratively
Take your S AT prep to the next level. Visit www.learnerator.com
319

320

The word literally is often used when the word figuratively is meant. Oftentimes, the
error caused by this type of usage can be repaired by simply removing the word
literally.

Incorrect Usage
She literally drowned herself in a river of tears.

Correct Usage
She drowned herself in a river of tears.

In the incorrect example above, the writer is stating that an individual actually
drowned in a real river that was created by tears. This would also imply that this
person died--literally. However, in the correct usage, the river of tears can be
perceived as being metaphorical; therefore, the actual meaning of the sentence is a
creative expression of a person crying so hard that an above average amount of
tears were shed in the process.

Sample Test Questions
In the following questions; please select the answer that CORRECTLY expresses the
idea that is intended by the conventional expressions used.

Take your S AT prep to the next level. Visit www.learnerator.com


320

321

1)
A) The man was literally thrown to the wolves.
B) The windshield factor was 20 below zero.
C) They stood together, irregardless of the outcome.
D) Scott got off scot free.

2)
A) For all intents and purposes, I could care less.
B) For all intensive purposes, I could care less.
C) For all in tents and porpoises, I couldnt care less.
D) For all intents and purposes, I couldnt care less.

3)
A) You need to nip it in the bud, because its a doggy-dog world.
B) You need to nip it in the bud, because its a dog-eat-dog world.
C) You need to nip it in the butt, because its a dog-eat-cat world.
D) You need to nip it in the butt, because its a doggy-dog world.

4)
A) The statue of limitations had just expired, so he got off scott free.
B) The statue of limitations had just expired, so he got off scot free.
C) The statute of limitations had just expired, so he got off scott free.
D) The statute of limitations had just expired, so he got off scot free.
E) All of the sentences are incorrect.

ANSWERS
1. D) An individual named Scott did not get punished for a crime the he
committed.
2. D) For all practical purposes, the writer is completely apathetic about the
situation at hand.

Take your S AT prep to the next level. Visit www.learnerator.com


321

322

3. B) You need to stop the undesirable behavior, because the world is an
extremely competitive place.
4. D) The deadline for filing a lawsuit has passed, so the individual will not be
punished for a crime that he has committed.

Take your S AT prep to the next level. Visit www.learnerator.com


322

323

Conventions of Punctuation
There are many types of uses for the various forms of punctuation in the English
language. There are also many times when these punctuation marks are misused.
The more popular types of these misuses will now be discussed.

End of sentence punctuation


There are three different punctuation marks used at the ends of sentences. They
are the period (declarative and imperative), the exclamation point (exclamatory
and imperative), and the question mark (interrogative). Using a different
punctuation mark at the end of a sentence can drastically alter its meaning.

I went to the store. (declarative sentence - states a fact)

I went to the store! (exclamatory sentence - expresses extreme emotion)

I went to the store? (interrogative sentence - asks a question)

In addition, there are also imperative and imperative/exclamatory hybrid


sentences.

Go to the store. (imperative sentence - a command)

Go to the store! (imperative/exclamatory - a command expressing emotion)

There are many instances when the context requires that a sentence uses a
specific form of punctuation, or else the intended meaning may be altered.
Incorrect Usages
1. Did he go to the store.
2. "I went to the store!" he whispered quietly so that no one would hear.
3. Will she go to the store after she gets back from work.

Take your S AT prep to the next level. Visit www.learnerator.com


323

324

Correct Usages
1. Did he go to the store?
2. "I went to the store," he whispered quietly so that no one would hear.
3. Will she go to the store after she gets back from work?
In the first example above, the sentence is worded in a way that only allows it to
be an interrogative sentence; therefore, it MUST end with a question mark. In the
second example above, the context makes it clear that the exclamation point is
inappropriate for the quoted material within the sentence. In the final example,
the sentence is constructed in a manner that requires a question mark at the end.

Punctuation within the sentence


There are three types of punctuation used to indicate sharp breaks of thought
within a sentence: colons, semicolons, and em dashes.
Colons

Colons are used immediately after independent clauses in order to present a list,
an individual thing, or a quote.

He wanted three things from the store: grapes, nuts, and avocadoes.

John said it the best: "I don't care if you care about me caring."

The was only one thing that mattered: his money.

By far, the most common colon error is committed when the colon does not
follow an independent clause.
Incorrect Usage
1. The dessert was made from: bananas, cherries, and oranges.
2. The one thing we can do is: wait for the firemen to arrive.
3. Henry spoke these words and said that: "I am waiting for you to wait for me.

Take your S AT prep to the next level. Visit www.learnerator.com


324

325

Correct Usage
1. The dessert was made from three different fruits: bananas, cherries, and
oranges.
2. There is one thing we can do: wait for the firemen to arrive.
3. Henry spoke these words: "I am waiting for you to wait for me."
In all of the above examples, the errors are corrected by reconstructing the
sentence so that the colon follows an independent clause that can stand alone as
a complete sentence.
Semicolons

There are two uses for semicolons: to connect two independent clauses and to
separate lists of three or more items if there are commas used in the components
of the list. This second type of usage will be discussed in an upcoming portion of
this article.
Semicolons can connect two independent clauses and can be used to correct
comma splices. The two clauses that are connected should be related.

He fled the country because he lost the election; he feared for his life.

The little kitty was playing in the grass; it was so cute.

The most common semicolon error occurs when at least one of the items that it
connects is not a complete independent clause.
Incorrect Usage
1. John knew that he was funny; people laughing at his jokes.
2. The way to the store; five blocks to the south.
Correct Usage
1. John knew that he was funny; people were always laughing at his jokes.
2. The way to the store was obvious; it was five blocks to the south.
Take your S AT prep to the next level. Visit www.learnerator.com
325

326

In the first example above, the final clause of the sentence is only a phrase; it is
corrected by converting it into an independent clause. In the second example
above, both elements of the sentence are phrases and need to be converted into
independent clauses.
Em dashes

Em dashes are used to distinguish parenthetical expressions from the rest of the
sentence. Commas and parenthesis are also used to perform this function.

The boy was not scared his father was only minutes away.

Kate joined in on the fun she had just finished her chores.

This usage of em dashes, parenthesis, and commas will be discussed in more


detail in an upcoming section of this article.

Possessive Nouns and Pronouns


When a noun or pronouns is possessive, an apostrophe and the letter s are
attached to the end of the word. When a word is plural, then an apostrophe is
added to the end of the word. Collective nouns (herd, flock, crowd, etc.) are
singular.

It was John's fault.

They loved all of the oranges' flavors; there were two dozen of them in the
bag.

The crowd's applause was deafening.

It is also important to note that there is no strict rule for adding an apostrophe
and the letter s or just an apostrophe to singular nouns that end with the letter s.
Thus, all of the sentences below could be considered correct depending on which
authority is consulted.

Take your S AT prep to the next level. Visit www.learnerator.com


326

327

It was Mrs. Jones's cat.

It was Mrs. Jones' cat.

The canvas's size was enormous.

The canvas' size was enormous.

The most common error with apostrophes occurs when a collective noun is
treated as though it were plural. Also, many errors occur when a plural noun is
pluralized as though it is singular.
Incorrect Usage
1. The ferocity of the herds' members was incredible.
2. The apple's colors ranged from light green to dark red.
Correct Usage
1. The ferocity of the herd's members was incredible.
2. The apples' colors ranged from light green to dark red.
In the first example, the word herd is a collective noun that needs to be treated
like a singular noun when it is converted into possessive form; thus, an
apostrophe followed by the letter s is needed in order for the sentence to be
correct. In the last example, the word apples is a plural noun and only needs an
apostrophe at the end in order to be correct.

Items in a series
The various components of lists of two or more items need to be separated with
commas. If the Oxford (serial) comma is used, then the final item of a list of three
or more items also needs to be separated with a comma just before the
conjunction near the end of the list. The majority of most English writers now
utilize the Oxford comma.

Take your S AT prep to the next level. Visit www.learnerator.com


327

328

She loved to play with puppies, kitties, and bunnies. (with the Oxford comma)

She loved to play with puppies, kitties and bunnies. (without the Oxford
comma)

The sample essay on the official ACT website uses the Oxford comma in the
second and final paragraphs. http://www.actstudent.org/writing/sample/six.html
Semicolons are used to separate the items in a list when the various items in the
list contain commas.

He loved to eat sweet, ripe oranges; small, delicate peas; and big, juicy steaks.

They had lived in San Diego, California; Portland, Oregon; and Houston,
Texas.

A common error occurs when the Oxford comma is used with a list of two that
employs a conjunction or when a comma is omitted in a list of two items that
does not use a conjunction. In addition, a semicolon is often incorrectly used to
separate the items of a list when a comma is required.
Incorrect Usage
1. The man liked to eat bananas, and peaches.
2. She loved sweet chewy raisins.
3. The girl liked to draw pictures of monkeys, horses; whales, dogs, and cats.
Correct Usage
1. The man liked to eat bananas and peaches.
2. She loved sweet, chewy raisins. OR She loved sweet and chewy raisins.
3. The girl liked to draw pictures of monkeys, horses, whales, dogs, and cats.
In the first example, the comma is unnecessary. In the second example, the list of
two can be separated with either a conjunction or a comma. In the last example,
the semicolon needs to be replaced with a comma.
Take your S AT prep to the next level. Visit www.learnerator.com
328

329

Nonrestrictive and parenthetical elements
Nonrestrictive and parenthetical elements are set off from the rest of the sentence
by using parentheses, em dashes, or commas. Parenthesis are the strongest
method of setting of such elements that have little if any relation to the rest of the
sentence. Em dashes are not quite as strong as parenthesis and are usually used
for material that is at least somewhat relevant to the context of the sentence.
Commas are the weakest and set off material that is very relevant to the sentence.

The mouse played (although it had been distracted numerous times) with the
cheese.

The mouse played it was quite fond of playing with its food with the cheese.

The mouse played, and kept playing, with the cheese.

In the above examples, all of the parenthetical elements have been correctly set
off from the sentences by using the three types of punctuation used for this type
of situation.
Incorrect Usage

The mouse (in the garage) was killed by a mousetrap.

His cousin Bob was a very large man.

Correct Usage

The mouse in the garage was killed by a mousetrap.

His cousin Bob was a very large man. (If subject has more than one cousin)

His cousin, Bob, was a very large man. (If subject has only one cousin)

In the first example, the prepositional phrase in the garage is a restrictive element of
the sentence and should not be set off with any type of punctuation. In the
second example, comma usage is determined by how many cousins the subject

Take your S AT prep to the next level. Visit www.learnerator.com


329

330

has. If the subject more than one cousin, then the word Bob is a restrictive element
and should not be separated with commas. If the subject has only one cousin,
then the word Bob is an nonrestrictive element and needs to be set off with
commas.
Restrictive or essential elements

Restrictive elements are necessary elements of a sentence that are needed in


order for the meaning of the sentence to be perfectly clear. They are not
separated from the rest of the sentence with commas.

The store that was near the pizza restaurant was burned down.

A woman who was in the store saw the arsonist.

In the first example above, the phrase that was near the pizza store tells the reader
which particular store was burned down. In the second example above, the
phrase who was in the store lets the reader know which woman saw the arsonist. Both
of these phrases are essential so that the reader has all of the detail needed to
understand the concepts expressed by these sentences.
Incorrect Examples
1. The security guard, who worked the night shift at the mall, was fat.
2. The mall, which was near the city zoo, was being renovated since the other
malls in town were doing much better.
Correct Examples
1. The security guard who worked the night shift at the mall was fat.
2. The mall that was near the city zoo was being renovated since the other malls
in town were doing much better.

Take your S AT prep to the next level. Visit www.learnerator.com


330

331

In the first example above, the phrase who worked the night shift at the mall is an essential
element that lets the reader know which security guard is the subject of the
sentence; therefore, it should not be separated from the rest of the sentences with
commas. In the second example above, the phrase which was near the city zoo is a
restrictive element that lets the reader know which mall is being talked about in
the sentence; thus, it should not be separated from the rest of the sentence with
commas.

Unnecessary punctuation
There are many ways that punctuation can be used incorrectly in a sentence;
however, there are a few situations that are very common. These more popular
errors will now be discussed.
Unnecessary apostrophes

Oftentimes, a writer will add an apostrophe when creating the plural form of a
word.
Incorrect Examples
1. The girl's played in the backyard.
2. There were many different kinds of football's at the store.
Correct Examples
1. The girls played in the backyard.
2. There were many different kinds of footballs at the store.
In the first example above, the word girls has an unnecessary apostrophe. In order
to fix this sentence, the apostrophe needs to be removed. In the second example,
the word footballs contains an unnecessary apostrophe that needs to be removed.

Take your S AT prep to the next level. Visit www.learnerator.com


331

332

Unnecessary quotation marks

A writer will often use quotation marks on material that is not a direct quote.
These unnecessary quotes are placed so that these words will receive more
emphasis.
Incorrect Examples
1. He said that he saw a "light at the end of the tunnel."
2. He said that the "party" which he attended last night was very lame.
Correct Examples
1. He said that he saw a light at the end of the tunnel.
2. He said that the party which he attended last night was very lame.
In the above examples, there are no direct quotations; however, quotations marks
are used to add emphasis. Merely remove these unnecessary quotation marks,
and both sentences will be grammatically correct.
Incorrect comma usage

Commas are frequently misused by either omitting them in necessary places or


by placing them in unnecessary places.
Commas are often omitted in necessary places when they are not placed before a
coordinating conjunction in a compound sentence. This is probably the most frequent
mistake made by writers of the English language. If

the writer does not have the seven

coordinating conjunctions memorized, then this mistake will continue


throughout everything that a writer creates. However, if the independent clauses
are very short, then the comma becomes optional.

He decided to go to the store, and then he decided to go to the movies.

She went to the downtown museum, so she could photograph some of the
art.

Take your S AT prep to the next level. Visit www.learnerator.com


332

333

He slept so she cooked breakfast.

In addition, writers often omit commas after the end of a dependent clause that
begins a complex sentence. These commas are very necessary in order for a
sentence to maintain clarity.

After she went to the store, she went to the movies.

Before he went to the movies, he took a walk in the park.

Writers will often place unnecessary commas in sentences. This situation usually
occurs when a writer places a comma before a second verb in a sentence that has
only one subject; however, this sort of error also occurs when a writer places a
comma before a subordinating conjunction that precedes a subordinating clause
at the end of a complex sentence.

She went to the store and then took a walk in the park.

He went to the museum before he went to the store.

In the first example above, there is only one subject, so there is no need for a
comma before the conjunction and. In the second example, the conjunction before is
a subordinating conjunction and does not require a comma directly before it.
Incorrect Usages
1. He went to the downtown zoo and then he went to the pet store.
2. He went to the downtown zoo, and then went to the pet store.
3. After she went to the downtown zoo she went to the pet store.
4. She went to the downtown zoo, after she went to the pet store.
Correct Usages
1. He went to the downtown zoo, and then he went to the pet store.
2. He went to the downtown zoo and then went to the pet store.
Take your S AT prep to the next level. Visit www.learnerator.com
333

334

3. After she went to the downtown zoo, she went to the pet store.
4. She went to the downtown zoo after she went to the pet store.
In the first example, two independent clauses are connected by the coordinating
conjunction and; therefore, a comma is required before the word and. In the second
example, there is only one subject, so there is no need for a comma before the
word and.
In the third example, a subordinating clause begins a complex sentence;
therefore, a comma is needed after the clause. In the last example, a
subordinating clause ends a complex sentence; therefore, no comma is needed.
In order to know when to correctly use commas in compound and complex
sentences, the writer MUST have the seven coordinating conjunctions
memorized. The seven coordinating conjunctions are for, and, nor, but, or, yet,
and so. These seven words can be easily memorized by using the acronym
FANBOYS.

All other conjunctions that are commonly used to connect sentences are

subordinating conjunctions.
Important exception

It is imperative to note that the word so is a coordinating conjunction; however,


when it is combined with the word that, it becomes a two-word subordinating
conjunction. Therefore, correct comma placement is indicated in these two
examples.

Rebecca drove to the lake so that she could see her uncle.

Rebecca drove to the lake, so she could see her uncle.

Excess exclamation points

Only one exclamation point is needed at any single point in a sentence. To use
more than one at a single location is grammatically incorrect.
Incorrect Usage
1. They went to the store, and then they went to the movies!!!!
Take your S AT prep to the next level. Visit www.learnerator.com
334

335

2. "Hey!!!" she yelled. "Get over here right now!!"
Correct Usage
1. They went to the store, and then they went to the movies!
2. "Hey!" she yelled. "Get over here right now!"
In the examples above, multiple exclamation points are used at single locations.
In order to correct these sentences, omit the extra exclamation points.
Incorrect quotation mark placement

Oftentimes, a writer will incorrectly place punctuation outside of a set of


quotation marks at the end of a quote.
Incorrect Usage
1. "Hello there," he said. "Let's go to the store".
2. "Hey there"! she said. "Let's go to the mall."
Correct Usage
1. "Hello there," he said. "Let's go to the store."
2. "Hey there!" she said. "Let's go to the mall."
In the first example above, a period is placed outside of the quotation marks;
therefore, in order to correct the sentence, the period should be placed within the
quotes. In the second example, an exclamation point is placed outside of the
quotes; therefore, it needs to be placed inside the quotes so that the sentence will
be correct.
Hyphen, En dash, and Em dash usage

There is much confusion over these three closely related punctuation marks, and
they are commonly confused an a wide variety of ways. Therefore, the specific

Take your S AT prep to the next level. Visit www.learnerator.com


335

336

usages for each type of mark will be presented so that a comprehensive
understanding may be gained.
The hyphen is used to connect two adjectives in a way that allows for a
combined meaning of the two words. It is also used to connect two or three
words that have been combined to produce a single idea.

It was a low-budget project that required clear-headed thinking.

Her mother-in-law had just bought an up-to-date gardening manual.

The hyphen is also used to connect numbers and fractions as words. It is also
used after the prefixes ex-, self-, and all-.

One-third of twenty-one is seven.

His ex-wife has an all-inclusive plan for self-righteousness.

The en dash is used to connect two items that are related to each other by
distance.

He read pages 34 56

He read from the September December issue of the journal.

The em dash is used to set off nonrestrictive elements in a sentence. This welldefined usage as well as its relationship to parentheses and commas was
explained on pages 4 5 of this article.
Incorrect Usages
1. He thought that he was finished with pages 56-58 of the script.
2. Doug bred rabbits in the fall-and guinea pigs in the spring-on his farm.
3. His ex wife was very upset with him.
4. They found thirty four errors in the text.
Correct Usages
Take your S AT prep to the next level. Visit www.learnerator.com
336

337

1. He thought that he was finished with pages 56 58 of the script.
2. Doug bred rabbits in the fall and guinea pigs in the spring on his farm.
3. His ex-wife was very upset with him.
4. They found thirty-four errors in the text.
In the first example, a hyphen is used to connect two items that are related by
distance; thus, the sentence requires an en dash in order to be correct. In the
second example, hyphens are used to set off a parenthetical expression from the
rest of the sentence; thus, en dashes may be used in order to make this sentence
correct. In the third example, an em dash is used in a common expression that
requires a hyphen. In the final example, an en dash is used for a written
expression of numbers; therefore, a hyphen needs to be used.
Sample Test Questions

1) Identify which sentence contains an INCORRECT usage of punctuation.


A) Her mother read the book.
B) Her mother read the book?
C) Her mother read the book
D) Her mother read the book!
2) Identify which sentence contains an INCORRECT usage of punctuation.
A) Elizabeth only wanted three things: money, fun, and candy.
B) She knew what she wanted; she spent her time acquiring them.
C) There was no time for amateurs; however, they insisted on being there.
D) The middle of the night; it was a time for reflection.
3) Identify which sentence contains an INCORRECT usage of punctuation.

Take your S AT prep to the next level. Visit www.learnerator.com


337

338

A) It was John's fault.
B) It was Johns fault.
C) It is John's fault.
D) It is his fault.
4) Identify which sentence contains an INCORRECT usage of punctuation.
A) They wanted beets, yams, and radishes.
B) They wanted beets; yams and radishes.
C) They wanted beets, yams, radishes, and cucumbers.
D) They wanted three things: beets, yams, and radishes.
5) Identify which sentence contains an INCORRECT usage of punctuation.
A) The cat over by the window was the cutest cat in the house.
B) The cat, over by the window, was the only cat in the house.
C) The cat was the cutest cat in the house.
D) The cat over by the window was the only cat in the house.
E) The cat was the only cat in the house.
6) Identify which sentence contains an INCORRECT usage of punctuation.
A) The girl's played in the field.
B) The girls played in the field.
C) The girls played and played in the field.
D) The girls played.
E) The girls played baseball in the field.
ANSWERS

Take your S AT prep to the next level. Visit www.learnerator.com


338

339

1. C) This sentence needs some type of punctuation at its end.
2. D) The first section of this sentence is not an independent clause; therefore, it
cannot be separated by another independent clause with a semicolon.
3. B) The word Johns is the possessive form of a proper noun and requires an
apostrophe before the letter s.
4. B) A semicolon is used incorrectly in a list of three items.
5. D) Since the cat is the only cat in the house, the phrase over by the window is
nonrestrictive and should be separated from the rest of the sentence with
commas.
6. A) The word girl's is not possessive and does not need an apostrophe.

Take your S AT prep to the next level. Visit www.learnerator.com


339

340

Sign up for Learnerator SAT Prep!



Well give you everything you need to start mastering the SAT today.
Our online test prep offers:

Over 300 SAT Integrated Reading & Writing practice questions

Over 300 SAT Math practice questions

Hundreds of practice questions for SAT IIs: Biology, Chemistry, French,


Literature, Math 1 & 2, Physics, US History, World History

Detailed personalized statistics based on your performance

A competitive online leaderboard to see how you stand compared to


others

Convenience with anytime access from an Internet-enabled device


Visit http://www.learnerator.com
Use SAT2016 for 10% off any Learnerator SAT Subject Guide


Take your S AT prep to the next level. Visit www.learnerator.com
340

Potrebbero piacerti anche